0% found this document useful (0 votes)
24 views178 pages

Math

The document is a comprehensive guide on Integral Calculus, covering topics such as indefinite and definite integrals, techniques of integration, and applications of integrals. It includes chapters on properties, rules, and practical examples to illustrate the concepts. The content is structured with exercises and a bibliography for further study.

Uploaded by

serhas2539
Copyright
© © All Rights Reserved
We take content rights seriously. If you suspect this is your content, claim it here.
Available Formats
Download as PDF, TXT or read online on Scribd
0% found this document useful (0 votes)
24 views178 pages

Math

The document is a comprehensive guide on Integral Calculus, covering topics such as indefinite and definite integrals, techniques of integration, and applications of integrals. It includes chapters on properties, rules, and practical examples to illustrate the concepts. The content is structured with exercises and a bibliography for further study.

Uploaded by

serhas2539
Copyright
© © All Rights Reserved
We take content rights seriously. If you suspect this is your content, claim it here.
Available Formats
Download as PDF, TXT or read online on Scribd
You are on page 1/ 178

‫َاز َل‬ ‫ضيَا ًء َو ْالقَ َم َر نُ ً‬

‫ورا َوقَد ََّرهُ َمن ِ‬ ‫س ِ‬ ‫﴿ ُه َو الَّذِي َج َع َل ال َّ‬


‫ش ْم َ‬
‫َّللاُ ذَ ِل َك ِإ َّّل‬
‫اب َما َخلَقَ َّ‬
‫س َ‬‫السنِينَ َو ْال ِح َ‬ ‫ِلتَ ْعلَ ُموا َ‬
‫عدَدَ ِ‬

‫ص ُل ْاْل َيا ِ‬
‫ت ِلقَ ْو ٍم َي ْعلَ ُمونَ ﴾‪1‬‬ ‫ِب ْال َح ِ‬
‫ق يُفَ ِ‬

‫‪1‬‬
‫]يونس‪[5 :‬‬
Contents
Chapter 1. INDEFINITE INTEGRAL ...............................................................1

1.1 Introduction ..................................................................................................1

1.2 Integration as an Inverse Process of Differentiation ....................................2

1.3 Properties of the Indefinite Integral .............................................................6

1.4 Basic Rules of Integration ............................................................................7


1.4.1 Integration of Trigonometric and Hyperbolic Integrals ........................................................ 15
1.4.2 Integration of Inverse Trigonometric and Inverse Hyperbolic Integrals ............................... 21

Exercises (1) .........................................................................................................24

Chapter 2. TECHNIQUES OF INTEGRATION ............................................31

2.1 The Substitution Rule .................................................................................31

2.2 Trigonometric Substitution ........................................................................40

2.3 Trigonometric Integrals ..............................................................................47


2.3.1 Integrating a Product of Powers of Sine and Cosine ............................................................. 47
2.3.2 Integrating a Product of Powers of Secants and Tangents ................................................... 53

2.4 Integration by Parts ....................................................................................57


2.4.1 Integration by Reduction ...................................................................................................... 70

2.5 Integration of Rational Functions by Partial Fractions ..............................76


2.5.1 CASE I: The denominator Q(x) is a product of distinct linear factors.................................... 78
2.5.2 CASE II: Q(x) is a product of linear factors, some of which are repeated ............................. 82
2.5.3 CASE III: Q(x) contains irreducible quadratic factors, none of which is repeated ................ 84
2.5.4 CASE IV: Q(x) contains a repeated irreducible quadratic factor ........................................... 86

2.6 Integrals Involving Quadratic Expressions ................................................89

Exercises (2) .........................................................................................................99

Chapter 3. DEFINITE INTEGRALS..............................................................106

i
3.1 The Fundamental Theorem of Calculus ...................................................107

3.2 Properties of Definite Integrals ................................................................111

3.3 Improper Integrals ....................................................................................121


3.3.1 Type 1: Infinite Intervals...................................................................................................... 122
3.3.2 Type 2: Discontinuous Integrands ....................................................................................... 129

Exercises (3) .......................................................................................................133

Chapter 4. APPLICATIONS OF INTEGRATIONS ....................................139

4.1 Areas .........................................................................................................140


4.1.1 Area under Simple Curves ................................................................................................... 140
4.1.2 Areas Between Curves ........................................................................................................ 148

4.2 Volumes....................................................................................................155
4.2.1 Slicing by Parallel Planes ..................................................................................................... 156
4.2.2 Solids of Revolution: The Disk Method ............................................................................... 158
4.2.3 Solids of Revolution: The Washer Method ......................................................................... 163

Exercises (4) .......................................................................................................166

Bibliography .........................................................................................................171

ii
iii
Chapter 1: INDEFINITE INTEGRAL

Chapter 1. INDEFINITE INTEGRAL

1.1 Introduction

Differential Calculus is centered on the concept of the derivative. The original


motivation for the derivative was the problem of defining tangent lines to the graphs
of functions and calculating the slope of such lines. Integral Calculus is motivated
by the problem of defining and calculating the area of the region bounded by the
graph of the functions. If a function 𝑓 is differentiable in an interval 𝐼, i.e., its
derivative 𝑓′ exists at each point of 𝐼, then a natural question arises that given 𝑓′ at
each point of 𝐼, can we determine the function? The functions that could possibly
have given function as a derivative are called anti-derivatives (or primitive) of the
function. Further, the formula that gives all these anti-derivatives is called the
indefinite integral of the function and such process of finding anti-derivatives is
called integration. Such type of problems arises in many practical situations. For
instance, if we know the instantaneous velocity of an object at any instant, then there
arises a natural question, i.e., can we determine the position of the object at any
instant? There are several such practical and theoretical situations where the process
of integration is involved. The development of integral calculus arises out of the
efforts of solving the problems of the following types:

(a) the problem of finding a function whenever its derivative is given,

(b) the problem of finding the area bounded by the graph of a function under certain

conditions.

These two problems lead to the two forms of the integrals, e.g., indefinite and
definite integrals, which together constitute the Integral Calculus.

1
Chapter 1: INDEFINITE INTEGRAL
There is a connection, known as the Fundamental Theorem of Calculus, between

indefinite integral and definite integral which makes the definite integral as a
practical tool for science and engineering. The definite integral is also used to solve
many interesting problems from various disciplines like economics, finance and
probability.

1.2 Integration as an Inverse Process of Differentiation

Integration is the inverse process of differentiation. Instead of differentiating a


function, we are given the derivative of a function and asked to find its primitive,
i.e., the original function. Such a process is called integration or anti differentiation.

Let us consider the following examples:

We know that

𝑑
(sin 𝑥) = cos 𝑥 (1)
𝑑𝑥

𝑑 𝑥3
( ) = 𝑥2 (2)
𝑑𝑥 3

and

𝑑 𝑥
(𝑒 ) = 𝑒 𝑥 (3)
𝑑𝑥

We observe that in (1), the function cos 𝑥 is the derived function of sin 𝑥. We say

that sin 𝑥 is an anti-derivative (or an integral) of cos 𝑥.

2
Chapter 1: INDEFINITE INTEGRAL
𝑥3
Similarly, in (2) and (3), and 𝑒 𝑥 are the anti-derivatives (or integrals) of 𝑥 2 and
3

𝑒 𝑥 , respectively. Again, we note that for any real number 𝐶, treated as constant
function, its derivative is zero and hence, we can write (1), (2) and (3) as follows:

𝑑
(sin 𝑥 + 𝐶) = cos 𝑥
𝑑𝑥

𝑑 𝑥3
( + 𝐶) = 𝑥 2
𝑑𝑥 3

and

𝑑 𝑥
(𝑒 + 𝐶) = 𝑒 𝑥
𝑑𝑥

Thus, anti-derivatives (or integrals) of the above cited functions are not unique.
Actually, there exist infinitely many anti-derivatives of each of these functions
which can be obtained by choosing 𝐶 arbitrarily from the set of real numbers. For
this reason, 𝐶 is customarily referred to as arbitrary constant. In fact, 𝐶 is the
parameter by varying which one gets different anti derivatives (or integrals) of the
given function.

More generally, if there is a function 𝐹 such that

𝑑
𝐹(𝑥) = 𝑓(𝑥), ∀ 𝑥 ∈ 𝐼 (interval),
𝑑𝑥

then for any arbitrary real number 𝐶, (also called constant of integration)

𝑑
[𝐹(𝑥) + 𝐶] = 𝑓(𝑥), 𝑥∈𝐼
𝑑𝑥

Thus, {𝐹 + 𝐶, 𝐶 ∈ 𝐑} denotes a family of anti-derivatives of 𝑓.

3
Chapter 1: INDEFINITE INTEGRAL
We introduce a new symbol, namely, ∫ 𝑓 (𝑥) 𝑑𝑥 which will represent the entire
class of anti-derivatives read as the indefinite integral of 𝑓 with respect to 𝑥.

Symbolically, we write

∫ 𝑓 (𝑥) 𝑑𝑥 = 𝐹(𝑥) + 𝐶.

Note: Given that


𝑑𝑦
= 𝑓(𝑥),
𝑑𝑥
We write

𝑦 = ∫ 𝑓 (𝑥)𝑑𝑥.

We already know the formulae for the derivatives of many important functions.

From these formulae, we can write down immediately the corresponding formulae

(referred to as standard formulae) for the integrals of these functions, as listed


below which will be used to find integrals of other functions.

4
Chapter 1: INDEFINITE INTEGRAL

5
Chapter 1: INDEFINITE INTEGRAL
1.3 Properties of the Indefinite Integral

Our first properties of antiderivatives follow directly from the simple constant factor,
sum, and difference rules for derivatives.

Suppose that 𝐹(𝑥) and 𝐺(𝑥) are antiderivatives of 𝑓(𝑥) and g(𝑥), respectively, and
that 𝑐 is a constant. Then:

1. A constant factor can be moved through an integral sign; that is,

∫ 𝑐𝑓(𝑥) 𝑑𝑥 = 𝑐 ∫ 𝑓(𝑥) 𝑑𝑥 = 𝑐𝐹(𝑥) + 𝐶.

2. An antiderivative of a sum (difference) is the sum (difference) of the


antiderivatives; that is,

∫[𝑓(𝑥) ± g(𝑥)] 𝑑𝑥 = ∫ 𝑓(𝑥) 𝑑𝑥 ± ∫ g(𝑥) 𝑑𝑥 = 𝐹(𝑥) ± 𝐺(𝑥) + 𝐶.

3. Integration cannot be distributed over multiplication or division.

∫[𝑓(𝑥) ∙ g(𝑥)] 𝑑𝑥 ≠ ∫ 𝑓(𝑥) 𝑑𝑥 ∙ ∫ g(𝑥) 𝑑𝑥

𝑓(𝑥) ∫ 𝑓(𝑥) 𝑑𝑥
∫ 𝑑𝑥 ≠
g(𝑥) ∫ g(𝑥) 𝑑𝑥

Example 1: Evaluate the integrals

(𝐚) ∫ 4 cos 𝑥 𝑑𝑥. (𝐛) ∫(𝑥 + 𝑥 2 )𝑑𝑥. (𝐜) ∫ 3𝑥 7 𝑑𝑥.

Solution:

6
Chapter 1: INDEFINITE INTEGRAL
(𝐚) ∫ 4 cos 𝑥 𝑑𝑥 = 4 ∫ cos 𝑥 𝑑𝑥 = 4 sin 𝑥 + 𝐶.

2 )𝑑𝑥 2
𝑥2 𝑥3
(𝐛) ∫(𝑥 + 𝑥 = ∫ 𝑥 𝑑𝑥 + ∫ 𝑥 𝑑𝑥 = + + 𝐶.
2 3

3
(𝐜) ∫ 3𝑥 7 𝑑𝑥 = 3 ∫ 𝑥 7 𝑑𝑥 = 𝑥 8 + 𝐶.
8

Example 2: Evaluate the integral

∫(3𝑥 6 − 2𝑥 2 + 7𝑥 + 1)𝑑𝑥.

Solution:

∫(3𝑥 6 − 2𝑥 2 + 7𝑥 + 1)𝑑𝑥 = 3 ∫ 𝑥 6 𝑑𝑥 − 2 ∫ 𝑥 2 𝑑𝑥 + 7 ∫ 𝑥 𝑑𝑥 + ∫ 1 𝑑𝑥

3𝑥 7 2𝑥 3 7𝑥 2
= − + + 𝑥 + 𝐶.
7 3 2

Note: When finding the indefinite integral of a function 𝑓, remember that it always
includes an arbitrary constant 𝐶.

1.4 Basic Rules of Integration

Now we will apply some basic rules of integration and generalize them.

𝑛
𝑥 𝑛+1 𝑛 ′ (𝑥)𝑑𝑥
[𝑓(𝑥)]𝑛+1
∫ 𝑥 𝑑𝑥 = + 𝐶, ∫[𝑓(𝑥)] 𝑓 = +𝐶
𝑛+1 𝑛+1

7
Chapter 1: INDEFINITE INTEGRAL
Example 3: Evaluate the integrals

𝟏. ∫[5𝑥 3 − 10𝑥 2 + 2𝑥 − 3]𝑑𝑥 = 5 ∫ 𝑥 3 𝑑𝑥 − 10 ∫ 𝑥 2 𝑑𝑥 + 2 ∫ 𝑥 𝑑𝑥 − ∫ 3 𝑑𝑥

5𝑥 4 10𝑥 3
= − + 𝑥 2 − 3𝑥 + 𝐶.
4 3

1 2 1
𝟐. ∫ (𝑥 + 3 ) 𝑑𝑥 = ∫ (𝑥 6 + 2 + 6 ) 𝑑𝑥
3
𝑥 𝑥

6 −6 )
𝑥7 𝑥 −5
= ∫(𝑥 + 2 + 𝑥 𝑑𝑥 = + 2𝑥 − + 𝐶.
7 5
1 7
3
𝟑. ∫(𝑥 + √𝑥 )(4 − 𝑥 2 )𝑑𝑥 = ∫ (4𝑥 − 𝑥 3 + 4𝑥 3 − 𝑥 3 ) 𝑑𝑥

2
𝑥4 4 3 10
= 2𝑥 − + 3𝑥 − 𝑥 3 + 𝐶.
3
4 10

4𝑥 10 − 2𝑥 4 + 15𝑥
𝟒. ∫ 𝑑𝑥 = ∫(4𝑥 7 − 2𝑥 + 15𝑥 −2 ) 𝑑𝑥
𝑥3

𝑥8 15
= − 𝑥2 − + 𝐶.
2 𝑥

1
10 10
1 (2𝑥 − 1)11
𝟓. ∫(2𝑥 − 1) 𝑑𝑥 = ∫ 2(2𝑥 − 1) 𝑑𝑥 = + 𝐶.
2 2 11

8
Chapter 1: INDEFINITE INTEGRAL
1
𝟔. ∫ sin5 𝑥 cos 𝑥 𝑑𝑥 = sin6 𝑥 + 𝐶.
6

(1 + tan 𝑥)2 2
1
𝟕. ∫ 𝑑𝑥 = ∫(1 + tan 𝑥) ∙ 𝑑𝑥
cos 2 𝑥 cos 2 𝑥

= ∫(1 + tan 𝑥)2 ∙ sec 2 𝑥 𝑑𝑥

1
= (1 + tan 𝑥)3 + 𝐶.
3

ln 𝑥 1 1
𝟖. ∫ 𝑑𝑥 = ∫ ln 𝑥 ∙ 𝑑𝑥 = (ln 𝑥)2 + 𝐶.
𝑥 𝑥 2
3
√1 + ln 𝑥 1 1 3 4
𝟗. ∫ 𝑑𝑥 = ∫(1 + ln 𝑥)3 ∙ 𝑑𝑥 = (1 + ln 𝑥)3 + 𝐶.
𝑥 𝑥 4

1
𝟏𝟎. ∫ 𝑥 2 √1 − 𝑥 3 𝑑𝑥 = ∫ 𝑥 2 (1 − 𝑥 3 )2 𝑑𝑥

1 1 12 3
= − ∫ −3𝑥 2 (1 − 𝑥 3 )2 𝑑𝑥 = − (1 − 𝑥 3 )2 + 𝐶
3 33

2 3
3 )2
=− (1 −𝑥 + 𝐶.
9

23
𝟏𝟏. ∫ ( 2
− sec 𝑥 tan 𝑥) 𝑑𝑥 = 23 tan−1 𝑥 − sec 𝑥 + 𝐶.
1+𝑥

9
Chapter 1: INDEFINITE INTEGRAL
1 1 √1 + 𝑥 − √𝑥
𝟏𝟐. ∫ ( ) 𝑑𝑥 = ∫ ( )∙ 𝑑𝑥
√1 + 𝑥 + √𝑥 √1 + 𝑥 + √𝑥 √1 + 𝑥 − √𝑥

√1 + 𝑥 − √𝑥
= ∫( ) 𝑑𝑥 = ∫(√1 + 𝑥 − √𝑥) 𝑑𝑥
1+𝑥−𝑥

1 1 2 3 3
= ∫ ((1 + 𝑥)2 − 𝑥 2 ) 𝑑𝑥 = [(1 + 𝑥)2 − 𝑥 2 ] + 𝐶.
3

1
2 )2
1 1
2 )2
√ 2
𝟏𝟑. ∫ 𝑥 1 + 𝑥 𝑑𝑥 = ∫ 𝑥(1 + 𝑥 𝑑𝑥 = ∫ 2𝑥(1 + 𝑥
2
1 2 3 1 3
= ∙ (1 + 𝑥 2 )2 + 𝐶 = (1 + 𝑥 2 )2 + 𝐶.
2 3 3

sin−1 𝑥 1 1
𝟏𝟒. ∫ 𝑑𝑥 = ∫ sin−1 𝑥 ∙ ( ) 𝑑𝑥 = (sin−1 𝑥)2 + 𝐶.
√1 − 𝑥 2 √1 − 𝑥 2 2

𝑑𝑥 1
𝟏𝟓. ∫ = ∫(sin−1 𝑥)−3 ∙ ( ) 𝑑𝑥
(sin−1 𝑥)3 √1 − 𝑥 2 √1 − 𝑥 2
1
= − (sin−1 𝑥)−2 + 𝐶.
2

tan−1 𝑥 −1
1 1
𝟏𝟔. ∫ 2
𝑑𝑥 = ∫ tan 𝑥 ∙ ( 2
) 𝑑𝑥 = (tan−1 𝑥)2 + 𝐶.
1+𝑥 1+𝑥 2

cosh−1 𝑥 1 1
𝟏𝟕. ∫ √ 𝑑𝑥 = ∫(cosh −1
𝑥) 2∙( ) 𝑑𝑥
𝑥2 − 1 √𝑥 2 − 1
2 3
= (cosh−1 𝑥)2 + 𝐶.
3

10
Chapter 1: INDEFINITE INTEGRAL
𝑑𝑥 𝑓 ′ (𝑥)
∫ = 2√𝑥 + 𝐶, ∫ 𝑑𝑥 = 2√𝑓(𝑥) + 𝐶
√𝑥 √𝑓(𝑥)

Example 4: Evaluate the integrals

𝑑𝑥 1 𝑎 𝑑𝑥 2
𝟏. ∫ = ∫ = √𝑎𝑥 + 𝑏 + 𝐶.
√𝑎𝑥 + 𝑏 𝑎 √𝑎𝑥 + 𝑏 𝑎

𝑑𝑥 1 −5 𝑑𝑥 2
𝟐. ∫ =− ∫ = − √2 − 5𝑥 + 𝐶.
√2 − 5𝑥 5 √𝑎𝑥 + 𝑏 5

2𝑥 − 1
𝟑. ∫ 𝑑𝑥 = 2√𝑥 2 − 𝑥 + 1 + 𝐶.
√𝑥 2 −𝑥+1

𝑥 1 2𝑥
𝟒. ∫ 𝑑𝑥 = − ∫ 𝑑𝑥 = −√1 − 𝑥 2 + 𝐶.
√1 − 𝑥 2 2 √1 − 𝑥 2

𝑑𝑥 1 𝑑𝑥 sec 2 𝑥
𝟓. ∫ =∫ ∙ =∫ 𝑑𝑥
cos 2 𝑥 √1 + tan 𝑥 cos 2 𝑥 √1 + tan 𝑥 √1 + tan 𝑥

= 2√1 + tan 𝑥 + 𝐶.

2𝑥 − sin−1 𝑥 2𝑥 sin−1 𝑥
𝟔. ∫ 𝑑𝑥 = ∫ ( − ) 𝑑𝑥
√1 − 𝑥 2 √1 − 𝑥 2 √1 − 𝑥 2

1
= −2√1 − 𝑥 2 − (sin−1 𝑥)2 + 𝐶.
2

11
Chapter 1: INDEFINITE INTEGRAL

∫ 𝑒 𝑥 𝑑𝑥 = 𝑒 𝑥 + 𝐶, ∫ 𝑒 𝑓(𝑥) 𝑓 ′ (𝑥) 𝑑𝑥 = 𝑒 𝑓(𝑥) + 𝐶

Example 5: Evaluate the integrals

1 1
𝟏. ∫ 𝑒 3𝑥 𝑑𝑥 = ∫ 3 𝑒 3𝑥 𝑑𝑥 = 𝑒 3𝑥 + 𝐶.
3 3

1 1
𝟐. ∫ 𝑒 (𝑎𝑥+𝑏) 𝑑𝑥 = ∫ 𝑎 𝑒 (𝑎𝑥+𝑏) 𝑑𝑥 = 𝑒 (𝑎𝑥+𝑏) + 𝐶.
𝑎 𝑎

3 1 3 1 5𝑥 3
𝟑. ∫ 𝑥 2 𝑒 5𝑥 𝑑𝑥 = ∫(15𝑥 2 )𝑒 5𝑥 𝑑𝑥 = 𝑒 + 𝐶.
15 15

2 −4) 1 2 1 2
𝟒. ∫(𝑥 − 2)𝑒 (𝑥 𝑑𝑥 = ∫ 2(𝑥 − 2)𝑒 (𝑥 −4) 𝑑𝑥 = 𝑒 (𝑥 −4) + 𝐶.
2 2

𝑒𝑥
𝟓. ∫ 𝑑𝑥 = 2√1 + 𝑒 𝑥 + 𝐶.
√1 + 𝑒 𝑥

𝑒 2𝑥 − 1 𝑒 2𝑥 1
𝟔. ∫ 𝑥
𝑑𝑥 = ∫ [ 𝑥 − 𝑥 ] 𝑑𝑥 = ∫[𝑒 𝑥 − 𝑒 −𝑥 ]𝑑𝑥 = 𝑒 𝑥 + 𝑒 −𝑥 + 𝐶.
𝑒 𝑒 𝑒

𝟕. ∫ sin 𝑥 𝑒 cos 𝑥 𝑑𝑥 = − ∫(− sin 𝑥)𝑒 cos 𝑥 𝑑𝑥 = −𝑒 cos 𝑥 + 𝐶.

𝟖. ∫ sec 2 𝑥 𝑒 tan 𝑥 𝑑𝑥 = ∫(sec 2 𝑥)𝑒 tan 𝑥 𝑑𝑥 = 𝑒 tan 𝑥 + 𝐶.

12
Chapter 1: INDEFINITE INTEGRAL
𝑥 𝑥
𝑒 𝑥 + 𝑒 −𝑥 𝑒 2𝑥 + 1
𝟗. ∫ 𝑒 cosh 𝑥 𝑑𝑥 = ∫ 𝑒 ( ) 𝑑𝑥 = ∫ ( ) 𝑑𝑥
2 2
1 1 1
= ∫(𝑒 2𝑥 + 1)𝑑𝑥 = [ 𝑒 2𝑥 + 𝑥] + 𝐶.
2 2 2

𝑒 ln 𝑥 1
𝟏𝟎. ∫ 𝑑𝑥 = ∫ 𝑒 ln 𝑥 ( ) 𝑑𝑥 = 𝑒 ln 𝑥 + 𝐶 = 𝑥 + 𝐶.
𝑥 𝑥

2 +ln 𝑥) 2 1 2 1 2
𝟏𝟏. ∫ 𝑒 (2𝑥 𝑑𝑥 = ∫(𝑒 2𝑥 ∙ 𝑒 ln 𝑥 ) 𝑑𝑥 = ∫(𝑒 2𝑥 ∙ 4𝑥) 𝑑𝑥 = 𝑒 2𝑥 + 𝐶.
4 4

√1 − sin 𝑥
𝟏𝟐. ∫ 𝑒 √1+sin 𝑥 √1 − sin 𝑥 𝑑𝑥 = 2 ∫ 𝑒 √1+sin 𝑥 𝑑𝑥 = 2𝑒 √1+sin 𝑥 + 𝐶.
2
Where,

𝑑 cos 𝑥 √1 − sin2 𝑥 √(1 − sin 𝑥)(1 + sin 𝑥)


[√1 + sin 𝑥] = = =
𝑑𝑥 2√1 + sin 𝑥 2√1 + sin 𝑥 2√1 + sin 𝑥
√1 − sin 𝑥
=
2

13
Chapter 1: INDEFINITE INTEGRAL
𝑑𝑥 𝑓′(𝑥)
∫ = ln 𝑥 + 𝐶, ∫ 𝑑𝑥 = ln 𝑓(𝑥) + 𝐶
𝑥 𝑓(𝑥)

Example 6: Evaluate the integrals


𝑥+1 1 4(𝑥 + 1) 1
𝟏. ∫ 𝑑𝑥 = ∫ 𝑑𝑥 = ln(2𝑥 2 + 4𝑥 + 3) + 𝐶.
2𝑥 2 + 4𝑥 + 3 4 2𝑥 2 + 4𝑥 + 3 4

3 4 𝑑𝑥 𝑑𝑥
𝟐. ∫ ( − ) 𝑑𝑥 = 3 ∫ − 4∫
𝑥−1 𝑥−2 𝑥−1 𝑥−2
= 3 ln(𝑥 − 1) − 4 ln(𝑥 − 2) + 𝐶.

sec 2 𝑥
𝟑. ∫ 𝑑𝑥 = ln(1 + tan 𝑥) + 𝐶.
1 + tan 𝑥

sin 𝑥 cos 𝑥 − sin 𝑥


1 − tan 𝑥 1− cos 𝑥 − sin 𝑥
𝟒. ∫ 𝑑𝑥 = ∫ cos 𝑥 𝑑𝑥 = ∫ cos 𝑥 𝑑𝑥 = ∫ 𝑑𝑥
1 + tan 𝑥 sin 𝑥 cos 𝑥 + sin 𝑥 cos 𝑥 + sin 𝑥
1+
cos 𝑥 cos 𝑥
= ln(cos 𝑥 + sin 𝑥) + 𝐶.

1
𝑑𝑥
𝟓. ∫ = ∫ 𝑥 𝑑𝑥 = ln(ln 𝑥) + 𝐶.
𝑥 ln 𝑥 ln 𝑥

1 + ln 𝑥
𝟔. ∫ 𝑑𝑥 = ln(3 + 𝑥 ln 𝑥) + 𝐶.
3 + 𝑥 ln 𝑥

1
𝑑𝑥 𝑥 2 𝑑𝑥 = ln(sin−1 𝑥) + 𝐶.
𝟕. ∫ = ∫ √1 −
√1 − 𝑥 2 sin−1 𝑥 sin−1 𝑥

14
Chapter 1: INDEFINITE INTEGRAL
1.4.1 Integration of Trigonometric and Hyperbolic Integrals

∫ cos 𝑥 𝑑𝑥 = sin 𝑥 + 𝐶. ∫ cos 𝑓(𝑥) 𝑓′(𝑥)𝑑𝑥 = sin 𝑓(𝑥) + 𝐶.

∫ sin 𝑥 𝑑𝑥 = − cos 𝑥 + 𝐶. ∫ sin 𝑓(𝑥) 𝑓 ′ (𝑥)𝑑𝑥 = − cos 𝑓(𝑥) + 𝐶.

∫ sec 2 𝑥 𝑑𝑥 = tan 𝑥 + 𝐶. ∫ sec 2 𝑓(𝑥) 𝑓 ′ (𝑥)𝑑𝑥 = tan 𝑓(𝑥) + 𝐶.

∫ csc 2 𝑥 𝑑𝑥 = − cot 𝑥 + 𝐶. ∫ csc 2 𝑓(𝑥) 𝑓 ′ (𝑥)𝑑𝑥 = −cot 𝑓(𝑥) + 𝐶.

∫ sec 𝑥 tan 𝑥 𝑑𝑥 = sec 𝑥 + 𝐶. ∫ sec 𝑓(𝑥) tan 𝑓(𝑥) 𝑓′(𝑥)𝑑𝑥 = sec 𝑓(𝑥) + 𝐶.

∫ csc 𝑥 cot 𝑥 𝑑𝑥 = −csc 𝑥 + 𝐶. ∫ csc 𝑓(𝑥) cot 𝑓(𝑥) 𝑓′(𝑥)𝑑𝑥 = −csc 𝑓(𝑥) + 𝐶.

∫ cosh 𝑥 𝑑𝑥 = sinh 𝑥 + 𝐶. ∫ cosh 𝑓(𝑥) 𝑓′(𝑥)𝑑𝑥 = sinh 𝑓(𝑥) + 𝐶.

∫ sinh 𝑥 𝑑𝑥 = cosh 𝑥 + 𝐶. ∫ sinh 𝑓(𝑥) 𝑓 ′ (𝑥)𝑑𝑥 = cosh 𝑓(𝑥) + 𝐶.

∫ sech2 𝑥 𝑑𝑥 = tanh 𝑥 + 𝐶. ∫ sech2 𝑓(𝑥) 𝑓 ′ (𝑥)𝑑𝑥 = tanh 𝑓(𝑥) + 𝐶.

∫ csch2 𝑥 𝑑𝑥 = − coth 𝑥 + 𝐶. ∫ csch2 𝑓(𝑥) 𝑓 ′ (𝑥)𝑑𝑥 = −coth 𝑓(𝑥) + 𝐶.

∫ sech 𝑥 tanh 𝑥 𝑑𝑥 = −sech 𝑥 + 𝐶.

∫ sech 𝑓(𝑥) tanh 𝑓(𝑥) 𝑓 ′ (𝑥)𝑑𝑥 = − sech 𝑓(𝑥) + 𝐶.

∫ csch 𝑥 coth 𝑥 𝑑𝑥 = −csch 𝑥 + 𝐶.

∫ csch 𝑓(𝑥) coth 𝑓(𝑥) 𝑓′(𝑥)𝑑𝑥 = −csch 𝑓(𝑥) + 𝐶.

15
Chapter 1: INDEFINITE INTEGRAL
Example 7: Evaluate the integrals

sin 𝑥
𝟏. ∫ tan 𝑥 𝑑𝑥 = ∫ 𝑑𝑥 = − ln(cos 𝑥) + 𝐶
cos 𝑥

1
= ln(cos 𝑥)−1 + 𝐶 = ln + 𝐶 = ln(sec 𝑥) + 𝐶.
cos 𝑥

sinh 𝑥
𝟐. ∫ tanh 𝑥 𝑑𝑥 = ∫ 𝑑𝑥 = ln(cosh 𝑥) + 𝐶.
cosh 𝑥

cos 𝑥
𝟑. ∫ cot 𝑥 𝑑𝑥 = ∫ 𝑑𝑥 = ln(sin 𝑥) + 𝐶.
sin 𝑥

cosh 𝑥
𝟒. ∫ coth 𝑥 𝑑𝑥 = ∫ 𝑑𝑥 = ln(sinh 𝑥) + 𝐶.
sinh 𝑥

sec 𝑥 + tan 𝑥 sec 2 𝑥 + sec 𝑥 tan 𝑥


𝟓. ∫ sec 𝑥 𝑑𝑥 = ∫ sec 𝑥 ∙ 𝑑𝑥 = ∫ 𝑑𝑥
sec 𝑥 + tan 𝑥 sec 𝑥 + tan 𝑥

= ln(sec 𝑥 + tan 𝑥) + 𝐶.

1 1 1
𝟔. ∫ sech 𝑥 𝑑𝑥 = ∫ 𝑑𝑥 = ∫ 𝑥 𝑑𝑥 = 2 ∫ 𝑑𝑥
cosh 𝑥 𝑒 + 𝑒 −𝑥 𝑒 𝑥 + 𝑒 −𝑥
2

1 𝑒𝑥 𝑒𝑥
= 2∫ 𝑥 ∙ 𝑑𝑥 = 2 ∫ 𝑥 2 𝑑𝑥 = 2 tan−1 (𝑒 𝑥 ) + 𝐶.
𝑒 + 𝑒 −𝑥 𝑒 𝑥 (𝑒 ) + 1

16
Chapter 1: INDEFINITE INTEGRAL
csc 𝑥 − cot 𝑥 csc 2 𝑥 − csc 𝑥 cot 𝑥
𝟕. ∫ csc 𝑥 𝑑𝑥 = ∫ csc 𝑥 ∙ 𝑑𝑥 = ∫ 𝑑𝑥
csc 𝑥 − cot 𝑥 csc 𝑥 − cot 𝑥

= ln(csc 𝑥 − cot 𝑥) + 𝐶.

csch 𝑥 − coth 𝑥 csch2 𝑥 − csch 𝑥 coth 𝑥


𝟖. ∫ csch 𝑥 𝑑𝑥 = ∫ csch 𝑥 ∙ 𝑑𝑥 = ∫ 𝑑𝑥
csch 𝑥 − coth 𝑥 csch 𝑥 − coth 𝑥

= ln(csch 𝑥 − coth 𝑥) + 𝐶.

1 1
𝟗. ∫ 𝑥 3 cos 𝑥 4 𝑑𝑥 = ∫ 4 𝑥 3 cos 𝑥 4 𝑑𝑥 = sin 𝑥 4 + 𝐶.
4 4

cos √𝑥 1
𝟏𝟎. ∫ 𝑑𝑥 = 2 ∫ cos √𝑥 ∙ 𝑑𝑥 = 2 sin √𝑥 + 𝐶.
√𝑥 2 √𝑥

𝟏𝟏. ∫ 𝑒 𝑥 sin 𝑒 𝑥 𝑑𝑥 = − cos 𝑒 𝑥 + 𝐶.

tan 𝑥 − sec 𝑥
𝟏𝟐. ∫ 𝑑𝑥 = ∫(tan 𝑥 − sec 𝑥) ∙ sec 𝑥 𝑑𝑥
cos 𝑥

= ∫(tan 𝑥 sec 𝑥 − sec 2 𝑥)𝑑𝑥 = sec 𝑥 − tan 𝑥 + 𝐶.

𝑑𝑥 𝑑𝑥 𝑑𝑥
𝟏𝟑. ∫ = ∫ = ∫
cos 2𝑥 + sin2 𝑥 cos 2 𝑥 −sin2 𝑥 + sin2 𝑥 cos 2 𝑥

17
Chapter 1: INDEFINITE INTEGRAL
= ∫ sec 2 𝑥 𝑑𝑥 = tan 𝑥 + 𝐶.

Where, cos 2𝑥 = cos 2 𝑥 −sin2 𝑥.

𝟏𝟒. ∫ cosh3 𝑥 𝑑𝑥 = ∫ cosh2 𝑥 ∙ cosh 𝑥 𝑑𝑥 = ∫(1 + sinh2 𝑥) ∙ cosh 𝑥 𝑑𝑥

1
= ∫(cosh 𝑥 + cosh 𝑥 ∙ sinh2 𝑥) 𝑑𝑥 = sinh 𝑥 + sinh3 𝑥 + 𝐶.
3

1.4.1.1 Integrals Involving a Square of Trigonometric and Hyperbolic


Functions
We use the following identities:

1 1
sin2 𝑥 = (1 − cos 2𝑥) cos 2 𝑥 = (1 + cos 2𝑥)
2 2

tan2 𝑥 = sec 2 𝑥 − 1 cot 2 𝑥 = csc 2 𝑥 − 1

1 1
sinh2 𝑥 = (cosh 2𝑥 − 1) cosh2 𝑥 = (cosh 2𝑥 + 1)
2 2

tanh2 𝑥 = 1 − sech2 𝑥 coth2 𝑥 = 1 + csch2 𝑥

Example 8: Evaluate the following integrals

1 1 sin 2𝑥
𝟏. ∫ sin2 𝑥 𝑑𝑥 = ∫(1 − cos 2𝑥) 𝑑𝑥 = [𝑥 − ] + 𝐶.
2 2 2

18
Chapter 1: INDEFINITE INTEGRAL
1 1 sin 2𝑥
𝟐. ∫ cos 2 𝑥 𝑑𝑥 = ∫(1 + cos 2𝑥) 𝑑𝑥 = [𝑥 + ] + 𝐶.
2 2 2

𝟑. ∫ tan2 𝑥 𝑑𝑥 = ∫(sec 2 𝑥 − 1) 𝑑𝑥 = tan 𝑥 − 𝑥 + 𝐶.

𝟒. ∫ cot 2 𝑥 𝑑𝑥 = ∫(csc 2 𝑥 − 1) 𝑑𝑥 = − cot 𝑥 − 𝑥 + 𝐶.

1 1 sinh 2𝑥
𝟓. ∫ sinh2 𝑥 𝑑𝑥 = ∫(cosh 2𝑥 − 1) 𝑑𝑥 = [ − 𝑥] + 𝐶.
2 2 2

1 1 sinh 2𝑥
𝟔. ∫ cosh2 𝑥 𝑑𝑥 = ∫(cosh 2𝑥 + 1) 𝑑𝑥 = [ + 𝑥] + 𝐶.
2 2 2

𝟕. ∫ tanh2 𝑥 𝑑𝑥 = ∫(1 − sech2 𝑥) 𝑑𝑥 = 𝑥 − tanh 𝑥 + 𝐶.

𝟖. ∫ coth2 𝑥 𝑑𝑥 = ∫(1 + csch2 𝑥) 𝑑𝑥 = 𝑥 − coth 𝑥 + 𝐶.

𝟗. ∫ sec 2 𝑥 𝑑𝑥 = tan 𝑥 + 𝐶.

𝟏𝟎. ∫ sech2 𝑥 𝑑𝑥 = tanh 𝑥 + 𝐶.

𝟏𝟏. ∫ csc 2 𝑥 𝑑𝑥 = − cot 𝑥 + 𝐶.

𝟏𝟐. ∫ csch2 𝑥 𝑑𝑥 = − coth 𝑥 + 𝐶.

19
Chapter 1: INDEFINITE INTEGRAL
1.4.1.2 Integrals Involving a Products of Trigonometric and Hyperbolic
Functions
We use the following identities:

1
sin 𝑥 cos 𝑦 = [sin(𝑥 + 𝑦) + sin(𝑥 − 𝑦)]
2
1
cos 𝑥 cos 𝑦 = [cos(𝑥 − 𝑦) + cos(𝑥 + 𝑦)]
2
1
sin 𝑥 sin 𝑦 = [cos(𝑥 − 𝑦) − cos(𝑥 + 𝑦)]
2

1
sinh 𝑥 cosh 𝑦 = [sinh(𝑥 + 𝑦) + sinh(𝑥 − 𝑦)]
2
1
cosh 𝑥 cosh 𝑦 = [cosh(𝑥 + 𝑦) + cosh(𝑥 − 𝑦)]
2
1
sinh 𝑥 sinh 𝑦 = [cosh(𝑥 + 𝑦) − cosh(𝑥 − 𝑦)]
2

Example 9: Evaluate the following integrals

1 1 cos 2𝑥 cos 8𝑥
𝟏. ∫ sin 5𝑥 cos 3𝑥 𝑑𝑥 = ∫(sin 8𝑥 + sin 2𝑥) 𝑑𝑥 = [− − ] + 𝐶.
2 2 2 8

1 1 sin 3𝑥 sin 11𝑥


𝟐. ∫ sin 7𝑥 sin 4𝑥 𝑑𝑥 = ∫(cos 3𝑥 − cos 11𝑥) 𝑑𝑥 = [ − ] + 𝐶.
2 2 3 11

20
Chapter 1: INDEFINITE INTEGRAL
1 1 sin 2𝑥 sin 8𝑥
𝟑. ∫ cos 5𝑥 cos 3𝑥 𝑑𝑥 = ∫(cos 2𝑥 + cos 8𝑥) 𝑑𝑥 = [ + ] + 𝐶.
2 2 2 8

1
𝟒. ∫ cosh 2𝑥 cosh 3𝑥 𝑑𝑥 = ∫(cosh 5𝑥 + cosh 𝑥) 𝑑𝑥
2
1 sinh 5𝑥
= [ + sinh 𝑥] + 𝐶.
2 5

𝟓. ∫ cosh 4𝑥 sinh 2𝑥 𝑑𝑥 = ∫ sinh 2𝑥 cosh 4𝑥 𝑑𝑥

1
= ∫(sinh 6𝑥 + sinh(−2𝑥)) 𝑑𝑥
2
1 cosh 6𝑥 cosh 2𝑥
= [ − ] + 𝐶.
2 6 2

1.4.2 Integration of Inverse Trigonometric and Inverse Hyperbolic Integrals

𝑥
𝑑𝑥 sin−1 +𝐶
∫ ={ 𝑎
𝑥
√𝑎2 − 𝑥 2 − cos −1 + 𝐶
𝑎

1 −1
𝑥
𝑑𝑥 tan +𝐶
∫ 2 ={ 𝑎 𝑎
𝑎 + 𝑥2 1 𝑥
− cot −1 + 𝐶
𝑎 𝑎
1 −1
𝑥
𝑑𝑥 sec +𝐶
∫ ={ 𝑎 𝑎
𝑥√𝑥 2 − 𝑎2 1 𝑥
− csc −1 + 𝐶
𝑎 𝑎

21
Chapter 1: INDEFINITE INTEGRAL
𝑑𝑥 𝑥
∫ = sinh−1 +𝐶
√𝑎2 + 𝑥 2 𝑎

𝑑𝑥 𝑥
∫ = cosh−1 +𝐶
√𝑥 2 − 𝑎2 𝑎

𝑑𝑥 1 −1
𝑥
∫ = tanh +𝐶
𝑎2 − 𝑥 2 𝑎 𝑎

𝑑𝑥 1 𝑥
∫ = − coth−1 + 𝐶
𝑥2 −𝑎 2 𝑎 𝑎

𝑑𝑥 1 𝑥
∫ = − sech−1 + 𝐶
𝑥√𝑎2 − 𝑥 2 𝑎 𝑎

𝑑𝑥 1 𝑥
∫ = − csch−1 + 𝐶
𝑥√𝑎2 + 𝑥 2 𝑎 𝑎

Example 10: Evaluate the following integrals


𝑑𝑥 𝑑𝑥 𝑥
𝟏. ∫ =∫ = sin−1 ( ) + 𝐶.
√9 − 𝑥 2 √(3)2 − 𝑥 2 3

𝑑𝑥 𝑑𝑥 1 𝑑𝑥 1 𝑥
𝟐. ∫ =∫ = ∫ = sin−1 + 𝐶.
√2 − 3𝑥 2 √3 √3
√3 (2 − 𝑥 2 ) √2 − 𝑥 2 √2
3 3 ( 3)

2𝑥 𝑑𝑥 2𝑥 𝑑𝑥
𝟑. ∫ =∫ = sin−1 (𝑥 2 ) + 𝐶.
√1 − 𝑥 4 √1 − (𝑥 2 )2

22
Chapter 1: INDEFINITE INTEGRAL
𝑑𝑥 𝑑𝑥 𝑥
𝟒. ∫ =∫ = sinh−1 ( ) + 𝐶.
√𝑥 2 + 9 √𝑥 2 + (3)2 3

𝑑𝑥 1 −1
𝑥
𝟓. ∫ = tan ( ) + 𝐶.
𝑥2 + 4 2 2

cos 𝑥 cos 𝑥
𝟔. ∫ 2
𝑑𝑥 = ∫ 2
𝑑𝑥 = tan−1 (sin 𝑥) + 𝐶.
1 + sin 𝑥 1 + (sin 𝑥)

1
𝑑𝑥 𝑥 ln 𝑥
𝟕. ∫ 𝑑𝑥 = ∫ 𝑑𝑥 = sin−1 ( ) + 𝐶.
𝑥√3 − ln2 𝑥 √3 − (ln 𝑥)2 √3

𝑑𝑥 𝑑𝑥 𝑒𝑥 𝑒𝑥
𝟖. ∫ 𝑥 −𝑥
=∫ 𝑥 −𝑥
∙ 𝑥=∫ 𝑥 2 𝑑𝑥 = tan−1 (𝑒 𝑥 ) + 𝐶.
𝑒 +𝑒 𝑒 +𝑒 𝑒 (𝑒 ) + 1

𝑒 𝑥 (1 + 𝑒 𝑥 ) 𝑒 𝑥 + 𝑒 2𝑥 𝑒𝑥 𝑒 2𝑥
𝟗. ∫ 𝑑𝑥 = ∫ 𝑑𝑥 = ∫ 𝑑𝑥 + ∫ 𝑑𝑥
√1 − 𝑒 2𝑥 √1 − 𝑒 2𝑥 √1 − 𝑒 2𝑥 √1 − 𝑒 2𝑥

𝑒𝑥 −1 −2𝑒 2𝑥
=∫ 𝑑𝑥 + ∫ 𝑑𝑥
√1 − (𝑒 𝑥 )2 2 √1 − 𝑒 2𝑥

= sin−1 (𝑒 𝑥 ) − √1 − 𝑒 2𝑥 + 𝐶.

23
Chapter 1: INDEFINITE INTEGRAL
Exercises (1)

Evaluate the following integrals


11
8
∫ 12𝑥 𝑑𝑥 ∫ 15𝑥 3 𝑑𝑥

𝑑𝑥 𝑑𝑥
∫ ∫
𝑥2 𝑥4
𝑑𝑥 𝑑𝑥
∫3 ∫ 3
√𝑥 5𝑥 2
1 1
∫ (𝑥 2 + 𝑥 −2 ) 𝑑𝑥
7
∫ √𝑥 + 1 𝑑𝑥

𝑥 4 + 3𝑥 2 + 1 𝑥+3
∫ 𝑑𝑥 ∫ 𝑑𝑥
𝑥3 𝑥
2 2 3 4
∫ (𝑥 3 + 1 + 𝑥 −3 ) 𝑑𝑥 ∫( − ) 𝑑𝑥
𝑥−1 𝑥−2
𝑥 𝑥2
∫ 𝑑𝑥 ∫ 3 𝑑𝑥
𝑥2 + 4 𝑥 +5
sin 𝑎𝑥 1 + cos 2𝑥
∫ 𝑑𝑥 ∫ 𝑑𝑥
1 + cos 𝑎𝑥 2𝑥 + sin 2𝑥
𝑒 3𝑥
∫ 3𝑥 𝑑𝑥 ∫(𝑒 𝑥 + 𝑒 −𝑥 )2 𝑑𝑥
𝑒 +5
2
∫ 𝑥 2 𝑒 5𝑥 𝑑𝑥 ∫ sec 2 𝑥 𝑒 tan 𝑥 𝑑𝑥

𝑒 sin 𝑥
∫ 𝑑𝑥 ∫ cos 3 𝑥 sin 𝑥 𝑑𝑥
sec 𝑥
5
∫ sin5 𝑥 cos 𝑥 𝑑𝑥 ∫ sec 2 𝑥 tan3 𝑥 𝑑𝑥

sin 𝑥 tan 𝑥 − sec 𝑥


∫ 𝑑𝑥 ∫ 𝑑𝑥
cos 3 𝑥 cos 𝑥

24
Chapter 1: INDEFINITE INTEGRAL
5
∫ sin 𝑥 √cos 𝑥 𝑑𝑥 ∫ sec 2 𝑥 sin 𝑥 𝑑𝑥

cos 𝑥
∫ csc 2 𝑥 cos 𝑥 𝑑𝑥 ∫ 𝑑𝑥
sin3 𝑥

∫ sec 𝑥 tan 𝑥 𝑑𝑥

(ln 𝑥)𝑛 √ln 𝑥


∫ 𝑑𝑥 ∫ 𝑑𝑥
𝑥 𝑥
5
√ln 𝑥 sin−1 𝑥
∫ 𝑑𝑥 ∫ 𝑑𝑥
𝑥 √1 − 𝑥 2
tan−1 𝑥 𝑥 + cos −1 (3𝑥)
∫ 𝑑𝑥 ∫ 𝑑𝑥
1 + 𝑥2 √1 − 9𝑥 2
𝑒 2𝑥 − 1
∫ 𝑒 𝑥 sin(𝑒 𝑥 ) 𝑑𝑥 ∫ 𝑑𝑥
𝑒𝑥
cos 𝑥
∫ 𝑑𝑥 ∫ 𝑥 3 cos 𝑥 4 𝑑𝑥
√1 + sin 𝑥
𝑥+1 𝑥+1
∫ 2 𝑑𝑥 ∫ 𝑑𝑥
𝑥 + 2𝑥 + 1 √𝑥 2 + 2𝑥
𝑥2 cot 𝑥 − csc 𝑥
∫ ∫ 𝑑𝑥
2 𝑑𝑥 sin 𝑥
(8𝑥 3 + 27)3
𝑑𝑥
∫ cos 3 𝑥 sin 2𝑥 𝑑𝑥 ∫
(sin−1 𝑥)3 √1 − 𝑥 2
𝑑𝑥
∫ ∫ 𝑒 𝑥 sin(𝑒 𝑥 ) 𝑑𝑥
cos 2 𝑥 √1 + tan 𝑥
(1 − 𝑥)2 √𝑥 − 𝑥 3 𝑒 𝑥 + 𝑥 2
∫ 𝑑𝑥 ∫ 𝑑𝑥
𝑥 √𝑥 𝑥3
cos 2𝑥 cos 𝑥
∫ 𝑑𝑥 ∫3 𝑑𝑥
cos 2 𝑥 sin2 𝑥 √ sin2 𝑥

25
Chapter 1: INDEFINITE INTEGRAL
cos 2𝑥 cos 5 𝑥
∫ 𝑑𝑥 ∫ 𝑑𝑥
1 + sin 𝑥 cos 𝑥 sin6 𝑥

∫ 𝑒 √cosh 𝑥+1 √cosh 𝑥 + 1 𝑑𝑥 ∫ √1 − 𝑒 𝑥 𝑒 𝑥 𝑑𝑥

𝑑𝑥 3
∫ ∫ sin √𝑥 𝑑𝑥
𝑒 𝑥 √1 − 𝑒 −2𝑥
ln(cos 𝑥) cot 𝑥
∫ 𝑑𝑥 ∫ 𝑑𝑥
cos 2 𝑥 ln(sin 𝑥)
ln(tan 𝑥) 𝑥 𝑑𝑥
∫ 𝑑𝑥 ∫
sin 𝑥 cos 𝑥 √2𝑥 2 + 1
3𝑥 − 2 sin 𝑥
∫ 𝑑𝑥 ∫ 𝑑𝑥
√3𝑥 2 − 4𝑥 + 2 √cos 𝑥
𝑑𝑥

cos 2 𝑥 √tan 𝑥

26
Chapter 1: INDEFINITE INTEGRAL
Notes:

27
Chapter 1: INDEFINITE INTEGRAL

28
Chapter 1: INDEFINITE INTEGRAL

29
Chapter 1: INDEFINITE INTEGRAL

30
Chapter 2: TECHNIQUES OF INTEGRATION

Chapter 2. TECHNIQUES OF
INTEGRATION

Overview In previous chapter, we discussed integrals of those functions which


were readily obtainable from derivatives of some functions. It was based on
inspection, i.e., on the search of a function 𝐹 whose derivative is 𝑓 which led us to
the integral of 𝑓. However, this method, which depends on inspection, is not very
suitable for many functions. Hence, we need to develop additional techniques or
methods for finding the integrals by reducing them into standard forms.

2.1 The Substitution Rule

Because of the Fundamental Theorem, it’s important to be able to find


antiderivatives. But our antidifferentiation formulas don’t tell us how to evaluate
integrals such as

∫ 2𝑥√1 + 𝑥 2 𝑑𝑥 (1)

To find this integral we use the problem-solving strategy of introducing something


extra. Here the “something extra” is a new variable; we change from the variable 𝑥
to a new variable 𝑢. Suppose that we let be the quantity under the root sign in (1),
𝑢 = 1 + 𝑥 2 . Then the differential of 𝑢 is 𝑑𝑢 = 2𝑥 𝑑𝑥. Notice that if the 𝑑𝑥 in the
notation for an integral were to be interpreted as a differential, then the differential
2𝑥 𝑑𝑥 would occur in (1)and so, formally, without justifying our calculation, we
could write

31
Chapter 2: TECHNIQUES OF INTEGRATION
∫ 2𝑥√1 + 𝑥 2 𝑑𝑥 = ∫ √1 + 𝑥 2 2𝑥𝑑𝑥 = ∫ √𝑢 𝑑𝑢

2 3 2 3
= 𝑢2 + 𝐶 = (1 + 𝑥 2 )2 + 𝐶 (2)
3 3

But now we can check that we have the correct answer by using the Chain Rule to
differentiate the final function of Equation (2):

𝑑 2 3 2 3 1
[ (1 + 𝑥 2 )2 + 𝐶] = ∙ (1 + 𝑥 2 )2 ∙ 2𝑥 = 2𝑥√1 + 𝑥 2 .
𝑑𝑥 3 3 2

In general, this method works whenever we have an integral that we can write in the
form ∫ 𝑓(g(𝑥)) g′(𝑥)𝑑𝑥. Observe that if 𝐹 ′ = 𝑓, then

∫ 𝐹 ′ (g(𝑥)) g ′ (𝑥)𝑑𝑥 = 𝐹(g(𝑥)) + 𝐶

because, by the Chain Rule,

𝑑
[𝐹(g(𝑥))] = 𝐹 ′ (g(𝑥))g ′ (𝑥).
𝑑𝑥

If we make the “change of variable” or “substitution” 𝑢 = g(𝑥), then we have

∫ 𝐹 ′ (g(𝑥)) g ′ (𝑥)𝑑𝑥 = 𝐹(g(𝑥)) + 𝐶 = 𝐹(𝑢) + 𝐶 = ∫ 𝐹 ′ (𝑢) 𝑑𝑢

Or writing 𝐹 ′ = 𝑓, we get

∫ 𝑓(g(𝑥)) g ′ (𝑥)𝑑𝑥 = ∫ 𝑓(𝑢) 𝑑𝑢.

Thus, we have proved the following rule

32
Chapter 2: TECHNIQUES OF INTEGRATION

THE SUBSTITUTION RULE If 𝑢 = g(𝑥) is a differentiable function


whose range is an interval 𝐼 and 𝑓 is continuous on 𝐼, then

∫ 𝑓(g(𝑥)) g ′ (𝑥)𝑑𝑥 = ∫ 𝑓(𝑢)𝑑𝑢

Example 1: Find

∫ 𝑥 2 cos 𝑥 3 𝑑𝑥.

Solution: Let 𝑢 = 𝑥 3 ⇒ 𝑑𝑢 = 3𝑥 2 𝑑𝑥

1
Thus, using 𝑥 2 𝑑𝑥 = 𝑑𝑢 and the Substitution Rule, we have
3

1 1
∫ 𝑥 2 cos 𝑥 3 𝑑𝑥 = ∫ cos 𝑢 𝑑𝑢 = sin 𝑢 + 𝐶
3 3

1
= sin 𝑥 3 + 𝐶.
3

Notice that at the final stage we had to return to the original variable 𝑥.

Example 2: Find

∫ 𝑥 3 cos(𝑥 4 + 2) 𝑑𝑥.

Solution: Let 𝑢 = 𝑥 4 + 2 ⇒ 𝑑𝑢 = 4𝑥 3 𝑑𝑥

1
Thus, using 𝑥 3 𝑑𝑥 = 𝑑𝑢 and the Substitution Rule, we have
4

33
Chapter 2: TECHNIQUES OF INTEGRATION
1 1
∫ 𝑥 3 cos(𝑥 4 + 2) 𝑑𝑥 = ∫ cos 𝑢 𝑑𝑢 = sin 𝑢 + 𝐶
4 4

1
= sin(𝑥 4 + 2) + 𝐶.
4

Example 3: Find

∫ √2𝑥 + 1 𝑑𝑥.

Solution: Let 𝑢 = √2𝑥 + 1. Then,

2
𝑑𝑢 = 𝑑𝑥 so 𝑑𝑥 = √2𝑥 + 1 𝑑𝑢 = 𝑢 𝑑𝑢
2√2𝑥 + 1

Thus, using the Substitution Rule, we have

1
∫ √2𝑥 + 1 𝑑𝑥 = ∫ 𝑢 ∙ 𝑢 𝑑𝑢 = ∫ 𝑢2 𝑑𝑢 = 𝑢3 + 𝐶
3

1 3
= (2𝑥 + 1)2 + 𝐶.
3

Example 4: Find

∫ 𝑥 3 √1 + 𝑥 4 𝑑𝑥.

Solution: Let 𝑢 = √1 + 𝑥 4 . Then,

4𝑥 3 2𝑥 3
𝑑𝑢 = 𝑑𝑥 so 𝑑𝑢 = 𝑑𝑥
2√1 + 𝑥 4 𝑢

34
Chapter 2: TECHNIQUES OF INTEGRATION
∴ 𝑢 𝑑𝑢 = 2𝑥 3 𝑑𝑥
1
Thus, using 𝑥 3 𝑑𝑥 = 𝑢 𝑑𝑢 and the Substitution Rule, we have
2

1 1
∫ 𝑥 3 √1 + 𝑥 4 𝑑𝑥 = ∫ 𝑢 ∙ 𝑢 𝑑𝑢 = 𝑢3 + 𝐶
2 6

1 3
4 )2
= (1 +𝑥 + 𝐶.
6

Example 5: Find

𝑥
∫ 𝑑𝑥.
√1 − 4𝑥 2

Solution:

Let 𝑢 = 1 − 4𝑥 2 . Then, 𝑑𝑢 = −8𝑥 𝑑𝑥, So

1
𝑥 𝑑𝑥 = − 𝑑𝑢
8

Thus, using the Substitution Rule, we have

𝑥 1 1 1 1 1
∫ 𝑑𝑥 = − ∫ 𝑑𝑢 = − ∫ 𝑢−2 𝑑𝑢 = − (2√𝑢) + 𝐶
√1 − 4𝑥 2 8 √𝑢 8 8

1
= − √1 − 4𝑥 2 + 𝐶.
4

35
Chapter 2: TECHNIQUES OF INTEGRATION
Example 6: Find

𝑥3
∫ 𝑑𝑥.
√𝑥 2 − 4

Solution:

Let 𝑢 = √𝑥 2 − 4. Then, 𝑢2 = 𝑥 2 − 4, So

2𝑢𝑑𝑢 = 2𝑥𝑑𝑥 ⇒ 𝑥 𝑑𝑥 = 𝑢 𝑑𝑢

𝑥3 𝑢2 + 4 1
∫ 𝑑𝑥 = ∫ ∙ 𝑢𝑑𝑢 = ∫(𝑢2 + 4) 𝑑𝑢 = 𝑢3 + 4𝑢 + 𝐶
√𝑥 2 − 4 𝑢 3

1 3
= (𝑥 2 − 4)2 + 4(𝑥 2 − 4) + 𝐶.
3

Example 7: Find

∫ 𝑥√𝑥 + 1 𝑑𝑥.

Solution:

Let 𝑢 = √𝑥 + 1. Then, 𝑢2 = 𝑥 + 1, So

2𝑢𝑑𝑢 = 𝑑𝑥 ⇒ 𝑑𝑥 = 2𝑢 𝑑𝑢

∫ 𝑥√𝑥 + 1 𝑑𝑥 = ∫(𝑢2 − 1) ∙ 𝑢 ∙ 2𝑢 𝑑𝑢 = 2 ∫(𝑢2 − 1)𝑢2 𝑑𝑢 = 2 ∫(𝑢4 − 𝑢2 ) 𝑑𝑢

𝑢5 𝑢3 1 5 1 3
= 2 [ − ] + 𝐶 = 2 [ (𝑥 + 1)2 − (𝑥 + 1)2 ] + 𝐶.
5 3 5 3

36
Chapter 2: TECHNIQUES OF INTEGRATION
Example 8: Find

∫ √1 + 𝑥 2 𝑥 5 𝑑𝑥.

Solution: An appropriate substitution becomes more obvious if we factor 𝑥 5 as


𝑥 4 ∙ 𝑥,

Let 𝑢 = 1 + 𝑥 2 . Then, 𝑑𝑢 = 2𝑥 𝑑𝑥, So

1
𝑥 𝑑𝑥 = 𝑑𝑢
2

Also, 𝑥 2 = 𝑢 − 1, so 𝑥 4 = (𝑢 − 1)2

1 1
∫ √1 + 𝑥 2 𝑥 5 𝑑𝑥 = ∫ √𝑢 ∙ (𝑢 − 1)2 𝑑𝑢 = ∫ √𝑢 ∙ (𝑢2 − 2𝑢 + 1) 𝑑𝑢
2 2

1 5 3 1 1 2 7 2 5 2 3
= ∫ [𝑢 − 2𝑢 + 𝑢 ] 𝑑𝑢 = [ 𝑢 − 2 𝑢2 + 𝑢2 ] + 𝐶
2 2 2 2
2 2 7 5 3

1 7 2 5 1 3
= (1 + 𝑥 2 )2 − (1 + 𝑥 2 )2 + (1 + 𝑥 2 )2 + 𝐶.
7 5 3

Example 9: Find

1
∫ 𝑑𝑥.
𝑒 𝑥 + 𝑒 −𝑥

Solution:

Let 𝑢 = 𝑒 𝑥 . Then, 𝑑𝑢 = 𝑒 𝑥 𝑑𝑥, So

1 𝑑𝑢
𝑒 −𝑥 = ⇒ 𝑑𝑥 =
𝑢 𝑢

37
Chapter 2: TECHNIQUES OF INTEGRATION
𝑑𝑢
1 𝑢 = ∫ 𝑑𝑢 = tan−1 𝑢 + 𝐶
∫ 𝑥 𝑑𝑥 = ∫
𝑒 + 𝑒 −𝑥 1 𝑢2 + 1
𝑢+
𝑢

= tan−1 (𝑒 𝑥 ) + 𝐶.

Example 10: Find

cos 𝑥
∫ 𝑑𝑥.
1 + sin2 𝑥

Solution:

Let 𝑢 = sin 𝑥. Then, 𝑑𝑢 = cos 𝑥 𝑑𝑥, So

cos 𝑥 1
∫ 2
𝑑𝑥 = ∫ 2
𝑑𝑢 = tan−1 𝑢 + 𝐶
1 + sin 𝑥 1+𝑢

= tan−1 (sin 𝑥) + 𝐶.

Example 11: Find

∫ tan 𝑥 𝑑𝑥.

Solution: First, we write tangent in terms of sine and cosine:

sin 𝑥
∫ tan 𝑥 𝑑𝑥 = ∫ 𝑑𝑥.
cos 𝑥

Let 𝑢 = cos 𝑥. Then, 𝑑𝑢 = − sin 𝑥 𝑑𝑥, So

38
Chapter 2: TECHNIQUES OF INTEGRATION
sin 𝑥 1
∫ tan 𝑥 𝑑𝑥 = ∫ 𝑑𝑥 = − ∫ 𝑑𝑢 = − ln|𝑢| + 𝐶
cos 𝑥 𝑢

= − ln|cos 𝑥| + 𝐶.

Since − ln|cos 𝑥| = ln(|cos 𝑥|)−1 = ln(1⁄|cos 𝑥|) = ln|sec 𝑥|. Then,

∫ tan 𝑥 𝑑𝑥 = ln|sec 𝑥| + 𝐶.

Example 12: Find

∫ sec 𝑥 𝑑𝑥.

Solution: First, we write sec function in terms of cosine:

1 cos 𝑥 cos 𝑥
∫ sec 𝑥 𝑑𝑥 = ∫ 𝑑𝑥 = ∫ 𝑑𝑥 = ∫ 𝑑𝑥
cos 𝑥 cos 2 𝑥 1 − sin2 𝑥

Let 𝑢 = sin 𝑥. Then, 𝑑𝑢 = cos 𝑥 𝑑𝑥, So

cos 𝑥 1
∫ sec 𝑥 𝑑𝑥 = ∫ 2
𝑑𝑥 = ∫ 2
𝑑𝑢 = tanh−1 𝑢 + 𝐶
1 − sin 𝑥 1−𝑢

= tanh−1 (sin 𝑥) + 𝐶.

1 1 + sin 𝑥
= ln ( ) + 𝐶.
2 1 − sin 𝑥

39
Chapter 2: TECHNIQUES OF INTEGRATION
2.2 Trigonometric Substitution

In finding the area of a circle or an ellipse, an integral of the form ∫ √𝑎2 − 𝑥 2 𝑑𝑥


arises, where 𝑎 > 0. If it were ∫ 𝑥√𝑎2 − 𝑥 2 𝑑𝑥, the substitution 𝑢 = 𝑎2 − 𝑥 2 would
be effective but, as it stands, ∫ √𝑎2 − 𝑥 2 𝑑𝑥 is more difficult. If we change the
variable from 𝑥 to 𝜃 by the substitution 𝑥 = 𝑎 sin 𝜃, then the identity 1 − sin2 𝜃 =
cos 2 𝜃 allows us to get rid of the root sign because

∫ √𝑎2 − 𝑥 2 𝑑𝑥 = ∫ √𝑎2 − 𝑎2 sin2 𝜃 𝑑𝑥 = ∫ √𝑎2 (1 − sin2 𝜃)𝑑𝑥

= ∫ √𝑎2 cos 2 𝜃 𝑑𝑥 = 𝑎|cos 𝜃|

Notice the difference between the substitution 𝑢 = 𝑎2 − 𝑥 2 (in which the new
variable is a function of the old one) and the substitution 𝑥 = 𝑎 sin 𝜃 (the old variable
is a function of the new one).

In general, we can make a substitution of the form 𝑥 = g(𝑡) by using the Substitution
Rule in reverse. To make our calculations simpler, we assume that g has an inverse
function; that is, g is one-to-one. In this case, if we replace 𝑢 by 𝑥 and 𝑥 by 𝑡 in the
Substitution Rule, we obtain

∫ 𝑓(𝑥)𝑑𝑥 = ∫ 𝑓(g(𝑡))g′(𝑡) 𝑑𝑡

This kind of substitution is called inverse substitution.

We can make the inverse substitution 𝑥 = 𝑎 sin 𝜃 provided that it defines a one-to-
one function. This can be accomplished by restricting 𝜃 to lie in the interval
[− 𝜋⁄2 , 𝜋⁄2].

40
Chapter 2: TECHNIQUES OF INTEGRATION
In the following table we list trigonometric substitutions that are effective for the
given radical expressions because of the specified trigonometric identities. In each
case the restriction on 𝜃 is imposed to ensure that the function that defines the
substitution is one-to-one.

(These are the same intervals used in defining the inverse functions.)

Expression Substitution identity


𝜋 𝜋 1 − sin2 𝜃 = cos 2 𝜃
√𝑎 2 − 𝑥 2 𝑥 = 𝑎 sin 𝜃 , − ≤𝜃≤
2 2
𝑑𝑥 = 𝑎 cos 𝜃 𝑑𝜃
√𝑎2 − 𝑥 2 = 𝑎 cos 𝜃
𝑥
𝜃 = sin−1 ( )
𝑎
𝜋 𝜋 sec 2 𝜃 − 1 = tan2 𝜃
√𝑎 2 + 𝑥 2 𝑥 = 𝑎 tan 𝜃 , − ≤ 𝜃 ≤
2 2
𝑑𝑥 = 𝑎 sec 2 𝜃 𝑑𝜃
√𝑎2 + 𝑥 2 = 𝑎 sec 𝜃
𝑥
𝜃 = tan−1 ( )
𝑎
√𝑥 2 − 𝑎 2 𝑥 = 𝑎 sec 𝜃, 1 + tan2 𝜃 = sec 2 𝜃
𝜋 𝜋
0≤𝜃≤ or − ≤ 𝜃 ≤ 𝜋
2 2
𝑑𝑥 = 𝑎 sec 𝜃 tan 𝜃 𝑑𝜃
√𝑥 2 − 𝑎2 = 𝑎 tan 𝜃
𝑥
𝜃 = sec −1 ( )
𝑎

41
Chapter 2: TECHNIQUES OF INTEGRATION
Example 13: Find

∫ √16 − 𝑥 2 𝑑𝑥.

Solution: Let 𝑥 = 4 sin 𝜃. Then, 𝑑𝑥 = 4 cos 𝜃 𝑑𝜃 and

∫ √16 − 𝑥 2 𝑑𝑥 = ∫ √16 − 16 sin2 𝜃 4 cos 𝜃 𝑑𝜃 = ∫ 4 cos 𝜃 4 cos 𝜃 𝑑𝜃

= 16 ∫ cos 2 𝜃 𝑑𝜃

4
= 8 ∫(1 + cos 2𝜃) 𝑑𝜃

sin 2𝜃
= 8 [𝜃 + ]+𝐶 √16 − 𝑥 2
2

= 8[𝜃 + sin 𝜃 cos 𝜃] + 𝐶

𝑥 1
= 8 [sin−1 ( ) + 𝑥 √16 − 𝑥 2 ] + 𝐶.
4 2

Example 14: Find

1
∫ 𝑑𝑥.
𝑥 2 √9 − 𝑥 2

Solution: Let 𝑥 = 3 sin 𝜃. Then, 𝑑𝑥 = 3 cos 𝜃 𝑑𝜃 and

1 1 3
∫ 𝑑𝑥 = ∫ 3 cos 𝜃 𝑑𝜃
𝑥 2 √9 − 𝑥 2 9 sin2 𝜃 √9 − 9 sin2 𝜃
1
=∫ 3 cos 𝜃 𝑑𝜃
9 sin2 𝜃 ∙ 3 cos 𝜃 √9 − 𝑥 2

42
Chapter 2: TECHNIQUES OF INTEGRATION
1 1
= ∫ csc 2 𝜃 𝑑𝜃 = − cot 𝜃 + 𝐶
9 9

1 √9 − 𝑥 2
=− [ ] + 𝐶.
9 𝑥

Example 15: Find

√2 − 𝑥 2
∫ 𝑑𝑥.
𝑥2

Solution: Let 𝑥 = √2 sin 𝜃. Then, 𝑑𝑥 = √2 cos 𝜃 𝑑𝜃 and

√2 − 𝑥 2 √2 − 2 sin2 𝑥
∫ 𝑑𝑥 = ∫ √2 cos 𝜃 𝑑𝜃
𝑥2 2 sin2 𝑥

√2 cos 𝜃 √2
=∫ √2 cos 𝜃 𝑑𝜃
2 sin2 𝑥

cos 2 𝑥
=∫ 2
𝑑𝜃 = ∫ cot 2 𝑥 𝑑𝜃 √2 − 𝑥 2
sin 𝑥

= ∫(csc 2 𝑥 − 1) 𝑑𝜃 = − cot 𝜃 − 𝜃 + 𝐶

√2 − 𝑥 2 𝑥
= − sin−1 ( ) + 𝐶.
𝑥 √2

Example 16: Find

√𝑥 2 − 9
∫ 𝑑𝑥.
𝑥

43
Chapter 2: TECHNIQUES OF INTEGRATION
Solution: Let 𝑥 = 3 sec 𝜃. Then, 𝑑𝑥 = 3 sec 𝜃 tan 𝜃 𝑑𝜃 and

√𝑥 2 − 9 √9 sec 2 𝜃 − 9
∫ 𝑑𝑥 = ∫ 3 sec 𝜃 tan 𝜃 𝑑𝜃
𝑥 3 sec 𝜃

= ∫ 3 tan 𝜃 ∙ tan 𝜃 𝑑𝜃

= 3 ∫ tan2 𝜃 𝑑𝜃 𝑥
√𝑥 2 − 9

= 3 ∫(sec 2 𝜃 − 1) 𝑑𝜃
3

= 3[tan 𝜃 − 𝜃] + 𝐶

√𝑥 2 − 9 𝑥
= 3[ − sec −1 ( )] + 𝐶.
𝑥 3

Example 17: Find

𝑥3
∫ 𝑑𝑥.
√𝑥 2 − 1

Solution: Let 𝑥 = sec 𝜃. Then, 𝑑𝑥 = sec 𝜃 tan 𝜃 𝑑𝜃 and

𝑥3 sec 3 𝜃
∫ 𝑑𝑥 = ∫ sec 𝜃 tan 𝜃 𝑑𝜃
√𝑥 2 − 1 √sec 2 𝜃 − 1

= ∫ sec 4 𝜃 𝑑𝜃 𝑥
√𝑥 2 − 1

= ∫ sec 2 𝜃 ∙ sec 2 𝜃 𝑑𝜃
1

44
Chapter 2: TECHNIQUES OF INTEGRATION
= ∫(1 + tan2 𝜃) ∙ sec 2 𝜃 𝑑𝜃

1
= [tan 𝜃 + tan3 𝜃] + 𝐶
3

1 3
= [√𝑥 2 − 1 + (𝑥 2 − 1)2 ] + 𝐶.
3

Example 18: Find

1
∫ 𝑑𝑥.
𝑥√𝑥 2 + 4

Solution: Let 𝑥 = 2 tan 𝜃. Then, 𝑑𝑥 = 2 sec 2 𝜃 𝑑𝜃 and

1 2 sec 2 𝜃
∫ 𝑑𝑥 = ∫ 𝑑𝜃
𝑥√𝑥 2 + 4 2 tan 𝜃 √4 tan2 𝜃 + 4

2 sec 2 𝜃
=∫ 𝑑𝜃 √𝑥 2 + 4
2 tan 𝜃 ∙ 2 sec 𝜃 𝑥

1 sec 𝜃
= ∫ 𝑑𝜃
2 tan 𝜃 2

1
= ∫ csc 𝜃 𝑑𝜃
2

1
= ln|csc 𝜃 − cot 𝜃| + 𝐶
2

1 √𝑥 2 + 4 2
= ln | − | + 𝐶.
2 𝑥 𝑥

45
Chapter 2: TECHNIQUES OF INTEGRATION
Example 19: Find

𝑥3
∫ 𝑑𝑥.
√1 + 𝑥 2

Solution: Let 𝑥 = tan 𝜃. Then, 𝑑𝑥 = sec 2 𝜃 𝑑𝜃 and

𝑥3 tan3 𝜃
∫ 𝑑𝑥 = ∫ sec 2 𝜃 𝑑𝜃
√1 + 𝑥2 √1 + tan2 𝜃

tan3 𝜃
=∫ sec 2 𝜃 𝑑𝜃 √1 + 𝑥 2
sec 𝜃 𝑥

= ∫ tan3 𝜃 ∙ sec 𝜃 𝑑𝜃
1

= ∫ tan2 𝜃 ∙ sec 𝜃 tan 𝜃 𝑑𝜃

= ∫(sec 2 𝜃 − 1) ∙ sec 𝜃 tan 𝜃 𝑑𝜃

1
= sec 3 𝜃 − sec 𝜃 + 𝐶
3
1 3
= (1 + 𝑥 2 )2 − √1 + 𝑥 2 + 𝐶.
3

Example 20: Find

√𝑥 2 + 4
∫ 𝑑𝑥.
𝑥

Solution: Let 𝑥 = 2 tan 𝜃. Then, 𝑑𝑥 = 2 sec 2 𝜃 𝑑𝜃 and

46
Chapter 2: TECHNIQUES OF INTEGRATION
√𝑥 2 + 4 √4tan2 𝜃 + 4
∫ 𝑑𝑥 = ∫ 2 sec 2 𝜃 𝑑𝜃
𝑥 2 tan 𝜃
√𝑥 2 + 4
2 sec 𝜃 𝑥
=∫ 2 sec 2 𝜃 𝑑𝜃
2 tan 𝜃

sec 𝜃 2
= 2∫ sec 2 𝜃 𝑑𝜃
tan 𝜃

1
= 2∫ (1 + tan2 𝜃) 𝑑𝜃
sin 𝜃

= 2 ∫ csc 𝜃 𝑑𝜃 + 2 ∫ sec 𝜃 tan 𝜃 𝑑𝜃

= 2 ln|csc 𝜃 − cot 𝜃| + 2 sec 𝜃 + 𝐶

√𝑥 2 + 4 2
= 2 ln | − | + √𝑥 2 + 4 + 𝐶.
𝑥 𝑥

2.3 Trigonometric Integrals

2.3.1 Integrating a Product of Powers of Sine and Cosine

In this section we use trigonometric identities to integrate certain combinations of


trigonometric functions. We start with powers of sine and cosine.

Strategy for Evaluating ∫ sin𝑚 𝑥 cos 𝑛 𝑥 𝑑𝑥

(a) If the power of cosine is odd 𝑛 = (2𝑘 + 1), save one cosine factor and use
cos 2 𝑥 = 1 − sin2 𝑥 to express the remaining factors in terms of sine:

47
Chapter 2: TECHNIQUES OF INTEGRATION
∫ sin𝑚 𝑥 cos 2𝑘+1 𝑥 𝑑𝑥 = ∫ sin𝑚 𝑥 (cos2 𝑥)𝑘 cos 𝑥 𝑑𝑥

= ∫ sin𝑚 𝑥 (1 − sin2 𝑥)𝑘 cos 𝑥 𝑑𝑥

Such that

1
∫ sin𝑘 𝑥 cos 𝑥 𝑑𝑥 = sin𝑘+1 𝑥 + 𝐶.
𝑘+1

(b) If the power of sine is odd 𝑚 = (2𝑘 + 1), save one cosine factor and use
sin2 𝑥 = 1 − cos 2 𝑥 to express the remaining factors in terms of cosine:

∫ sin2𝑘+1 𝑥 cos 𝑛 𝑥 𝑑𝑥 = ∫ (sin2 𝑥)𝑘 cos 𝑛 𝑥 sin 𝑥 𝑑𝑥

= ∫ (1 − cos 2 𝑥)𝑘 cos 𝑛 𝑥 sin 𝑥 𝑑𝑥

Such that

1
∫ cos 𝑘 𝑥 sin 𝑥 𝑑𝑥 = − cos 𝑘+1 𝑥 + 𝐶.
𝑘+1

[Note that if the powers of both sine and cosine are odd, either (a) or (b) can
be used].

(c) If the powers of both sine and cosine are even, use the half-angle identities

1 1
sin2 𝑥 = (1 − cos 2𝑥), cos 2 𝑥 = (1 + cos 2𝑥).
2 2

It is sometimes helpful to use the identity

1
sin 𝑥 cos 𝑥 = sin 2𝑥.
2

48
Chapter 2: TECHNIQUES OF INTEGRATION
Example 21: Evaluate ∫ cos 3 𝑥 𝑑𝑥

Solution: In order to integrate powers of cosine, we would need an extra sin 𝑥


factor. Similarly, a power of sine would require an extra cos 𝑥 factor. Thus, here
we can separate one cosine factor and convert the remaining cos 2 𝑥 factor to an
expression involving sine using the identity cos 2 𝑥 = 1 − sin2 𝑥,

∫ cos 3 𝑥 𝑑𝑥 = ∫ cos 2 𝑥 cos 𝑥 𝑑𝑥 = ∫(1 − sin2 𝑥) cos 𝑥 𝑑𝑥

= ∫ cos 𝑥 𝑑𝑥 − ∫ sin2 𝑥 cos 𝑥 𝑑𝑥

1
= sin 𝑥 − sin3 𝑥 + 𝐶.
3

Example 22: Evaluate ∫ sin3 𝑥 𝑑𝑥

Solution:

∫ sin3 𝑥 𝑑𝑥 = ∫ sin2 𝑥 sin 𝑥 𝑑𝑥 = ∫(1 − cos 2 𝑥) sin 𝑥 𝑑𝑥

= ∫ sin 𝑥 𝑑𝑥 − ∫ cos 2 𝑥 sin 𝑥 𝑑𝑥

1
= − cos 𝑥 + cos 3 𝑥 + 𝐶.
3

Example 23: Evaluate ∫ sin3 𝑥 cos 2 𝑥 𝑑𝑥

Solution:

∫ sin3 𝑥 cos 2 𝑥 𝑑𝑥 = ∫ sin2 𝑥 ∙ cos 2 𝑥 ∙ sin 𝑥 𝑑𝑥

49
Chapter 2: TECHNIQUES OF INTEGRATION

= ∫(1 − cos 2 𝑥) ∙ cos 2 𝑥 sin 𝑥 𝑑𝑥

= ∫(cos 2 𝑥 − cos 4 𝑥) sin 𝑥 𝑑𝑥

1 1
= − cos 3 𝑥 + cos 5 𝑥 + 𝐶.
3 5

Example 24: Evaluate ∫ cos 5 𝑥 𝑑𝑥

Solution:

∫ cos 5 𝑥 𝑑𝑥 = ∫ cos 4 𝑥 cos 𝑥 𝑑𝑥 = ∫(1 − sin2 𝑥)2 cos 𝑥 𝑑𝑥

= ∫(1 − 2 sin2 𝑥 + sin4 𝑥) cos 𝑥 𝑑𝑥

2 1
= sin 𝑥 − sin3 𝑥 + sin5 𝑥 + 𝐶.
3 5

Example 25: Evaluate

cos 3 𝑥
∫3 𝑑𝑥
√sin 𝑥

Solution:

cos 3 𝑥 −
1
3 −
1
∫3 𝑑𝑥 = ∫ sin 3 𝑥 cos 𝑥 𝑑𝑥 = ∫ sin 3𝑥 cos 2 𝑥 cos 𝑥 𝑑𝑥
√sin 𝑥
1
= ∫ sin−3 𝑥 (1 − sin2 𝑥) cos 𝑥 𝑑𝑥

1 5
= ∫ (sin−3 𝑥 − sin3 𝑥) cos 𝑥 𝑑𝑥

50
Chapter 2: TECHNIQUES OF INTEGRATION
3 2 3 8
= sin3 𝑥 − sin3 𝑥 + 𝐶.
2 8

Example 26: Evaluate ∫ sin3 𝑥 cos 3 𝑥 𝑑𝑥

Solution:

∫ sin3 𝑥 cos 2 𝑥 𝑑𝑥 = ∫ sin3 𝑥 ∙ cos 2 𝑥 ∙ cos 𝑥 𝑑𝑥

= ∫ sin3 𝑥 (1 − sin2 𝑥) cos 𝑥 𝑑𝑥

= ∫(sin3 𝑥 − sin5 𝑥) cos 𝑥 𝑑𝑥

1 1
= sin4 𝑥 − sin6 𝑥 + 𝐶.
4 6
Another Solution:

∫ sin3 𝑥 cos 2 𝑥 𝑑𝑥 = ∫ sin2 𝑥 ∙ cos 3 𝑥 ∙ sin 𝑥 𝑑𝑥

= ∫(1 − cos 2 𝑥) cos 3 𝑥 sin 𝑥 𝑑𝑥

= ∫(cos 3 𝑥 − cos 5 𝑥) sin 𝑥 𝑑𝑥

1 1
= − cos 4 𝑥 + cos 6 𝑥 + 𝐶.
4 6

Example 27: Evaluate ∫ sin2 𝑥 cos 2 𝑥 𝑑𝑥

Solution:

1 1
∫ sin2 𝑥 cos 2 𝑥 𝑑𝑥 = ∫ [ (1 − cos 2𝑥)] ∙ [ (1 + cos 2𝑥)] 𝑑𝑥
2 2

51
Chapter 2: TECHNIQUES OF INTEGRATION
1
= ∫(1 − cos 2 2𝑥) 𝑑𝑥
4
1 1
= ∫ [1 − (1 + cos 4𝑥)] 𝑑𝑥
4 2
1 1 1
= ∫ [ − cos 4𝑥] 𝑑𝑥
4 2 2
1
= ∫[1 − cos 4𝑥] 𝑑𝑥
8
1 1
= [𝑥 − sin 4𝑥] + 𝐶.
8 4
1 1
= 𝑥 − sin 4𝑥 + 𝐶.
8 32

Example 28: Evaluate ∫ sin4 𝑥 𝑑𝑥

Solution:

2
4 2 2
1
∫ sin 𝑥 𝑑𝑥 = ∫(sin 𝑥) 𝑑𝑥 = ∫ [ (1 − cos 2𝑥)] 𝑑𝑥
2
1
= ∫(1 − 2 cos 2𝑥 + cos 2 2𝑥) 𝑑𝑥
4
1 1
= ∫ (1 − 2 cos 2𝑥 + (1 − cos 4𝑥)) 𝑑𝑥
4 2
1 3 1
= ∫ ( − 2 cos 2𝑥 − cos 4𝑥) 𝑑𝑥
4 2 2
1 3 1 1 1
= [ 𝑥 − 2 ∙ sin 2𝑥 − ∙ sin 4𝑥] + 𝐶.
4 2 2 2 4
3 1 1
= 𝑥 − sin 2𝑥 − sin 4𝑥 + 𝐶.
8 4 32

52
Chapter 2: TECHNIQUES OF INTEGRATION
2.3.2 Integrating a Product of Powers of Secants and Tangents

We can use a similar strategy to evaluate integrals of the form ∫ tan𝑚 𝑥 sec 𝑛 𝑥 𝑑𝑥.
Since (𝑑⁄𝑑𝑥 ) tan 𝑥 = sec 2 𝑥, we can separate a sec 2 𝑥 factor and convert the
remaining (even) power of secant to an expression involving tangent using the
identity sec 2 𝑥 = 1 + tan2 𝑥 .

Or, since (𝑑⁄𝑑𝑥 ) sec 𝑥 = sec 𝑥 tan 𝑥, we can separate a sec 𝑥 tan 𝑥 factor and
convert the remaining (even) power of tangent to secant.

Strategy for Evaluating ∫ tan𝑚 𝑥 sec 𝑛 𝑥 𝑑𝑥

(a) If the power of secant is even (𝑛 = 2, 𝑘 ≥ 2), save a factor of sec 2 𝑥 and
use sec 2 𝑥 = 1 + tan2 𝑥 to express the remaining factors in terms of tan 𝑥:

∫ tan𝑚 𝑥 sec 2𝑘 𝑥 𝑑𝑥 = ∫ tan𝑚 𝑥 (sec 2 𝑥)𝑘−1 sec 2 𝑥 𝑑𝑥

= ∫ tan𝑚 𝑥 (1 + tan2 𝑥)𝑘−1 sec 2 𝑥 𝑑𝑥

Such that

1
∫ tan𝑘 𝑥 sec 2 𝑥 = tan𝑘+1 𝑥 + 𝐶.
𝑘+1

(b) If the power of tangent is odd 𝑚 = (2𝑘 + 1), save a factor of sec 𝑥 tan 𝑥
and use tan2 𝑥 = sec 2 𝑥 − 1 to express the remaining factors in terms of
sec 𝑥:

∫ tan2𝑘+1 𝑥 sec 𝑛 𝑥 = ∫(tan2 𝑥)𝑘 sec 𝑛−1 𝑥 sec 𝑥 tan 𝑥 𝑑𝑥

= ∫(sec 2 𝑥 − 1)𝑘 sec 𝑛−1 𝑥 sec 𝑥 tan 𝑥 𝑑𝑥

53
Chapter 2: TECHNIQUES OF INTEGRATION
Such that

1
∫ sec 𝑘 𝑥 sec 𝑥 tan 𝑥 𝑑𝑥 = sec 𝑘+1 𝑥 + 𝐶.
𝑘+1

Example 29: Evaluate ∫ tan6 𝑥 sec 4 𝑥 𝑑𝑥.

Solution: If we separate one sec 2 𝑥 factor, we can express the remaining sec 2 𝑥
factor in terms of tangent using the identity sec 2 𝑥 = 1 + tan2 𝑥. Then,

∫ tan6 𝑥 sec 4 𝑥 𝑑𝑥 = ∫ tan6 𝑥 sec 2 𝑥 ∙ sec 2 𝑥 𝑑𝑥

= ∫ tan6 𝑥 (1 + tan2 𝑥) sec 2 𝑥 𝑑𝑥

= ∫(tan6 𝑥 + tan8 𝑥) sec 2 𝑥 𝑑𝑥

1 1
= tan7 𝑥 + tan9 𝑥 + 𝐶.
7 9

Example 30: Evaluate ∫ tan3 𝑥 sec 5 𝑥 𝑑𝑥.

Solution:

∫ tan3 𝑥 sec 5 𝑥 𝑑𝑥 = ∫ tan2 𝑥 sec 4 𝑥 ∙ sec 𝑥 ∙ tan 𝑥 𝑑𝑥

= ∫(sec 2 𝑥 − 1) sec 4 𝑥 ∙ sec 𝑥 ∙ tan 𝑥 𝑑𝑥

= ∫(sec 6 𝑥 − sec 4 𝑥) ∙ sec 𝑥 tan 𝑥 𝑑𝑥

54
Chapter 2: TECHNIQUES OF INTEGRATION
1 1
= sec 7 𝑥 + sec 5 𝑥 + 𝐶.
7 5

Example 31: Evaluate ∫ tan2 𝑥 sec 4 𝑥 𝑑𝑥.

Solution:

∫ tan2 𝑥 sec 4 𝑥 𝑑𝑥 = ∫ tan2 𝑥 sec 2 𝑥 ∙ sec 2 𝑥 𝑑𝑥

= ∫ tan2 𝑥 (1 + tan2 𝑥) sec 2 𝑥 𝑑𝑥

= ∫(tan2 𝑥 + tan4 𝑥) sec 2 𝑥 𝑑𝑥

1 1
= tan3 𝑥 + tan5 𝑥 + 𝐶.
3 5

Example 32: Evaluate ∫ tan5 𝑥 sec 6 𝑥 𝑑𝑥.

Solution:

∫ tan5 𝑥 sec 6 𝑥 𝑑𝑥 = ∫ tan4 𝑥 sec 5 𝑥 ∙ sec 𝑥 ∙ tan 𝑥 𝑑𝑥

= ∫(tan2 𝑥)2 sec 5 𝑥 ∙ sec 𝑥 ∙ tan 𝑥 𝑑𝑥

= ∫(sec 2 𝑥 − 1)2 sec 5 𝑥 ∙ sec 𝑥 ∙ tan 𝑥 𝑑𝑥

= ∫(sec 4 𝑥 − 2 sec 2 𝑥 + 1) sec 5 𝑥 ∙ sec 𝑥 ∙ tan 𝑥 𝑑𝑥

55
Chapter 2: TECHNIQUES OF INTEGRATION
= ∫(sec 9 𝑥 − 2 sec 7 𝑥 + sec 5 𝑥) ∙ sec 𝑥 tan 𝑥 𝑑𝑥

1 1 1
= sec10 𝑥 − sec 8 𝑥 + sec 6 𝑥 + 𝐶.
10 4 6

Another Solution:

∫ tan5 𝑥 sec 6 𝑥 𝑑𝑥 = ∫ tan5 𝑥 sec 4 𝑥 ∙ sec 2 𝑥 𝑑𝑥

= ∫ tan5 𝑥 (1 + tan2 𝑥)2 ∙ sec 2 𝑥 𝑑𝑥

= ∫ tan5 𝑥 (1 + +2 tan2 𝑥 + tan4 𝑥) ∙ sec 2 𝑥 𝑑𝑥

= ∫(tan5 𝑥 + +2 tan7 𝑥 + tan9 𝑥) ∙ sec 2 𝑥 𝑑𝑥

1 1 1
= tan6 𝑥 + tan8 𝑥 + tan10 𝑥 + 𝐶.
6 4 10

Example 33: Evaluate ∫ tan4 𝑥 𝑑𝑥.

Solution:

∫ tan4 𝑥 𝑑𝑥 = ∫ tan2 𝑥 ∙ tan2 𝑥 𝑑𝑥

= ∫( sec 2 𝑥 − 1) ∙ tan2 𝑥 𝑑𝑥

= ∫ sec 2 𝑥 tan2 𝑥 𝑑𝑥 − ∫ tan2 𝑥 𝑑𝑥

56
Chapter 2: TECHNIQUES OF INTEGRATION
= ∫ sec 2 𝑥 tan2 𝑥 𝑑𝑥 − ∫( sec 2 𝑥 − 1) 𝑑𝑥

1
= tan3 𝑥 − tan 𝑥 + 𝑥 + 𝐶.
3

Example 34: Evaluate ∫ sec 4 𝑥 𝑑𝑥.

Solution:

∫ sec 4 𝑥 𝑑𝑥 = ∫ sec 2 𝑥 ∙ sec 2 𝑥 𝑑𝑥

= ∫(1 + tan2 𝑥) ∙ sec 2 𝑥 𝑑𝑥

1
= tan 𝑥 + tan3 𝑥 + 𝐶.
3

2.4 Integration by Parts

Every differentiation rule has a corresponding integration rule. For instance, the
Substitution Rule for integration corresponds to the Chain Rule for differentiation.
The rule that corresponds to the Product Rule for differentiation is called the rule for
integration by parts.

The Product Rule states that if 𝑓 and g are differentiable functions, then

𝑑
[𝑓(𝑥)g(𝑥)] = 𝑓(𝑥)g ′ (𝑥) + 𝑓 ′ (𝑥)g(𝑥)
𝑑𝑥

In the notation for indefinite integrals this equation becomes

57
Chapter 2: TECHNIQUES OF INTEGRATION
∫[𝑓(𝑥)g ′ (𝑥) + 𝑓 ′ (𝑥)g(𝑥)]𝑑𝑥 = 𝑓(𝑥)g(𝑥)

Or

∫ 𝑓(𝑥)g ′ (𝑥)𝑑𝑥 + ∫ 𝑓 ′ (𝑥)g(𝑥)𝑑𝑥 = 𝑓(𝑥)g(𝑥)

We can rearrange this equation as

∫ 𝑓(𝑥)g ′ (𝑥)𝑑𝑥 = 𝑓(𝑥)g(𝑥) − ∫ 𝑓 ′ (𝑥)g(𝑥)𝑑𝑥

This formula is called the formula for integration by parts. It is perhaps easier to
remember in the following notation.

Let 𝑢 = 𝑓(𝑥) and 𝑣 = g(𝑥). Then the differentials are 𝑑𝑢 = 𝑓′(𝑥) and 𝑑𝑣 = g′(𝑥),
so by substitution rule, the formula of integration by parts becomes

∫ 𝑢 𝑑𝑣 = 𝑢𝑣 − ∫ 𝑣 𝑑𝑢.

Example 35: Evaluate ∫ 𝑥 sin 𝑥 𝑑𝑥.

Solution: Let
𝑢=𝑥 𝑑𝑣 = sin 𝑥 𝑑𝑥

𝑑𝑢 = 𝑑𝑥 𝑣 = − cos 𝑥

Then,

𝑢 𝑑𝑣 𝑢 𝑣 𝑣 𝑑𝑢
𝑥⏞
∫ 𝑥 sin 𝑥 𝑑𝑥 = ∫ ⏞ 𝑥⏞
sin 𝑥 𝑑𝑥 = ⏞ (−cos 𝑥) − ∫ ⏞ ⏞
(−cos 𝑥) 𝑑𝑥

58
Chapter 2: TECHNIQUES OF INTEGRATION
= −𝑥 cos 𝑥 + ∫ cos 𝑥 𝑑𝑥

= −𝑥 cos 𝑥 + sin 𝑥 + 𝐶.

Example 36: Evaluate ∫ 𝑥 𝑒 𝑥 𝑑𝑥.

Solution: Let
𝑢=𝑥 𝑑𝑣 = 𝑒 𝑥 𝑑𝑥

𝑑𝑢 = 𝑑𝑥 𝑣 = 𝑒𝑥

Then,

𝑢 𝑑𝑣 𝑢 𝑣 𝑣 𝑑𝑢
𝑥
∫ 𝑥 𝑒 𝑑𝑥 = 𝑥⏞
∫⏞ 𝑥
𝑒 𝑑𝑥 ⏞
𝑥 (𝑒
=⏞ ⏞
𝑥 ) − ∫ (𝑒 ⏞
𝑥 ) 𝑑𝑥

= 𝑥 𝑒 𝑥 − ∫ 𝑒 𝑥 𝑑𝑥

= 𝑥 𝑒 𝑥 = 𝑒 𝑥 + 𝐶.

Example 37: Evaluate ∫ 𝑥 cos 𝑥 𝑑𝑥.

Solution: Let
𝑢=𝑥 𝑑𝑣 = cos 𝑥 𝑑𝑥

𝑑𝑢 = 𝑑𝑥 𝑣 = sin 𝑥

Then,

59
Chapter 2: TECHNIQUES OF INTEGRATION
∫ 𝑥 cos 𝑥 𝑑𝑥 = 𝑥 sin 𝑥 − ∫ sin 𝑥 𝑑𝑥

= 𝑥 sin 𝑥 − cos 𝑥 + 𝐶.

Example 38: Evaluate ∫ 𝑥 2 𝑒 𝑥 𝑑𝑥.

Solution: Let
𝑢 = 𝑥2 𝑑𝑣 = 𝑒 𝑥 𝑑𝑥

𝑑𝑢 = 2𝑥𝑑𝑥 𝑣 = 𝑒𝑥

Then,

∫ 𝑥 2 𝑒 𝑥 𝑑𝑥 = 𝑥 2 𝑒 𝑥 − 2 ∫ 𝑥 𝑒 𝑥 𝑑𝑥

Let
𝑢=𝑥 𝑑𝑣 = 𝑒 𝑥 𝑑𝑥

𝑑𝑢 = 𝑑𝑥 𝑣 = 𝑒𝑥

Then,

∫ 𝑥 2 𝑒 𝑥 𝑑𝑥 = 𝑥 2 𝑒 𝑥 − 2 [𝑥 𝑒 𝑥 − ∫ 𝑒 𝑥 𝑑𝑥]

= 𝑥 2 𝑒 𝑥 − 2[𝑥 𝑒 𝑥 − 𝑒 𝑥 ]

= 𝑥 2 𝑒 𝑥 − 2𝑥 𝑒 𝑥 + 2𝑒 𝑥 + 𝐶.

Another Solution: We start off by choosing 𝑢 and 𝑑𝑣 as we always would.


However, instead of computing 𝑑𝑢 and 𝑣 we put these into the following table. We
then differentiate down the column corresponding to 𝑢 until we hit zero. In the

60
Chapter 2: TECHNIQUES OF INTEGRATION
column corresponding to 𝑑𝑣 we integrate once for each entry in the first column.

𝒖(𝑫) 𝒗(𝐼)

𝑥2 𝑒𝑥
+
2𝑥 𝑒𝑥

𝑥 𝑒𝑥
+
0 − 𝑒𝑥

∫ 𝑥 2 𝑒 𝑥 𝑑𝑥 = 𝑥 2 𝑒 𝑥 − 2𝑥 𝑒 𝑥 + 2𝑒 𝑥 + 𝐶.

Example 39: Evaluate ∫ 𝑥 3 sin 𝑥 𝑑𝑥.

Solution:

𝒖(𝑫) 𝒗(𝐼)

𝑥3 sin 𝑥
+
3𝑥 2 − cos 𝑥
−1111111
6𝑥 − sin 𝑥
+
𝑥 cos 𝑥

0 + sin 𝑥

∫ 𝑥 3 sin 𝑥 𝑑𝑥 = −𝑥 3 cos 𝑥 + 3𝑥 2 sin 𝑥 + 6𝑥 cos 𝑥 − 𝑥 sin 𝑥 + 𝐶.

61
Chapter 2: TECHNIQUES OF INTEGRATION
Example 40: Evaluate ∫ 𝑥√𝑥 + 1 𝑑𝑥.

Solution: Let
𝑢=𝑥 𝑑𝑣 = √𝑥 + 1𝑑𝑥
𝑑𝑢 = 𝑑𝑥 2 3
𝑣 = (𝑥 + 1)2
3
Then,

2 3 2 3
∫ 𝑥√𝑥 + 1 𝑑𝑥 = 𝑥(𝑥 + 1)2 − ∫(𝑥 + 1)2 𝑑𝑥
3 3

2 3 4 5
= 𝑥(𝑥 + 1)2 − (𝑥 + 1)2 + 𝐶.
3 15

Example 41: Evaluate ∫ ln 𝑥 𝑑𝑥.

Solution: Let
𝑢 = ln 𝑥 𝑑𝑣 = 𝑑𝑥

1 𝑣=𝑥
𝑑𝑢 = 𝑑𝑥
𝑥
Then,

∫ ln 𝑥 𝑑𝑥 = 𝑥 ln 𝑥 − ∫ 𝑑𝑥

= 𝑥 ln 𝑥 − 𝑥 + 𝐶.

Example 41: Evaluate ∫ 𝑥 ln 𝑥 𝑑𝑥.

62
Chapter 2: TECHNIQUES OF INTEGRATION
Solution: Let
𝑢 = ln 𝑥 𝑑𝑣 = 𝑥𝑑𝑥
1 1
𝑑𝑢 = 𝑑𝑥 𝑣 = 𝑥2
𝑥 2
Then,

1 1
∫ ln 𝑥 𝑑𝑥 = 𝑥 2 ln 𝑥 − ∫ 𝑥𝑑𝑥
2 2

1 1
= 𝑥 2 ln 𝑥 − 𝑥 2 + 𝐶.
2 4

Example 42: Evaluate ∫ ln(𝑥 2 + 1) 𝑑𝑥.

Solution: Let
𝑢 = ln(𝑥 2 + 1) 𝑑𝑣 = 𝑑𝑥

2𝑥 𝑣=𝑥
𝑑𝑢 = 𝑑𝑥
𝑥2 + 1
Then,

2 2
𝑥2
∫ ln(𝑥 + 1) 𝑑𝑥 = 𝑥 ln(𝑥 + 1) − 2 ∫ 2 𝑑𝑥
𝑥 +1

2
𝑥2 + 1 − 1
= 𝑥 ln(𝑥 + 1) − 2 ∫ 𝑑𝑥
𝑥2 + 1

1
= 𝑥 ln(𝑥 2 + 1) − 2 ∫ [1 − ] 𝑑𝑥
𝑥2 + 1

= 𝑥 ln(𝑥 2 + 1) − 2[𝑥 − tan−1 𝑥] + 𝐶.

63
Chapter 2: TECHNIQUES OF INTEGRATION
Example 43: Evaluate ∫ sin−1 𝑥 𝑑𝑥.

Solution: Let
𝑢 = sin−1 𝑥 𝑑𝑣 = 𝑑𝑥

1 𝑣=𝑥
𝑑𝑢 = 𝑑𝑥
√1 − 𝑥 2
Then,

𝑥
∫ sin−1 𝑥 𝑑𝑥 = 𝑥 sin−1 𝑥 − ∫ 𝑑𝑥
√1 − 𝑥 2

1 −2𝑥
= 𝑥 sin−1 𝑥 + ∫ 𝑑𝑥
2 √1 − 𝑥 2

= 𝑥 sin−1 𝑥 + √1 − 𝑥 2 + 𝐶.

Example 44: Evaluate ∫ tan−1 𝑥 𝑑𝑥.

Solution: Let
𝑢 = tan−1 𝑥 𝑑𝑣 = 𝑑𝑥

1 𝑣=𝑥
𝑑𝑢 = 𝑑𝑥
1 + 𝑥2
Then,

𝑥
∫ tan−1 𝑥 𝑑𝑥 = 𝑥 tan−1 𝑥 − ∫ 𝑑𝑥
1 + 𝑥2

1 2𝑥
= 𝑥 tan−1 𝑥 − ∫ 𝑑𝑥
2 1 + 𝑥2

1
= 𝑥 tan−1 𝑥 + ln(1 + 𝑥 2 ) + 𝐶.
2

64
Chapter 2: TECHNIQUES OF INTEGRATION
Example 45: Evaluate ∫ 𝑥 tan−1 𝑥 𝑑𝑥.

Solution: Let
𝑢 = tan−1 𝑥 𝑑𝑣 = 𝑥𝑑𝑥

1 1
𝑑𝑢 = 𝑑𝑥 𝑣 = 𝑥2
1 + 𝑥2 2
Then,

−1
1 2 −1
1 𝑥2
∫ 𝑥 tan 𝑥 𝑑𝑥 = 𝑥 tan 𝑥 − ∫ 𝑑𝑥
2 2 1 + 𝑥2

1 2 −1
1 𝑥2 + 1 − 1
= 𝑥 tan 𝑥 − ∫ 𝑑𝑥
2 2 𝑥2 + 1

1 1 1
= 𝑥 2 tan−1 𝑥 − ∫ [1 − 2 ] 𝑑𝑥
2 2 𝑥 +1

1 1
= 𝑥 2 tan−1 𝑥 − [𝑥 − tan−1 𝑥] + 𝐶.
2 2

Example 46: Evaluate ∫ sin2 𝑥 𝑑𝑥.

Solution: Let
𝑢 = sin 𝑥 𝑑𝑣 = sin 𝑥 𝑑𝑥

𝑑𝑢 = cos 𝑥 𝑑𝑥 𝑣 = − cos 𝑥

Then,

∫ sin2 𝑥 𝑑𝑥 = − sin 𝑥 cos 𝑥 + ∫ cos 2 𝑥 𝑑𝑥

65
Chapter 2: TECHNIQUES OF INTEGRATION
= − sin 𝑥 cos 𝑥 + ∫(1 − sin2 𝑥) 𝑑𝑥

∫ sin2 𝑥 𝑑𝑥 = − sin 𝑥 cos 𝑥 + ∫ 𝑑𝑥 − ∫ sin2 𝑥 𝑑𝑥

2 ∫ sin2 𝑥 𝑑𝑥 = − sin 𝑥 cos 𝑥 + 𝑥 + 𝐶.

1
∫ sin2 𝑥 𝑑𝑥 = [− sin 𝑥 cos 𝑥 + 𝑥 + 𝐶].
2

Example 47: Evaluate ∫ sec 3 𝑥 𝑑𝑥.

Solution: Let
𝑢 = sec 𝑥 𝑑𝑣 = sec 2 𝑥 𝑑𝑥

𝑑𝑢 = sec 𝑥 tan 𝑥 𝑑𝑥 𝑣 = tan 𝑥

Then,

∫ sec 3 𝑥 𝑑𝑥 = sec 𝑥 tan 𝑥 − ∫ sec 𝑥 tan2 𝑥 𝑑𝑥

= sec 𝑥 tan 𝑥 − ∫ sec 𝑥 (sec 2 𝑥 − 1) 𝑑𝑥

∫ sec 3 𝑥 𝑑𝑥 = sec 𝑥 tan 𝑥 − ∫ sec 3 𝑥 𝑑𝑥 + ∫ sec 𝑥 𝑑𝑥

2 ∫ sec 3 𝑥 𝑑𝑥 = sec 𝑥 tan 𝑥 + ln|sec 𝑥 + tan 𝑥| + 𝐶.

1
∫ sec 3 𝑥 𝑑𝑥 = [sec 𝑥 tan 𝑥 + ln|sec 𝑥 + tan 𝑥| + 𝐶].
2

66
Chapter 2: TECHNIQUES OF INTEGRATION
Example 48: Evaluate ∫ 𝑒 𝑥 sin 𝑥 𝑑𝑥.

Solution: Let
𝑢 = 𝑒𝑥 𝑑𝑣 = sin 𝑥 𝑑𝑥

𝑑𝑢 = 𝑒 𝑥 𝑑𝑥 𝑣 = − cos 𝑥

Then,

∫ 𝑒 𝑥 sin 𝑥 𝑑𝑥 = −𝑒 𝑥 cos 𝑥 + ∫ 𝑒 𝑥 cos 𝑥 𝑑𝑥

Let
𝑢 = 𝑒𝑥 𝑑𝑣 = cos 𝑥 𝑑𝑥

𝑑𝑢 = 𝑒 𝑥 𝑑𝑥 𝑣 = sin 𝑥

Then,

∫ 𝑒 𝑥 sin 𝑥 𝑑𝑥 = −𝑒 𝑥 cos 𝑥 + [𝑒 𝑥 sin 𝑥 − ∫ 𝑒 𝑥 sin 𝑥 𝑑𝑥]

2 ∫ 𝑒 𝑥 sin 𝑥 𝑑𝑥 = −𝑒 𝑥 cos 𝑥 + 𝑒 𝑥 sin 𝑥 + 𝐶

1
∫ 𝑒 𝑥 sin 𝑥 𝑑𝑥 = [−𝑒 𝑥 cos 𝑥 + 𝑒 𝑥 sin 𝑥 + 𝐶]
2

𝑒𝑥
= [sin 𝑥 − cos 𝑥 + 𝐶].
2

Find another solution?

67
Chapter 2: TECHNIQUES OF INTEGRATION
Example 49: Evaluate ∫ sin(ln 𝑥) 𝑑𝑥.

Solution: using substitution rule, let


𝑢 = ln 𝑥 𝑥 = 𝑒𝑢

𝑑𝑥 = 𝑒 𝑢 𝑑𝑢

Then,

∫ sin(ln 𝑥) 𝑑𝑥 = ∫ 𝑒 𝑢 sin 𝑢 𝑑𝑢

𝑒𝑢
= [sin 𝑢 − cos 𝑢 + 𝐶].
2
𝑥
= [sin(ln 𝑥) − cos(ln 𝑥) + 𝐶].
2

Example 50: Evaluate

1 + 𝑥 ln 𝑥 𝑥
∫( ) 𝑒 𝑑𝑥.
𝑥

Solution:

1 + 𝑥 ln 𝑥 𝑥 𝑒𝑥
∫( ) 𝑒 𝑑𝑥 = ∫ 𝑑𝑥 + ∫ 𝑒 𝑥 ln 𝑥 𝑑𝑥
𝑥 𝑥

Let
𝑢 = ln 𝑥 𝑑𝑣 = 𝑒 𝑥 𝑑𝑥
1 𝑣 = 𝑒𝑥
𝑑𝑢 = 𝑑𝑥
𝑥
Then,

68
Chapter 2: TECHNIQUES OF INTEGRATION
𝑥
1 + 𝑥 ln 𝑥 𝑥 𝑒
∫( ) 𝑒 𝑑𝑥 = ∫ 𝑑𝑥 + ∫ 𝑒 𝑥 ln 𝑥 𝑑𝑥
𝑥 𝑥

𝑒𝑥 𝑥
𝑒𝑥
= ∫ 𝑑𝑥 + [𝑒 ln 𝑥 − ∫ 𝑑𝑥]
𝑥 𝑥

= 𝑒 𝑥 ln 𝑥 + 𝐶.

Example 51: Evaluate

1 + sin 𝑥 𝑥
∫( ) 𝑒 𝑑𝑥.
1 + cos 𝑥

Solution: use the identities

𝑥 𝑥
sin 𝑥 = 2 sin ( ) cos ( )
2 2
𝑥
cos 𝑥 = 2 cos 2 ( ) − 1
2
𝑥
1 + cos 𝑥 = 2 cos 2 ( )
2
𝑥 𝑥
1 + sin 𝑥 𝑥 1 + 2 sin ( ) cos sin ( )
∫( ) 𝑒 𝑑𝑥 = ∫ [ 2 2 ] 𝑒 𝑥 𝑑𝑥
1 + cos 𝑥 𝑥
2 cos 2 (2)

1 𝑥 𝑥
= ∫ sec 2 ( ) 𝑒 𝑥 𝑑𝑥 + ∫ tan ( ) 𝑒 𝑥 𝑑𝑥
2 2 2

Let
𝑢 = 𝑒𝑥 1 𝑥
𝑑𝑣 = sec 2 ( ) 𝑑𝑥
2 2
𝑑𝑢 = 𝑒 𝑥 𝑑𝑥 𝑥
𝑣 = tan ( )
2

69
Chapter 2: TECHNIQUES OF INTEGRATION
Then,

1 + sin 𝑥 𝑥 1 𝑥 𝑥
∫( ) 𝑒 𝑑𝑥 = ∫ sec 2 ( ) 𝑒 𝑥 𝑑𝑥 + ∫ tan ( ) 𝑒 𝑥 𝑑𝑥
1 + cos 𝑥 2 2 2

𝑥 𝑥 𝑥
= 𝑒 𝑥 tan ( ) − ∫ tan ( ) 𝑒 𝑥 𝑑𝑥 + ∫ tan ( ) 𝑒 𝑥 𝑑𝑥
2 2 2
𝑥
= 𝑒 𝑥 tan ( ) + 𝐶.
2

2.4.1 Integration by Reduction

Integration by reduction is a technique used to simplify the process of integrating


complex functions by reducing them to simpler forms. This method often involves
using a recurrence relation to express the integral of a function in terms of the
integral of a simpler function. Here's a step-by-step outline of the process:

1. Identify the Integral: Start with the integral you need to solve, which is often
in a form that is difficult to integrate directly.

2. Set Up a Recurrence Relation: Express the integral in terms of a simpler


integral. This usually involves using integration by parts or some other
algebraic manipulation to create a relationship between the original integral
and a simpler one.

3. Solve the Recurrence Relation: Use the recurrence relation to reduce the
original integral step by step until you reach a base case that is easy to
integrate.

4. Combine Results: Once you have the base case, you can work backwards
using the recurrence relation to find the value of the original integral.

70
Chapter 2: TECHNIQUES OF INTEGRATION
Example 52: Prove the reduction formula:

1 𝑛−1
∫ sin𝑛 𝑥 𝑑𝑥 = − cos 𝑥 sin𝑛−1 𝑥 + ∫ sin𝑛−2 𝑥 𝑑𝑥.
𝑛 𝑛

Where 𝑛 ≥ 2 is an integer.

Solution: Let
𝑢 = sin𝑛−1 𝑥 𝑑𝑣 = sin 𝑥 𝑑𝑥

𝑑𝑢 = (𝑛 − 1) sin𝑛−2 𝑥 cos 𝑥 𝑑𝑥 𝑣 = − cos 𝑥

Then,

∫ sin𝑛 𝑥 𝑑𝑥 = − sin𝑛−1 𝑥 cos 𝑥 + (𝑛 − 1) ∫ sin𝑛−2 𝑥 cos 2 𝑥 𝑑𝑥

Since cos 2 𝑥 = 1 − sin2 𝑥, then

∫ sin𝑛 𝑥 𝑑𝑥 = − sin𝑛−1 𝑥 cos 𝑥 + (𝑛 − 1) ∫ sin𝑛−2 𝑥 (1 − sin2 𝑥) 𝑑𝑥

∫ sin𝑛 𝑥 𝑑𝑥 = − sin𝑛−1 𝑥 cos 𝑥 + (𝑛 − 1) ∫ sin𝑛−2 𝑥 𝑑𝑥 − (𝑛 − 1) ∫ sin𝑛 𝑥 𝑑𝑥

𝑛 ∫ sin𝑛 𝑥 𝑑𝑥 = − sin𝑛−1 𝑥 cos 𝑥 + (𝑛 − 1) ∫ sin𝑛−2 𝑥 𝑑𝑥

1
∫ sin𝑛 𝑥 𝑑𝑥 = [− sin𝑛−1 𝑥 cos 𝑥 + (𝑛 − 1) ∫ sin𝑛−2 𝑥 𝑑𝑥]
𝑛

1 𝑛−1
= − sin𝑛−1 𝑥 cos 𝑥 + ∫ sin𝑛−2 𝑥 𝑑𝑥. ∎
𝑛 𝑛

71
Chapter 2: TECHNIQUES OF INTEGRATION
Example 53: Prove the reduction formula:

1 𝑛 𝑎𝑥 𝑛
∫ 𝑥 𝑛 𝑒 𝑎𝑥 𝑑𝑥 = 𝑥 𝑒 − ∫ 𝑥 𝑛−1 𝑒 𝑎𝑥 𝑑𝑥,
𝑎 𝑎

where 𝑛 is a positive integer. Then find

∫ 𝑥 4 𝑒 2𝑥 𝑑𝑥.

Solution: Let
𝑢 = 𝑥𝑛 𝑑𝑣 = 𝑒 𝑎𝑥 𝑑𝑥

𝑑𝑢 = 𝑛 𝑥 𝑛−1 𝑑𝑥 𝑣 = 𝑒 𝑎𝑥
1
𝑎

Then,

1 𝑛 𝑎𝑥 𝑛
∫ 𝑥 𝑛 𝑒 𝑎𝑥 𝑑𝑥 = 𝑥 𝑒 − ∫ 𝑥 𝑛−1 𝑒 𝑎𝑥 𝑑𝑥. ∎
𝑎 𝑎

Let

I𝑛 = ∫ 𝑥 𝑛 𝑒 𝑎𝑥 𝑑𝑥

Then,

I𝑛−1 = ∫ 𝑥 𝑛−1 𝑒 𝑎𝑥 𝑑𝑥

1 𝑛 𝑎𝑥 𝑛
∴ I𝑛 = 𝑥 𝑒 − I𝑛−1
𝑎 𝑎
1
I4 = ∫ 𝑥 4 𝑒 2𝑥 𝑑𝑥 = 𝑥 4 𝑒 2𝑥 − 2I3
2

1 3
I3 = ∫ 𝑥 3 𝑒 2𝑥 𝑑𝑥 = 𝑥 3 𝑒 2𝑥 − I2
2 2

72
Chapter 2: TECHNIQUES OF INTEGRATION
1
I2 = ∫ 𝑥 2 𝑒 2𝑥 𝑑𝑥 = 𝑥 2 𝑒 2𝑥 − I1
2

1 1
I1 = ∫ 𝑥 𝑒 2𝑥 𝑑𝑥 = 𝑥𝑒 2𝑥 − I0
2 2

1
I0 = ∫ 𝑒 2𝑥 𝑑𝑥 = 𝑒 2𝑥 + 𝐶
2

4 2𝑥 2𝑥
𝑥4 3
3𝑥 2 3𝑥 3
∴ I4 = ∫ 𝑥 𝑒 𝑑𝑥 = 𝑒 [ −𝑥 + − + ] + 𝐶.
2 2 2 4

Example 54: Find the reduction formula for the integration:

∫ 𝑥 𝑚 (ln 𝑥)𝑛 𝑑𝑥,

where 𝑚 ≠ −1 and 𝑛 > 0.

Solution: Let
𝑢 = (ln 𝑥)𝑛 𝑑𝑣 = 𝑥 𝑚 𝑑𝑥

𝑛−1
1 𝑥 𝑚+1
𝑑𝑢 = 𝑛(ln 𝑥) 𝑑𝑥 𝑣=
𝑥 𝑚+1
Then,

𝑚 𝑛
𝑥 𝑚+1 𝑛 1
∫ 𝑥 (ln 𝑥) 𝑑𝑥 = (ln 𝑥)𝑛 − ∫ 𝑥 𝑚+1 (ln 𝑥)𝑛−1 𝑑𝑥
𝑚+1 𝑚+1 𝑥

1
= [𝑥 𝑚+1 (ln 𝑥)𝑛 − 𝑛 ∫ 𝑥 𝑚 (ln 𝑥)𝑛−1 𝑑𝑥] . ∎
𝑚+1

Or

73
Chapter 2: TECHNIQUES OF INTEGRATION
𝑚 (ln 𝑛
𝑥 𝑚+1 𝑛
I𝑚,𝑛 = ∫𝑥 𝑥) 𝑑𝑥 = (ln 𝑥)𝑛 − I .
𝑚+1 𝑚 + 1 𝑚,𝑛−1

Example 55: Find the reduction formula for the integration:

∫ tan𝑛 𝑥 𝑑𝑥,

where 𝑛 > 1.

Solution:

I𝑛 = ∫ tan𝑛 𝑥 𝑑𝑥 = ∫ tan𝑛−2 𝑥 ∙ tan2 𝑥 𝑑𝑥

= ∫ tan𝑛−2 𝑥(sec 2 𝑥 − 1) 𝑑𝑥

= ∫ tan𝑛−2 𝑥 sec 2 𝑥 𝑑𝑥 − ∫ tan𝑛−2 𝑥 𝑑𝑥

1
∴ I𝑛 = tan𝑛−1 𝑥 − I𝑛−2 . ∎
𝑛−1

Example 56: Find the reduction formula for the integration:

∫ sec 𝑛 𝑥 𝑑𝑥.

Solution:

I𝑛 = ∫ sec 𝑛 𝑥 𝑑𝑥 = ∫ sec 𝑛−2 𝑥 ∙ sec 2 𝑥 𝑑𝑥

74
Chapter 2: TECHNIQUES OF INTEGRATION
let
𝑢 = sec 𝑛−2 𝑥 𝑑𝑣 = sec 2 𝑥 𝑑𝑥

𝑑𝑢 = (𝑛 − 2)sec 𝑛−3 𝑥 ∙ sec 𝑥 tan 𝑥 𝑑𝑥 𝑣 = tan 𝑥


= (𝑛 − 2)sec 𝑛−2 𝑥 ∙ tan 𝑥 𝑑𝑥
Then,

I𝑛 = ∫ sec 𝑛 𝑥 𝑑𝑥 = sec 𝑛−2 𝑥 tan 𝑥 − (𝑛 − 2) ∫ sec 𝑛−2 𝑥 ∙ tan2 𝑥 𝑑𝑥

= sec 𝑛−2 𝑥 tan 𝑥 − (𝑛 − 2) ∫ sec 𝑛−2 𝑥(sec 2 𝑥 − 1)𝑑𝑥

= sec 𝑛−2 𝑥 tan 𝑥 − (𝑛 − 2) ∫ sec 𝑛 𝑥 𝑑𝑥 + (𝑛 − 2) ∫ sec 𝑛−2 𝑥 𝑑𝑥

I𝑛 = sec 𝑛−2 𝑥 tan 𝑥 − (𝑛 − 2)I𝑛 + (𝑛 − 2)I𝑛−2

(1 + 𝑛 − 2)I𝑛 = sec 𝑛−2 𝑥 tan 𝑥 + (𝑛 − 2)I𝑛−2

(𝑛 − 1)I𝑛 = sec 𝑛−2 𝑥 tan 𝑥 + (𝑛 − 2)I𝑛−2

1 𝑛−2
∴ I𝑛 = sec 𝑛−2 𝑥 tan 𝑥 + I . ∎
𝑛−1 𝑛 − 1 𝑛−2

75
Chapter 2: TECHNIQUES OF INTEGRATION
2.5 Integration of Rational Functions by Partial Fractions

In this section we show how to integrate any rational function (a ratio of


polynomials) by expressing it as a sum of simpler fractions, called partial fractions,
that we already know how to integrate.

To illustrate the method, observe that by taking the fractions 2⁄𝑥 − 1 and 1⁄𝑥 + 2
to a common denominator we obtain

2 1 2(𝑥 + 2) − (𝑥 − 1) 𝑥+5
− = = 2
𝑥−1 𝑥+2 (𝑥 − 1)(𝑥 + 2) 𝑥 +𝑥−2

If we now reverse the procedure, we see how to integrate the function on the right
side of this equation:

𝑥+5 2 1
∫ 𝑑𝑥 = ∫ ( − ) 𝑑𝑥 = 2 ln|𝑥 − 1| − ln|𝑥 + 2| + 𝐶.
𝑥2 + 𝑥 − 2 𝑥−1 𝑥+2

To see how the method of partial fractions works in general, let’s consider a rational
function

𝑃(𝑥)
𝑓(𝑥) =
𝑄(𝑥)

where 𝑃(𝑥) and 𝑄(𝑥) are polynomials. It’s possible to express 𝑓 as a sum of simpler
fractions provided that the degree 𝑃 of is less than the degree of 𝑄. Such a rational
function is called proper. Recall that if

𝑃(𝑥) = 𝑎𝑛 𝑥 𝑛 + 𝑎𝑛−1 𝑥 𝑛−1 + ⋯ + 𝑎1 𝑥 + 𝑎0 ,

where 𝑎𝑛 ≠ 0, then the degree of 𝑃 is 𝑛 and we write deg(𝑃) = 𝑛.

If 𝑓 is improper, that is, deg(𝑃) ≥ deg (𝑄), then we must take the preliminary step

76
Chapter 2: TECHNIQUES OF INTEGRATION
of dividing 𝑄 into 𝑃 (by long division) until a remainder 𝑅(𝑥) is obtained such that
deg(𝑅) < deg (𝑄). The division statement is

𝑃(𝑥) 𝑅(𝑥)
𝑓(𝑥) = = 𝑆(𝑥) + 𝑓(𝑥) =
𝑄(𝑥) 𝑄(𝑥)

Where 𝑅 and 𝑆 are also polynomials.

Example 57: Evaluate

𝑥3 + 𝑥
∫ 𝑑𝑥.
𝑥−1

Solution: Since the degree of the numerator is greater than the degree of the
denominator, we first perform the long division. This enables us to write

𝑥3 + 𝑥 2
∫ 𝑑𝑥 = ∫ (𝑥 2 + 𝑥 + 2 + ) 𝑑𝑥
𝑥−1 𝑥−1

1 1
= 𝑥 3 + 𝑥 2 + 2𝑥 + 2 ln|𝑥 − 1| + 𝐶.
3 2

The next step is to factor the denominator 𝑄(𝑥) as far as possible. It can be shown
that any polynomial 𝑄 can be factored as a product of linear factors (of the form
𝑎𝑥 + 𝑏) and irreducible quadratic factors (of the form 𝑎𝑥 2 + 𝑏𝑥 + 𝑐), where 𝑏 2 −
4𝑎𝑐 < 0). For instance, if 𝑄(𝑥) = 𝑥 4 − 16, we could factor it as

𝑄(𝑥) = (𝑥 2 − 4)(𝑥 2 + 4) = (𝑥 − 2)(𝑥 + 2)(𝑥 2 + 4)

77
Chapter 2: TECHNIQUES OF INTEGRATION
The third step is to express the proper rational function 𝑃(𝑥)⁄𝑄(𝑥) as a sum of
partial fractions of the form

𝐴 𝐴𝑥 + 𝑏
𝑜𝑟
(𝑎𝑥 + 𝑏)𝑖 (𝑎𝑥 2 + 𝑏𝑥 + 𝑐)𝑖

We explain the details for the four cases that occur.

2.5.1 CASE I: The denominator Q(x) is a product of distinct linear factors

This means that we can write

𝑄(𝑥) = (𝑎1 𝑥 + 𝑏1 )(𝑎2 𝑥 + 𝑏2 ) … (𝑎𝑘 𝑥 + 𝑏𝑘 )

where no factor is repeated (and no factor is a constant multiple of another). In this


case the partial fraction theorem states that there exist constants 𝐴1 , 𝐴2 , … , 𝐴𝑘 such
that

𝑃(𝑥) 𝐴1 𝐴2 𝐴𝑘
= + +⋯+
𝑄(𝑥) 𝑎1 𝑥 + 𝑏1 𝑎2 𝑥 + 𝑏2 𝑎𝑘 𝑥 + 𝑏𝑘

These constants can be determined as in the following example.

Example 58: Evaluate

𝑥 2 + 2𝑥 − 1
∫ 3 𝑑𝑥.
2𝑥 + 3𝑥 2 − 2𝑥

Solution: Since the degree of the numerator is less than the degree of the
denominator, we don’t need to divide. We factor the denominator as

2𝑥 3 + 3𝑥 2 − 2𝑥 = 𝑥(2𝑥 2 + 3𝑥 − 2) = 𝑥(2𝑥 − 1)(𝑥 + 2)

78
Chapter 2: TECHNIQUES OF INTEGRATION
Since the denominator has three distinct linear factors, the partial fraction
decomposition has the form

𝑥 2 + 2𝑥 − 1 𝐴 𝐵 𝐶
= + +
2𝑥 3 + 3𝑥 2 − 2𝑥 𝑥 2𝑥 − 1 𝑥 + 2

To determine the values of 𝐴, 𝐵, and 𝐶, we multiply both sides of this equation by


the product of the denominators, 𝑥(2𝑥 − 1)(𝑥 + 2) , obtaining

𝑥 2 + 2𝑥 − 1 = 𝐴(2𝑥 − 1)(𝑥 + 2) + 𝐵𝑥(𝑥 + 2) + 𝐶𝑥(2𝑥 − 1)

Expanding the right side of the equation and writing it in the standard form for
polynomials, we get

𝑥 2 + 2𝑥 − 1 = (2𝐴 + 𝐵 + 2𝐶)𝑥 2 + (3𝐴 + 2𝐵 − 𝐶)𝑥 − 2𝐴

The polynomials in the previous equation are identical, so their coefficients must be
equal. The coefficient of 𝑥 2 on the right side, (2𝐴 + 𝐵 + 2𝐶), must equal the
coefficient of 𝑥 2 on the left side—namely, 1. Likewise, the coefficients of 𝑥 are
equal and the constant terms are equal. This gives the following system of equations
for 𝐴,𝐵, and 𝐶:

2𝐴 + 𝐵 + 2𝐶 = 1

3𝐴 + 2𝐵 − 𝐶 = 2

−2𝐴 = −1

Solving, we get,

1 1 1
𝐴= , 𝐵= and = −
2 5 10

Then,

79
Chapter 2: TECHNIQUES OF INTEGRATION
𝑥 2 + 2𝑥 − 1 1 1 1
∫ 3 𝑑𝑥 = ∫ [ + − ] 𝑑𝑥
2𝑥 + 3𝑥 2 − 2𝑥 2𝑥 5(2𝑥 − 1) 10(𝑥 + 2)

1 1 1
= ln 𝑥 + ln(2𝑥 − 1) − ln(𝑥 + 2) + 𝐶.
2 5 10

Example 59: Evaluate

4𝑥 2 − 13𝑥 − 9
∫ 3 𝑑𝑥.
𝑥 + 2𝑥 2 − 3𝑥

Solution: We factor the denominator as

𝑥 3 + 2𝑥 2 − 3𝑥 = 𝑥(𝑥 2 + 2𝑥 − 3) = 𝑥(𝑥 + 3)(𝑥 − 1)

Since the denominator has three distinct linear factors, the partial fraction
decomposition has the form

4𝑥 2 − 13𝑥 − 9 𝐴 𝐵 𝐶
= + +
𝑥 3 + 2𝑥 2 − 3𝑥 𝑥 𝑥 + 3 𝑥 − 1

To determine the values of 𝐴, 𝐵, and 𝐶, we multiply both sides of this equation by


the product of the denominators, 𝑥(𝑥 + 3)(𝑥 − 1), obtaining

4𝑥 2 − 13𝑥 − 9 = 𝐴(𝑥 + 3)(𝑥 − 1) + 𝐵𝑥(𝑥 − 1) + 𝐶𝑥(𝑥 + 3)

Here, we have 3 unknowns, 𝐴, 𝐵, and 𝐶, they can be founded by substituting values


for 𝑥 that make the various factors zero. If we substitute 𝑥 = 0, we get

−9 = −3𝐴 ⇒ 𝐴 = 3.

Letting 𝑥 = 1, we get

8 = 4𝐶 ⇒ 𝐶 = 2.

80
Chapter 2: TECHNIQUES OF INTEGRATION
Finally, 𝑥 = −3, we get

−12 = 12𝐵 ⇒ 𝐵 = −1.

Then,

4𝑥 2 − 13𝑥 − 9 3 1 2
∫ 3 𝑑𝑥 = ∫ [ − + ] 𝑑𝑥
𝑥 + 2𝑥 2 − 3𝑥 𝑥 𝑥+3 𝑥−1

= 3 ln 𝑥 − ln(𝑥 + 3) + 2 ln(𝑥 − 1) + 𝐶.

Example 60: Evaluate

sec 2 𝑥
∫ 𝑑𝑥.
tan2 𝑥 − 2 tan 𝑥 − 3

Solution: Let 𝑢 = tan 𝑥 ⇒ 𝑑𝑢 = sec 2 𝑥 𝑑𝑥

Thus, using sec 2 𝑥 𝑑𝑥 = 𝑑𝑢 and the Substitution Rule, we have

sec 2 𝑥 𝑑𝑢
∫ 𝑑𝑥 = ∫
tan2 𝑥 − 2 tan 𝑥 − 3 𝑢2 − 2𝑢 − 3

We factor the denominator as

𝑢2 − 2𝑢 − 3 = (𝑢 + 1)(𝑢 − 3)

Since the denominator has two distinct linear factors, the partial fraction
decomposition has the form

1 𝐴 𝐵
= +
𝑢2 − 2𝑢 − 3 𝑢 + 1 𝑢 − 3

To determine the values of 𝐴, and 𝐵, we multiply both sides of this equation by the
product of the denominators, (𝑢 + 1)(𝑢 − 3), obtaining

81
Chapter 2: TECHNIQUES OF INTEGRATION
1 = 𝐴(𝑢 − 3) + 𝐵(𝑢 + 1)

Substituting 𝑢 = −1, we get

1
1 = −4𝐴 ⇒𝐴=−
4

And letting 𝑢 = 3, we get

1
1 = 4𝐵 ⇒𝐵=
4

Thus,

𝑑𝑢 1 1 1
∫ = ∫ [ − ] 𝑑𝑢
𝑢2 − 2𝑢 − 3 4 𝑢−3 𝑢+1

1
= [ln(𝑢 − 3) − ln(𝑢 + 1)] + 𝐶
4
1 𝑢−3
= [ln ]+𝐶
4 𝑢+1

1 tan 𝑥 − 3
= [ln ] + 𝐶.
4 tan 𝑥 + 1

2.5.2 CASE II: Q(x) is a product of linear factors, some of which are repeated

Suppose the first linear factor (𝑎1 𝑥 + 𝑏1 ) is repeated 𝑟 times; that is, (𝑎1 𝑥 + 𝑏1 )𝑟
occurs in the factorization of 𝑄(𝑥). Then instead of the single term 𝐴1 ⁄(𝑎1 𝑥 + 𝑏1 ).
We would use

𝐴1 𝐴2 𝐴𝑟
+ + ⋯ + .
(𝑎1 𝑥 + 𝑏1 ) (𝑎1 𝑥 + 𝑏1 )2 (𝑎1 𝑥 + 𝑏1 )𝑟

Example 61: Evaluate

82
Chapter 2: TECHNIQUES OF INTEGRATION
3𝑥 3 − 18𝑥 2 + 29𝑥 − 4
∫ 𝑑𝑥.
(𝑥 + 1)(𝑥 − 2)3

Solution:

3𝑥 3 − 18𝑥 2 + 29𝑥 − 4 𝐴 𝐵 𝐶 𝐷
= + + +
(𝑥 + 1)(𝑥 − 2)3 (𝑥 + 1) (𝑥 − 2) (𝑥 − 2)2 (𝑥 − 2)3

To determine the values of 𝐴, 𝐵, 𝐶 and 𝐷, we multiply both sides of this equation


by the product of the denominators, (𝑥 + 1)(𝑥 − 2)3 , obtaining

3𝑥 3 − 18𝑥 2 + 29𝑥 − 4
= 𝐴(𝑥 − 2)3 + 𝐵(𝑥 + 1)(𝑥 − 2)2 + 𝐶(𝑥 + 1)(𝑥 − 2) + 𝐷(𝑥 + 1)

Here, two of the unknown constants may be determined easily. If we let 𝑥 = 2, we


obtain

6 = 3𝐷 ⇒ 𝐷 = 2.

Letting 𝑥 = −1, we get

−54 = −27𝐴 ⇒ 𝐴 = 2.

Now, choose any (but easy for calculations) other two values for 𝑥, e.g. let 𝑥 = 0,we
get

−4 = −8𝐴 + 4𝐵 − 2𝐶 + 𝐷

Since 𝐴 = 2 and 𝐷 = 2, then

−4 = −16 + 4𝐵 − 2𝐶 + 2 ⇒ 4𝐵 − 2𝐶 = 10

∴ 2𝐵 − 𝐶 = 5 (1)

Let 𝑥 = 1,we get

83
Chapter 2: TECHNIQUES OF INTEGRATION
10 = −2 + 2𝐵 − 2𝐶 + 4

∴ 2𝐵 − 2𝐶 = 8 (2)

Solving (1) and (2), we get 𝐵 = 1 and 𝐶 = −3. Then,

3𝑥 3 − 18𝑥 2 + 29𝑥 − 4 2 2 3 2
∫ 𝑑𝑥 = ∫ [ + − + ] 𝑑𝑥
(𝑥 + 1)(𝑥 − 2)3 (𝑥 + 1) (𝑥 − 2) (𝑥 − 2)2 (𝑥 − 2)3

3 1
= 2 ln(𝑥 + 1) + 2 ln(𝑥 + 2) + − + 𝐶.
𝑥 − 2 (𝑥 − 2)2

2.5.3 CASE III: Q(x) contains irreducible quadratic factors, none of which is

repeated

If 𝑄(𝑥) has the factor 𝑎𝑥 2 + 𝑏𝑥 + 𝑐, where 𝑏 2 − 4𝑎𝑐 < 0, then, in addition to the
partial fractions, the expression for 𝑃(𝑥)⁄𝑄(𝑥) will have a term of the form

𝐴𝑥 + 𝐵
𝑎𝑥 2 + 𝑏𝑥 + 𝑐

where 𝐴, and 𝐵 are constants to be determined. The term given in this form can be
integrated by completing the square and using the formula

𝑑𝑥 1 −1
𝑥
∫ = tan ( ) + 𝐶.
𝑥 2 + 𝑎2 𝑎 𝑎

Example 62: Evaluate

2𝑥 2 − 𝑥 + 4
∫ 𝑑𝑥.
𝑥 3 + 4𝑥

84
Chapter 2: TECHNIQUES OF INTEGRATION
Solution: Since 𝑥 3 + 4𝑥 = 𝑥(𝑥 2 + 4) can’t be factored further, we write

2𝑥 2 − 𝑥 + 4 𝐴 𝐵𝑥 + 𝐶
= + 2
𝑥 3 + 4𝑥 𝑥 𝑥 +4

Multiplying by 𝑥(𝑥 2 + 4), we have

2𝑥 2 − 𝑥 + 4 = 𝐴(𝑥 2 + 4) + (𝐵𝑥 + 𝐶)𝑥

= (𝐴 + 𝐵)𝑥 2 + 𝐶𝑥 + 4𝐴

Equating coefficients, we obtain

𝐴+𝐵 =2 𝐶 = −1 4𝐴 = 4

Thus, 𝐴 = 1, 𝐵 = 1 and 𝐶 = −1 and so

2𝑥 2 − 𝑥 + 4 1 𝑥−1
∫ 𝑑𝑥 = ∫ [ + ] 𝑑𝑥
𝑥 3 + 4𝑥 𝑥 𝑥2 + 4

1 𝑥 1
= ∫[ + 2 − 2 ] 𝑑𝑥
𝑥 𝑥 +4 𝑥 +4

1 1 𝑥
= ln 𝑥 + ln(𝑥 2 + 4) − tan−1 ( ) + 𝐶.
2 2 2

Example 63: Evaluate

𝑥 2 − 𝑥 − 21
∫ 3 𝑑𝑥.
2𝑥 − 𝑥 2 + 8𝑥 − 4

Solution: The denominator may be factored by grouping as follows

2𝑥 3 − 𝑥 2 + 8𝑥 − 4 = 𝑥 2 (2𝑥 − 1) + 4(2𝑥 − 1) = (𝑥 2 + 4)(2𝑥 − 1)

85
Chapter 2: TECHNIQUES OF INTEGRATION
𝑥 2 − 𝑥 − 21 𝐴𝑥 + 𝐵 𝐶
= +
2𝑥 3 − 𝑥 2 + 8𝑥 − 4 𝑥 2 + 4 2𝑥 − 1

Multiplying by (𝑥 2 + 4)(2𝑥 − 1), we have

𝑥 2 − 𝑥 − 21 = (𝐴𝑥 + 𝐵)(2𝑥 − 1) + 𝐶(𝑥 2 + 4)

= (2𝐴 + 𝐶)𝑥 2 + (2𝐵 − 𝐴)𝑥 + (4𝐶 − 𝐵)

Equating coefficients, we obtain

2𝐴 + 𝐶 = 1 2𝐵 − 𝐴 = −1 4𝐶 − 𝐵 = −21

Thus, 𝐴 = 3, 𝐵 = 1 and 𝐶 = −5 and so

𝑥 2 − 𝑥 − 21 3𝑥 + 1 5
∫ 3 𝑑𝑥 = ∫ [ − ] 𝑑𝑥
2𝑥 − 𝑥 2 + 8𝑥 − 4 𝑥 2 + 4 2𝑥 − 1

3𝑥 1 5
= ∫[ 2 + 2 − ] 𝑑𝑥
𝑥 + 4 𝑥 + 4 2𝑥 − 1

3 1 𝑥 5
= ln(𝑥 2 + 4) − tan−1 ( ) − ln(2𝑥 − 1) + 𝐶.
2 2 2 2

2.5.4 CASE IV: Q(x) contains a repeated irreducible quadratic factor

If 𝑄(𝑥) has the factor (𝑎𝑥 2 + 𝑏𝑥 + 𝑐)𝑟 , where 𝑏 2 − 4𝑎𝑐 < 0, then, in addition to
the partial fractions, the expression for 𝑃(𝑥)⁄𝑄(𝑥) will have a term of the form

𝐴1 𝑥 + 𝐵1 𝐴2 𝑥 + 𝐵2 𝐴𝑟 𝑥 + 𝐵𝑟
+ + ⋯ + .
𝑎𝑥 2 + 𝑏𝑥 + 𝑐 (𝑎𝑥 2 + 𝑏𝑥 + 𝑐)2 (𝑎𝑥 2 + 𝑏𝑥 + 𝑐)𝑟

86
Chapter 2: TECHNIQUES OF INTEGRATION
Example 64: Evaluate

5𝑥 3 − 3𝑥 2 + 7𝑥 − 3
∫ 𝑑𝑥.
(𝑥 2 + 1)2

Solution:

5𝑥 3 − 3𝑥 2 + 7𝑥 − 3 𝐴𝑥 + 𝐵 𝐶𝑥 + 𝐷
= +
(𝑥 2 + 1)2 𝑥 2 + 1 (𝑥 2 + 1)2

To determine the values of 𝐴, 𝐵, 𝐶 and 𝐷, we multiply both sides of this equation


by the product of the denominators, (𝑥 2 + 1)2 , obtaining

5𝑥 3 − 3𝑥 2 + 7𝑥 − 3 = (𝐴𝑥 + 𝐵)(𝑥 2 + 1) + 𝐶𝑥 + 𝐷

= 𝐴𝑥 3 + 𝐵𝑥 2 + (𝐴 + 𝐶)𝑥 + (𝐵 + 𝐷)

Equating coefficients, we obtain

𝐴=5 𝐵 = −3

𝐴+𝐶 =7 ⇒𝐶=2

𝐵 + 𝐷 = −3 ⇒𝐷=0

Thus,

5𝑥 3 − 3𝑥 2 + 7𝑥 − 3 5𝑥 − 3 2𝑥
∫ 𝑑𝑥 = ∫ [ + ] 𝑑𝑥
(𝑥 2 + 1)2 𝑥 2 + 1 (𝑥 2 + 1)2

5𝑥 3 2𝑥
= ∫[ 2 − 2 + 2 ] 𝑑𝑥
𝑥 + 1 𝑥 + 1 (𝑥 + 1)2

5 1
= ln(𝑥 2 + 1) − 3 tan−1 (𝑥) + 2 + 𝐶.
2 𝑥 +1

87
Chapter 2: TECHNIQUES OF INTEGRATION
Example 65: Evaluate

1 − 𝑥 + 2𝑥 2 − 𝑥 3
∫ 𝑑𝑥.
𝑥(𝑥 2 + 1)2

Solution: The form of the partial fraction decomposition is

1 − 𝑥 + 2𝑥 2 − 𝑥 3 𝐴 𝐵𝑥 + 𝐶 𝐷𝑥 + 𝐸
= + +
𝑥(𝑥 2 + 1)2 𝑥 𝑥 2 + 1 (𝑥 2 + 1)2

To determine the values of 𝐴, 𝐵, 𝐶, 𝐷 and 𝐸, we multiply both sides of this equation


by the product of the denominators,𝑥(𝑥 2 + 1)2 , obtaining

−𝑥 3 + 2𝑥 2 − 𝑥 + 1 = 𝐴𝑥(𝑥 2 + 1)2 + (𝐵𝑥 + 𝐶)𝑥(𝑥 2 + 1) + (𝐷𝑥 + 𝐸)𝑥

= 𝐴(𝑥 4 + 2𝑥 2 + 1) + 𝐵(𝑥 4 + 𝑥 2 ) + 𝐶(𝑥 3 + 𝑥) + 𝐷𝑥 2 + 𝐸𝑥

= (𝐴 + 𝐵)𝑥 4 + 𝐶𝑥 3 + (2𝐴 + 𝐵 + 𝐷)𝑥 2 + (𝐶 + 𝐸)𝑥 + 𝐴

Equating coefficients, we obtain

𝐴+𝐵 =0 𝐶 = −1

2𝐴 + 𝐵 + 𝐷 = 2

𝐶 + 𝐸 = −1 𝐴=1

Which have the solutions: 𝐴 = 1, 𝐵 = −1, 𝐶 = −1, 𝐷 = 1 and 𝐸 = 0. Thus,

1 − 𝑥 + 2𝑥 2 − 𝑥 3 1 𝑥+1 𝑥
∫ 𝑑𝑥 = ∫ [ − + ] 𝑑𝑥
𝑥(𝑥 2 + 1)2 𝑥 𝑥 2 + 1 (𝑥 2 + 1)2

1 𝑥 1 𝑥
= ∫[ − 2 − 2 + 2 ] 𝑑𝑥
𝑥 𝑥 + 1 𝑥 + 1 (𝑥 + 1)2

1 1
= ln 𝑥 − ln(𝑥 2 + 1) − tan−1 (𝑥) + + 𝐶.
2 2(𝑥 2 + 1)

88
Chapter 2: TECHNIQUES OF INTEGRATION
2.6 Integrals Involving Quadratic Expressions

Many integrals containing a square root or negative power of a quadratic polynomial


𝑎𝑥² + 𝑏𝑥 + 𝑐 can be simplified by the process of completing the square. The key to
solving these integrals is recognizing the pattern and choosing the appropriate
method. Let's explore the different types and their solution methods.

Types of Quadratic Integrals

𝟏. ∫(𝑎𝑥² + 𝑏𝑥 + 𝑐)𝑑𝑥

𝑑𝑥
𝟐. ∫
𝑎𝑥² + 𝑏𝑥 + 𝑐

𝑑𝑥
𝟑. ∫
√𝑎𝑥² + 𝑏𝑥 + 𝑐

𝐴𝑥 + 𝐵
𝟒. ∫ 𝑑𝑥
𝑎𝑥² + 𝑏𝑥 + 𝑐

𝐴𝑥 + 𝐵
𝟓. ∫ 𝑑𝑥
√𝑎𝑥² + 𝑏𝑥 + 𝑐

To illustrate this process of completing the square, consider the polynomial


(𝑎𝑥 2 + 𝑏𝑥 + 𝑐)

Step 1: Take 𝑎 as common factor

𝑏 𝑐
𝑎 [(𝑥² + 𝑥 + )]
𝑎 𝑎

Step 2: Take the coefficient of 𝑥, divide it by 2, and square it to get

𝑏 𝑏 𝑏 2 𝑏2
⇒ , ( ) = 2
𝑎 2𝑎 2𝑎 4𝑎

89
Chapter 2: TECHNIQUES OF INTEGRATION
Step 3: Add and subtract this number and factor the result to get

𝑏 𝑐
𝑎𝑥 2 + 𝑏𝑥 + 𝑐 = 𝑎 [(𝑥² + 𝑥 + )]
𝑎 𝑎

2
𝑏 𝑏2 𝑐 𝑏2
= 𝑎 [𝑥 + 𝑥 + 2 + − 2 ]
𝑎 4𝑎 𝑎 4𝑎

𝑏 2 𝑐 𝑏2
= 𝑎 [(𝑥 + ) + − 2 ]
2𝑎 𝑎 4𝑎

𝑏 2 𝑏2
= 𝑎 (𝑥 + ) + (𝑐 − ).
2𝑎 4𝑎

For example: complete the square for: 𝑥 2 − 6𝑥 + 13

𝑥 2 − 6𝑥 + 13 = 𝑥 2 − 6𝑥 + 9 + 13 − 9
= (𝑥 − 3)2 − 6𝑥 + 22 .
Another Example: complete the square for: 2𝑥 2 − 3𝑥 + 2

3
2𝑥 2 − 3𝑥 + 2 = 2 [𝑥 2 − 𝑥 + 1]
2
3 9 9
= 2 [𝑥 2 − 𝑥 + +1− ]
2 16 16
3 2 7
= 2 [(𝑥 − ) + ]
4 16
2
3 2 √7
= 2 [(𝑥 − ) + ( ) ].
4 4

Solution Methods

Method 1: Direct Integration For simple quadratic expressions:

90
Chapter 2: TECHNIQUES OF INTEGRATION
𝑎𝑥 3 𝑏𝑥 2
∫(𝑎𝑥² + 𝑏𝑥 + 𝑐)𝑑𝑥 = + + 𝑐𝑥 + 𝐶.
3 2

Method 2: Completing the Square

Type of Quadratic Integrals After Completing the Square


𝑑𝑥 𝑑𝑥 1 𝑥
∫ ∫ = tan−1 + 𝐶
𝑎𝑥² + 𝑏𝑥 + 𝑐 𝑎² + 𝑥² 𝑎 𝑎

𝑑𝑥 1 −1
𝑥
∫ = tanh +𝐶
𝑎2 − 𝑥² 𝑎 𝑎

𝑑𝑥 1 𝑥
∫ = − coth−1
𝑥² + 𝑎² 𝑎 𝑎

𝑑𝑥 𝑑𝑥 𝑥
∫ ∫ = sinh−1 +𝐶
√𝑎𝑥² + 𝑏𝑥 + 𝑐 √𝑎² + 𝑥² 𝑎

𝑑𝑥 𝑥
∫ = sin−1 +𝐶
√𝑎2 − 𝑥² 𝑎

𝑑𝑥 𝑥
∫ = cosh−1
√𝑥² + 𝑎² 𝑎

𝐴𝑥 + 𝐵 𝑓′(𝑥)
∫ 𝑑𝑥 ∫ 𝑑𝑥 = ln 𝑓(𝑥) + 𝐶
𝑎𝑥² + 𝑏𝑥 + 𝑐 𝑓(𝑥)

𝐴𝑥 + 𝐵 𝑓′(𝑥)
∫ 𝑑𝑥 ∫ 𝑑𝑥 = 2√𝑓(𝑥) + 𝐶
√𝑎𝑥² + 𝑏𝑥 + 𝑐 √𝑓(𝑥)

91
Chapter 2: TECHNIQUES OF INTEGRATION
Example 66: Evaluate

𝑑𝑥
∫ .
√𝑥 2 + 6𝑥 + 13

Solution: Complete the square for: 𝑥 2 + 6𝑥 + 13

𝑥 2 + 6𝑥 + 13 = 𝑥 2 + 6𝑥 + 9 + 13 − 9
= (𝑥 + 3)2 + 22
Then,

𝑑𝑥 𝑑𝑥
∫ =∫
√𝑥 2 + 6𝑥 + 13 √(𝑥 + 3)2 + 22

Using substitution role, Let 𝑢 = 𝑥 + 3, then 𝑑𝑢 = 𝑑𝑥

𝑑𝑥 𝑑𝑢
∫ =∫
√𝑥 2 + 6𝑥 + 13 √𝑢2 + 22
𝑢
= sinh−1 + 𝐶
2
𝑥+3
= sinh−1 ( ) + 𝐶.
2

Example 67: Evaluate

𝑑𝑥
∫ .
2𝑥 2 − 3𝑥 + 2

Solution: Complete the square for: 2𝑥 2 − 3𝑥 + 2

3 9 9
2𝑥 2 − 3𝑥 + 2 = 2 [𝑥 2 − 𝑥 + +1− ]
2 16 16

92
Chapter 2: TECHNIQUES OF INTEGRATION
2
3 2 √7
= 2 [(𝑥 − ) + ( ) ]
4 4

Then,

𝑑𝑥 1 𝑑𝑥
∫ = ∫ 2
2𝑥 2 − 3𝑥 + 2 2 3 2 √7
(𝑥 − 4) + ( 4 )

3
Using substitution role, Let 𝑢 = 𝑥 − , then 𝑑𝑢 = 𝑑𝑥
4

𝑑𝑥 1 𝑑𝑢
∫ = ∫ 2
2𝑥 2 − 3𝑥 + 2 2 √7
𝑢2 + ( )
4

1 4 𝑢
= [ tan−1 ]+𝐶
2 √7 √7
4
3
2 𝑥−
= tan−1 ( 4 ) + 𝐶.
√7 √7
4
2 4𝑥 − 3
= tan−1 ( ) + 𝐶.
√7 √7

Example 68: Evaluate

𝑑𝑥
∫ .
√5 − 2𝑥 − 𝑥 2

Solution: Complete the square for: 5 − 2𝑥 − 𝑥 2

5 − 2𝑥 − 𝑥 2 = −[𝑥 2 + 2𝑥 − 5]

93
Chapter 2: TECHNIQUES OF INTEGRATION
= −[𝑥 2 + 2𝑥 + 1 − 5 − 1]

= −[(𝑥 + 1)2 − 6]
2
= (√6) − (𝑥 + 1)2

Then,

𝑑𝑥 𝑑𝑥
∫ =∫
√5 − 2𝑥 − 𝑥 2 2
√(√6) − (𝑥 + 1)2

Using substitution role, Let 𝑢 = 𝑥 + 1, then 𝑑𝑢 = 𝑑𝑥

𝑑𝑥 𝑑𝑢
∫ =∫
√5 − 2𝑥 − 𝑥 2 2
√(√6) − 𝑢2

𝑢
= sin−1 +𝐶
√6
𝑥+1
= sin−1 ( ) + 𝐶.
√6

Example 69: Evaluate

3𝑥 − 1
∫ 𝑑𝑥.
𝑥 2 + 10𝑥 + 28

Solution: Complete the square for: 𝑥 2 + 10𝑥 + 28

𝑥 2 + 10𝑥 + 28 = 2[𝑥 2 + 10𝑥 + 25 + 28 − 25]


= (𝑥 + 5)2 + 3
Then,

94
Chapter 2: TECHNIQUES OF INTEGRATION
3𝑥 − 1 3𝑥 − 1
∫ 2 𝑑𝑥 = ∫ 𝑑𝑥
𝑥 + 10𝑥 + 28 (𝑥 + 5)2 + 3

Using substitution role, Let 𝑢 = 𝑥 + 5, then 𝑥 = 𝑢 − 5 and 𝑑𝑢 = 𝑑𝑥

3𝑥 − 1 3(𝑢 − 5) − 1
∫ 𝑑𝑥 = ∫ 𝑑𝑢
𝑥 2 + 10𝑥 + 28 𝑢2 + 3
3𝑢 − 16
=∫ 𝑑𝑢
𝑢2 + 3
𝑢 1
= 3∫ 𝑑𝑢 − 16 ∫ 𝑑𝑢
𝑢2 + 3 𝑢2 + 3
3 16 𝑢
= ln(𝑢2 + 3) − tan−1 ( ) + 𝐶
2 √3 √3
3 16 𝑥+5
= ln((𝑥 + 5)2 + 3) − tan−1 ( )+𝐶
2 √3 √3
3 16 𝑥+5
= ln(𝑥 2 + 10𝑥 + 28) − tan−1 ( ) + 𝐶.
2 √3 √3

Example 70: Evaluate

5𝑥 + 7
∫ 𝑑𝑥.
√𝑥 2 + 6𝑥 + 13

Solution: Complete the square for: 𝑥 2 + 6𝑥 + 13

𝑥 2 + 6𝑥 + 13 = 𝑥 2 + 6𝑥 + 9 + 13 − 9
= (𝑥 + 3)2 + 22
Then,

5𝑥 + 7 5𝑥 + 7
∫ 𝑑𝑥 = ∫ 𝑑𝑥
√𝑥 2 + 6𝑥 + 13 √(𝑥 + 3)2 + 22

95
Chapter 2: TECHNIQUES OF INTEGRATION
Using substitution role, Let 𝑢 = 𝑥 + 3, then 𝑥 = 𝑢 − 3 and 𝑑𝑢 = 𝑑𝑥

5𝑥 + 7 5(𝑢 − 3) + 7
∫ 𝑑𝑥 = ∫ 𝑑𝑥
√𝑥 2 + 6𝑥 + 13 √𝑢2 + 22
5𝑢 − 8
=∫ 𝑑𝑢
√𝑢2 + 22
𝑢 1
= 5∫ 𝑑𝑢 − 8 ∫ 𝑑𝑢
√𝑢2 + 22 √𝑢2 + 22
5 𝑢
= 2√𝑢2 + 22 − 8 sinh−1 ( ) + 𝐶
2 2
𝑥+3
= 5√𝑥 2 + 6𝑥 + 13 − 8 sinh−1 ( ) + 𝐶.
2

Example 71: Evaluate

2𝑥 + 3
∫√ 𝑑𝑥.
3𝑥 + 2

Solution:

2𝑥 + 3 √2𝑥 + 3 2𝑥 + 3
√ × =
3𝑥 + 2 √2𝑥 + 3 √6𝑥 2 + 13𝑥 + 6

Then,

2𝑥 + 3 2𝑥 + 3
∫√ 𝑑𝑥 = ∫ 𝑑𝑥
3𝑥 + 2 √6𝑥 2 + 13𝑥 + 6

Complete the square for: 6𝑥 2 + 13𝑥 + 6

13 169 169
𝑥 2 + 6𝑥 + 13 = 6 [𝑥 2 + 𝑥+ +1− ]
6 144 144

96
Chapter 2: TECHNIQUES OF INTEGRATION
13 2 5 2
= 6 [(𝑥 + ) − ( ) ]
12 12
Then,

2𝑥 + 3 1 2𝑥 + 3
∫√ 𝑑𝑥 = ∫ 𝑑𝑥
3𝑥 + 2 √6 2 2
√(𝑥 + 13) − ( 5 )
12 12
13 13
Using substitution role, Let 𝑢 = 𝑥 + , then 𝑥 = 𝑢 − and 𝑑𝑢 = 𝑑𝑥
12 12

13
2𝑥 + 3 1 2 (𝑢 − 12) + 3
∫√ 𝑑𝑥 = ∫ 𝑑𝑥
3𝑥 + 2 √6 5 2
√𝑢 2 − ( )
12
5
1 2𝑢 +
= ∫ 6 𝑑𝑢
√6 2
√𝑢 2 − ( 5 )
12

1 2𝑢 5 1
= ∫ 𝑑𝑢 + ∫ 𝑑𝑢
√6 2 6 2
√𝑢 2 − ( 5 ) √𝑢 2 − ( 5 )
[ 12 12 ]

2
1 5 5 𝑢
= [√𝑢2 − ( ) + cosh−1 ( )] + 𝐶
√6 12 6 5
12
13
1 13 2 5 2 5 𝑥+
12 )] + 𝐶
= [√(𝑥 + ) − ( ) + cosh−1 (
√6 12 12 6 5
12

1 13 5 12𝑥 + 13
= [√ 𝑥 2 + 𝑥 + 1 + cosh−1 ( )] + 𝐶.
√6 6 6 5

97
Chapter 2: TECHNIQUES OF INTEGRATION
Example 72: Evaluate

1+𝑥
∫√ 𝑑𝑥.
1−𝑥

Solution:

1 + 𝑥 √1 + 𝑥 𝑥
√ × =
1 − 𝑥 √1 + 𝑥 √1 − 𝑥 2

Then,

1+𝑥 1+𝑥
∫√ 𝑑𝑥 = ∫ 𝑑𝑥
1−𝑥 √1 − 𝑥 2

1 𝑥
=∫ 𝑑𝑥 + ∫ 𝑑𝑥
√1 − 𝑥 2 √1 − 𝑥 2

1 −2𝑥
=∫ 𝑑𝑥 − ∫ 𝑑𝑥
√1 − 𝑥 2 2√1 − 𝑥 2

= sinh−1 𝑥 − √1 + 𝑥 + 𝐶.

98
Chapter 2: TECHNIQUES OF INTEGRATION
Exercises (2)

1. Evaluate the integrals

∫ √9 − 𝑥 2 𝑑𝑥 ∫ √1 − 4𝑥 2 𝑑𝑥

∫ √𝑥 2 − 25 𝑑𝑥 ∫ √2𝑥 2 − 3 𝑑𝑥

∫ √𝑥 2 + 5 𝑑𝑥 ∫ √25𝑥 2 + 1 𝑑𝑥
𝑑𝑥 𝑑𝑥
∫ ∫
√2𝑥 2 − 7 √5𝑥 2 + 9
𝑑𝑥 𝑑𝑥
∫ ∫
√9 − 2𝑥 2 𝑥 2 √1 − 𝑥 2
𝑥2 √𝑥 2 + 1
∫ 𝑑𝑥 ∫ 𝑑𝑥
√𝑥 2 + 1 𝑥2
𝑑𝑥 𝑑𝑥
∫ ∫
𝑥 2 √𝑥 2 − 2 𝑥 2 √𝑥 2 + 𝑎2
1
∫ sin √𝑥 𝑑𝑥 ∫ √𝑥 ln 𝑥 𝑑𝑥
√𝑥
∫ 𝑥 cos 𝑥 𝑑𝑥 ∫ 𝑥 3 sin 𝑥 𝑑𝑥

∫ 𝑥 3 cos 𝑥 𝑑𝑥 ∫ 𝑒 𝑥 sin 𝑥 𝑑𝑥

∫ 𝑒 𝑥 cos 𝑥 𝑑𝑥 ∫ cot −1 𝑥 𝑑𝑥

∫ 𝑒 2𝑥 sin 5𝑥 𝑑𝑥 ∫ csc −1 𝑥 𝑑𝑥

∫ sec −1 𝑥 𝑑𝑥 ∫ 𝑥 sec −1 𝑥 𝑑𝑥

∫ 𝑥 cos −1 𝑥 𝑑𝑥 ∫ 𝑥 cos 2 𝑥 𝑑𝑥

∫ 𝑥 tan−1 𝑥 𝑑𝑥 ∫ 𝑥 tan2 𝑥 𝑑𝑥
𝑥
∫ 𝑥 sin2 𝑥 𝑑𝑥 ∫ 𝑑𝑥
√𝑥 2 − 𝑥 + 1

99
Chapter 2: TECHNIQUES OF INTEGRATION
∫ 𝑥 sec 2 𝑥 𝑑𝑥 ∫ cos 3 𝑥 𝑑𝑥

∫ sec 5 𝑥 𝑑𝑥 ∫ sinh−1 𝑥 𝑑𝑥

∫ cosh−1 𝑥 𝑑𝑥 ∫ tanh−1 𝑥 𝑑𝑥

∫ sech−1 𝑥 𝑑𝑥 ∫ 𝑥 sinh−1 𝑥 𝑑𝑥

∫ 𝑥 cosh−1 𝑥 𝑑𝑥 ∫ 𝑥 tanh−1 𝑥 𝑑𝑥
sin−1 √𝑥 ∫ 𝑥 ln(1 + 𝑥 3 ) 𝑑𝑥
∫ 𝑑𝑥
√1 − 𝑥
𝑥𝑒 𝑥 ln3 (𝑥)
∫ 𝑑𝑥 ∫ 𝑑𝑥
√1 + 𝑒 𝑥 𝑥2
𝑥−2
∫ 𝑥 𝑛 ln 𝑥 𝑑𝑥 ∫ 2 𝑑𝑥
𝑥 + 4𝑥 + 5
𝑥−2 𝑑𝑥
∫ 𝑑𝑥 ∫
√𝑥 2
+ 4𝑥 + 5 √9 + 8𝑥 − 𝑥 2
𝑑𝑥 𝑑𝑥
∫ 2 𝑑𝑥 ∫
2𝑥 − 5𝑥 + 3 √3𝑥 2 + 6𝑥 + 4
𝑑𝑥 𝑥 𝑑𝑥
∫ ∫
4 + √𝑥 √𝑥 + 3
𝑥 2 + 2𝑥 + 3 𝑥+5
∫ 𝑑𝑥 ∫ 3 𝑑𝑥
𝑥3 − 𝑥 𝑥 − 3𝑥 + 2
3𝑥 2 + 𝑥 − 2 1
∫ 3 𝑑𝑥 ∫ 3 𝑑𝑥
𝑥 − 𝑥2 + 𝑥 − 1 𝑥 −1
𝑥3 + 1 𝑥
∫ 3 𝑑𝑥 ∫ 𝑑𝑥
𝑥 −1 (𝑥 − 2)(𝑥 3 − 8)
𝑥3 2
∫ 4 𝑑𝑥 ∫ 2 𝑑𝑥
𝑥 − 𝑥2 + 1 𝑥 + 6𝑥 + 2
1 𝑑𝑥
∫ 𝑑𝑥 ∫
2 + 3𝑥 − 𝑥 2 √𝑥 2 + 6𝑥 + 10
𝑑𝑥 6𝑥 + 7
∫ ∫ 2 𝑑𝑥
√2𝑥 2 + 3𝑥 − 2 𝑥 −𝑥−1
5𝑥 + 1 𝑥+1
∫ 2 𝑑𝑥 ∫ 𝑑𝑥
2𝑥 + 4𝑥 + 3 √2𝑥 2 + 𝑥 − 3

100
Chapter 2: TECHNIQUES OF INTEGRATION
Find the reduction formula for the integration:

∫ cot 𝑛 𝑥 𝑑𝑥 ∫ csc 𝑛 𝑥 𝑑𝑥

∫ tanh𝑛 𝑥 𝑑𝑥 ∫ coth𝑛 𝑥 𝑑𝑥

∫ 𝑥 7 cos 𝑥 𝑑𝑥 ∫ 𝑥 6 sin 𝑥 𝑑𝑥

∫ tan5 𝑥 𝑑𝑥 ∫ cos 6 𝑥 𝑑𝑥

∫ cot 4 𝑥 𝑑𝑥 ∫ csc 3 𝑥 𝑑𝑥

∫ 𝑒 2𝑥 sin 3𝑥 𝑑𝑥 ∫ 𝑒 3𝑥 cos 2𝑥 𝑑𝑥

∫(sin−1 𝑥)5 𝑑𝑥 ∫ 𝑥 5 sin 𝑥 𝑑𝑥

101
Chapter 2: TECHNIQUES OF INTEGRATION
Notes:

102
Chapter 2: TECHNIQUES OF INTEGRATION

103
Chapter 2: TECHNIQUES OF INTEGRATION

104
Chapter 2: TECHNIQUES OF INTEGRATION

105
Chapter 3: DEFINITE INTEGRALS

Chapter 3. DEFINITE INTEGRALS

If 𝑓 is a function defined for 𝑎 ≤ 𝑥 ≤ 𝑏, we divide the interval [𝑎, 𝑏] into 𝑛


subintervals of equal width ∆𝑥 = (𝑏 − 𝑎)⁄𝑛. We let 𝑥0 (= 𝑎), 𝑥1 , 𝑥2 , … , 𝑥𝑛 (= 𝑏) be
the endpoints of these subintervals and we let 𝑥1 ∗ , 𝑥2 ∗ , … , 𝑥𝑛 ∗ be any sample points
in these subintervals, so 𝑥𝑖 ∗ lies in the 𝑖th subinterval [𝑥𝑖−1 , 𝑥𝑖 ]. Then the definite
integral of 𝒇 from 𝒂 to 𝒃 is

𝑏 𝑛

∫ 𝑓(𝑥)𝑑𝑥 = lim ∑ 𝑓(𝑥𝑖 ∗ ) ∆𝑥


𝑎 𝑛→∞
𝑖=1

provided that this limit exists. If it does exist, we say that 𝑓 is integrable on [𝑎, 𝑏].

The symbol ∫ was introduced by Leibniz and is called an integral sign. It is an


elongated 𝑆 and was chosen because an integral is a limit of sums.
𝑏
In the notation ∫𝑎 𝑓(𝑥)𝑑𝑥, 𝑓(𝑥) is called the integrand and 𝑎 and 𝑏 are called the
limits of integration; 𝑎 is the lower limit and 𝑏 is the upper limit. For now, the
𝑏
symbol 𝑑𝑥 has no meaning by itself; ∫𝑎 𝑓(𝑥)𝑑𝑥 is all one symbol. The 𝑑𝑥 simply
indicates that the independent variable is 𝑥. The procedure of calculating an integral
is called integration.

106
Chapter 3: DEFINITE INTEGRALS
The value of the definite integral of a function over any particular interval depends
on the function, not on the letter we choose to represent its independent variable. If
we decide to use 𝑡 or 𝑢 instead of 𝑥, we simply write the integral as
𝑏 𝑏 𝑏
∫ 𝑓(𝑡)𝑑𝑡 or ∫ 𝑓(𝑢)𝑑𝑢 instead of ∫ 𝑓(𝑥)𝑑𝑥
𝑎 𝑎 𝑎

No matter how we write the integral, it is still the same number that is defined as a
limit of Riemann sums. Since it does not matter what letter we use, the variable of
integration is called a dummy variable representing the real numbers in the closed
interval [𝑎, 𝑏].

3.1 The Fundamental Theorem of Calculus

This theorem describes how to evaluate definite integrals without having to calculate
limits of Riemann sums. Instead, we find and evaluate an antiderivative at the upper
and lower limits of integration.

If 𝑓 is continuous over [𝑎, 𝑏] and 𝐹 is any antiderivative of 𝑓on [𝑎, 𝑏], then
𝑏
∫ 𝑓(𝑥)𝑑𝑥 = 𝐹(𝑏) − 𝐹(𝑎).
𝑎

Example 1: Evaluate

3
∫ 𝑥 2 𝑑𝑥
1

107
Chapter 3: DEFINITE INTEGRALS
Solution:

3 3
𝑥3 1
∫ 𝑥 𝑑𝑥 = [ ] = [33 − 13 ]
2
1 3 1 3

1 26
= [27 − 1] = .
3 3

Example 2: Evaluate

𝜋
∫ cos 𝑥 𝑑𝑥
0

Solution:

𝜋
∫ cos 𝑥 𝑑𝑥 = [sin 𝑥]𝜋0
0

= sin 𝜋 − sin 0 = 0 − 0 = 0.

Example 3: Evaluate

4
3 4
∫ ( √𝑥 − 2 ) 𝑑𝑥.
1 2 𝑥

Solution:

4
3 4 3 3 3 44
∫ ( √𝑥 − 2 ) 𝑑𝑥 = [ ∙ 𝑥 2 + ]
1 2 𝑥 2 2 𝑥1

108
Chapter 3: DEFINITE INTEGRALS
3 44 3 4 3 44
= [𝑥 2 + ] = [4 + ] − [1 + ]
2 2
𝑥1 4 11

= [8 + 1] − 5 = 4.

Example 4: Evaluate

1
𝑑𝑥
∫ .
0 1+𝑥

Solution:

1
𝑑𝑥
∫ = [ln(1 + 𝑥)]10
0 1+𝑥

= ln(2) − ln(1) = ln 2.

Example 5: Evaluate

1
𝑑𝑥
∫ .
0 1 + 𝑥2

Solution:

1
𝑑𝑥
∫ 2
= [tan−1 𝑥]10
0 1+𝑥

𝜋 𝜋
= tan−1 1 − tan−1 0 = −0= .
4 4

Example 6: Evaluate

109
Chapter 3: DEFINITE INTEGRALS
4
∫ √16 − 𝑥 2 𝑑𝑥
0

Solution: Let 𝑥 = 4 sin 𝜃, then 𝑑𝑥 = 4 cos 𝜃 𝑑𝜃

𝜋
At 𝑥=0 ⇒𝜃=0 and at 𝑥 = 4 ⇒𝜃=
2
𝜋
4
2
∫ √16 − 𝑥 2 𝑑𝑥 = ∫ √16 − 16 sin2 𝜃 4 cos 𝜃 𝑑𝜃
0 0

𝜋
2
= 16 ∫ √1 − sin2 𝜃 cos 𝜃 𝑑𝜃
0

𝜋
2
= 16 ∫ cos 2 𝜃 𝑑𝜃
0

1
cos 2 𝜃 = (1 + cos 2𝜃), then
2

𝜋
4
2
∫ √16 − 𝑥 2 𝑑𝑥 = 8 ∫ (1 + cos 2𝜃) 𝑑𝜃
0 0

𝜋
sin 2𝜃 2 𝜋
= 8 [𝜃 + ] = 8 [ − 0] = 4𝜋.
2 0 2

Example 7: Evaluate

1
∫ tan−1 𝑥 𝑑𝑥.
0

110
Chapter 3: DEFINITE INTEGRALS
Solution: Let

𝑢 = tan−1 𝑥 𝑑𝑣 = 𝑑𝑥

1 𝑣=𝑥
𝑑𝑢 = 𝑑𝑥
1 + 𝑥2
Then,
1 1
𝑥
∫ tan −1
𝑥 𝑑𝑥 = [𝑥 tan −1
𝑥]10 −∫ 𝑑𝑥
0 0 1 + 𝑥2

1
−1
1 2)
= [𝑥 tan 𝑥 − ln(1 + 𝑥 ]
2 0

1
= [tan−1 1 − (ln 2 − ln 1)]
2

𝜋 1
= − ln 2 = 0.438825
4 2

3.2 Properties of Definite Integrals

When 𝑓 and 𝑔 are integrable over the interval[𝑎, 𝑏], the definite integral satisfies the
following rules

1. Order of integration:

𝒃 𝒂
∫ 𝒇(𝒙) 𝒅𝒙 = − ∫ 𝒇(𝒙) 𝒅𝒙
𝒂 𝒃

Proof:

𝑏
∫ 𝑓(𝑥) 𝑑𝑥 = [𝐹(𝑥)]𝑏𝑎 = 𝐹(𝑏) − 𝐹(𝑎)
𝑎

111
Chapter 3: DEFINITE INTEGRALS
= −[𝐹(𝑎) − 𝐹(𝑏)]
𝑎
= − ∫ 𝑓(𝑥) 𝑑𝑥. ∎
𝑏

2. Zero Width Interval:

𝒂
∫ 𝒇(𝒙) 𝒅𝒙 = 𝟎
𝒂

Proof:

𝑎
∫ 𝑓(𝑥) 𝑑𝑥 = [𝐹(𝑥)]𝑎𝑎 = 𝐹(𝑎) − 𝐹(𝑎)
𝑎

= 0. ∎

3. Constant Multiple

𝒃 𝒃
∫ 𝒌𝒇(𝒙) 𝒅𝒙 = 𝒌 ∫ 𝒇(𝒙) 𝒅𝒙.
𝒂 𝒂

112
Chapter 3: DEFINITE INTEGRALS
4. Sum and Difference

𝒃 𝒃 𝒃
∫ (𝒇(𝒙) ± 𝒈(𝒙)) 𝒅𝒙 = ∫ 𝒇(𝒙) 𝒅𝒙 ± ∫ 𝒈(𝒙) 𝒅𝒙
𝒂 𝒂 𝒂

5. Additivity

𝒃 𝒄 𝒃
∫ 𝒇(𝒙) 𝒅𝒙 = ∫ 𝒇(𝒙) 𝒅𝒙 + ∫ 𝒇(𝒙) 𝒅𝒙
𝒂 𝒂 𝒄

Proof:

𝑐 𝑏
𝑅. 𝐻. 𝑆 = ∫ 𝑓(𝑥) 𝑑𝑥 + ∫ 𝑓(𝑥) 𝑑𝑥
𝑎 𝑐

= [𝐹(𝑥)]𝑐𝑎 + [𝐹(𝑥)]𝑏𝑐

= 𝐹(𝑐) − 𝐹(𝑎) + 𝐹(𝑏) − 𝐹(𝑐)

= 𝐹(𝑏) − 𝐹(𝑎)
𝑏
= ∫ 𝑓(𝑥) 𝑑𝑥. ∎
𝑎

𝒃 𝒃
𝟔. ∫ 𝒇(𝒙) 𝒅𝒙 = ∫ 𝒇(𝒂 + 𝒃 − 𝒙) 𝒅𝒙
𝒂 𝒂
𝒂 𝒂
𝟕. ∫ 𝒇(𝒙) 𝒅𝒙 = ∫ 𝒇(𝒂 − 𝒙) 𝒅𝒙
𝟎 𝟎
𝒂
𝟐𝒂
𝟐 ∫ 𝒇(𝒙) 𝒅𝒙, if 𝑓(2𝑎 − 𝑥) = 𝑓(𝑥)
𝟖. ∫ 𝒇(𝒙) 𝒅𝒙 = { 𝟎
𝟎
𝟎, if 𝑓(2𝑎 − 𝑥) = −𝑓(𝑥)

113
Chapter 3: DEFINITE INTEGRALS
𝒂 𝒂
𝟗. ∫ 𝒇(𝒙) 𝒅𝒙 = 𝟐 ∫ 𝒇(𝒙) 𝒅𝒙
−𝒂 𝟎

If 𝑓(𝑥) is even function, i.e., 𝑓(−𝑥) = 𝑓(𝑥).


𝒂
𝟏𝟎. ∫ 𝒇(𝒙) 𝒅𝒙 = 𝟎
−𝒂

If 𝑓(𝑥) is odd function, i.e., 𝑓(−𝑥) = −𝑓(𝑥).

Some important identities

tan 𝑥 ± tan 𝑦
∗ tan(𝑥 ± 𝑦) =
1 ∓ tan 𝑥 tan 𝑦

𝐴
∗ ln ( ) = ln 𝐴 − ln 𝐵
𝐵

∗ sin(−𝑥) = − sin 𝑥 , ∗ cos(−𝑥) = cos 𝑥


𝜋 𝜋
∗ sin ( − 𝑥) = cos 𝑥 , ∗ cos ( − 𝑥) = sin 𝑥
2 2

Example 8: Evaluate
𝜋
2 sin 𝑥
∫ 𝑑𝑥 .
0 sin 𝑥 + cos 𝑥

Solution:
𝜋 𝜋 𝜋
2 sin 𝑥 2 sin ( − 𝑥)
I=∫ 𝑑𝑥 = ∫ 2 𝑑𝑥
𝜋 𝜋
0 sin 𝑥 + cos 𝑥 0 sin ( − 𝑥) + cos ( − 𝑥)
2 2
𝜋 𝜋
2 cos 𝑥 2 cos 𝑥 + sin 𝑥 − sin 𝑥
=∫ 𝑑𝑥 = ∫ 𝑑𝑥
0 cos 𝑥 + sin 𝑥 0 cos 𝑥 + sin 𝑥

114
Chapter 3: DEFINITE INTEGRALS
𝜋
2 sin 𝑥
= ∫ (1 − ) 𝑑𝑥
0 cos 𝑥 + sin 𝑥
𝜋 𝜋
2 2 sin 𝑥
= ∫ 𝑑𝑥 − ∫ 𝑑𝑥
0 0 cos 𝑥 + sin 𝑥
𝜋
2
I = ∫ 𝑑𝑥 − I
0
𝜋
𝜋
2I = [𝑥]02 =
2
𝜋
2 sin 𝑥
∴I=∫ 𝑑𝑥 = 𝜋.
0 sin 𝑥 + cos 𝑥

Example 9: Evaluate
𝜋
4
∫ ln(1 + tan 𝑥) 𝑑𝑥 .
0

Solution:
𝜋 𝜋
4 𝜋 4
I = ∫ ln(1 + tan 𝑥) 𝑑𝑥 = ∫ ln (1 + tan ( − 𝑥)) 𝑑𝑥
0 0 4
𝜋 𝜋
4 tan − tan 𝑥
= ∫ ln (1 + 4 ) 𝑑𝑥
𝜋
0 1 + tan tan 𝑥
4
∵ tan(𝜋⁄4) = 1, then
𝜋
4 1 − tan 𝑥
= ∫ ln (1 + ) 𝑑𝑥
0 1 + tan 𝑥
𝜋
4 1 + tan 𝑥 + 1 − tan 𝑥
= ∫ ln ( ) 𝑑𝑥
0 1 + tan 𝑥

115
Chapter 3: DEFINITE INTEGRALS
𝜋
4 2
= ∫ ln ( ) 𝑑𝑥
0 1 + tan 𝑥
𝜋
4
= ∫ [ln(2) − ln(1 + tan 𝑥)]𝑑𝑥
0
𝜋 𝜋
4 4
= ∫ ln(2) 𝑑𝑥 − ∫ ln(1 + tan 𝑥) 𝑑𝑥
0 0
𝜋
4
I = ln(2) ∫ 𝑑𝑥 − I
0
𝜋
I= ln(2) [𝑥]04 −I
𝜋
2I = ln(2)
4
𝜋
4 𝜋
∴ I = ∫ ln(1 + tan 𝑥) 𝑑𝑥 = ln(2).
0 8

Example 10: Evaluate


𝜋
2 sin𝑛 𝑥
∫ 𝑛 𝑥 + cos 𝑛 𝑥
𝑑𝑥 .
0 sin

Solution:
𝜋 𝜋 𝜋 𝑛
sin 𝑥
2 𝑛 [sin
2 ( − 𝑥)]
I=∫ 𝑑𝑥 = ∫ 2
𝑛 𝑛 𝜋 𝑛 𝜋 𝑛 𝑑𝑥
0 sin 𝑥 + cos 𝑥 0 [sin ( − 𝑥)] + [cos ( − 𝑥)]
2 2
𝜋
2 cos 𝑛 𝑥
=∫ 𝑛 𝑛
𝑑𝑥
0 cos 𝑥 + sin 𝑥
𝜋 𝜋
sin𝑛 𝑥2 2 cos 𝑛 𝑥
∵ I=∫ 𝑛 𝑛
𝑑𝑥 = ∫ 𝑛 𝑛
𝑑𝑥
0 sin 𝑥 + cos 𝑥 0 cos 𝑥 + sin 𝑥

116
Chapter 3: DEFINITE INTEGRALS
Then,
𝜋 𝜋
sin𝑛 𝑥
2 2 cos 𝑛 𝑥
2I = ∫ 𝑛 𝑛
𝑑𝑥 + ∫ 𝑛 𝑛
𝑑𝑥
0 sin 𝑥 + cos 𝑥 0 cos 𝑥 + sin 𝑥
𝜋 𝜋
𝜋
2 sin𝑛
𝑥 + cos 𝑛 𝑥 2 𝜋
[𝑥] 2
=∫ 𝑛 𝑛
𝑑𝑥 = ∫ 𝑑𝑥 = 0 =
0 sin 𝑥 + cos 𝑥 0 2
𝜋 𝜋
2I = ⇒I=
2 4

Example 11: Evaluate


𝜋
2
∫ 𝑥 sin2 𝑥 cos 2 𝑥 𝑑𝑥 .
𝜋

2

Solution: Let 𝑓(𝑥) = 𝑥 sin2 𝑥 cos 2 𝑥


𝑓(−𝑥) = (−𝑥) sin2 (−𝑥) cos 2 (−𝑥)
= (−𝑥)(− sin 𝑥)2 (cos 𝑥)2
𝑓(−𝑥) = −𝑓(𝑥), i.e., the function 𝑓(𝑥) is odd. Then,
𝜋
2
∫ 𝑥 sin2 𝑥 cos 2 𝑥 𝑑𝑥 = 0.
𝜋

2

Example 12: Evaluate


𝜋
2
∫ 𝑥 3 sin2 𝑥 cos 𝑥 𝑑𝑥 .
𝜋

2

Solution: Let 𝑓(𝑥) = 𝑥 3 sin2 𝑥 cos 𝑥


𝑓(−𝑥) = (−𝑥)3 sin2 (−𝑥) cos(−𝑥)
= (−𝑥)(− sin 𝑥)2 cos 𝑥

117
Chapter 3: DEFINITE INTEGRALS
𝑓(−𝑥) = −𝑓(𝑥), i.e., the function 𝑓(𝑥) is odd. Then,
𝜋
2
∫ 𝑥 3 sin2 𝑥 cos 𝑥 𝑑𝑥 = 0.
𝜋

2

Example 13: Evaluate


𝜋
𝑥
∫ 𝑑𝑥 .
0 1 + sin 𝑥

Solution:

𝜋 𝜋 (𝜋 − 𝑥)
𝑥
I=∫ 𝑑𝑥 = ∫ 𝑑𝑥
0 1 + sin 𝑥 0 1 + sin(𝜋 − 𝑥)

𝜋 (𝜋 − 𝑥)
=∫ 𝑑𝑥
0 1 + sin 𝑥
𝜋 𝜋 (𝜋
𝑥 − 𝑥)
∵ I=∫ 𝑑𝑥 = ∫ 𝑑𝑥
0 1 + sin 𝑥 0 1 + sin 𝑥
Then,
𝜋 𝜋 (𝜋
𝑥 − 𝑥)
2I = ∫ 𝑑𝑥 + ∫ 𝑑𝑥
0 1 + sin 𝑥 0 1 + sin 𝑥
𝜋 𝜋
𝑥+𝜋−𝑥 𝜋
=∫ 𝑑𝑥 = ∫ 𝑑𝑥
0 1 + sin 𝑥 0 1 + sin 𝑥
𝜋
1 1 − sin 𝑥
= 𝜋∫ ∙ 𝑑𝑥
0 1 + sin 𝑥 1 − sin 𝑥
𝜋 𝜋
1 − sin 𝑥 1 sin 𝑥
= 𝜋∫ 2
𝑑𝑥 = 𝜋 ∫ [ 2
− ] 𝑑𝑥
0 1 − sin 𝑥 0 cos 𝑥 cos 2 𝑥
𝜋
= 𝜋 ∫ [sec 2 𝑥 − sec 𝑥 tan 𝑥]𝑑𝑥
0

118
Chapter 3: DEFINITE INTEGRALS
= 𝜋[tan 𝑥 − sec 𝑥]𝜋0
= 𝜋[(tan 𝜋 − sec 𝜋) − (tan 0 − sec 0)]
= 𝜋[(0 − (−1)) − (0 − 1)]
2I = 2𝜋 ⇒ I = 𝜋.

Example 14: Evaluate


𝜋
2
∫ ln(sin 𝑥) 𝑑𝑥 .
0

Solution:
𝜋 𝜋
2 2 𝜋
I = ∫ ln(sin 𝑥) 𝑑𝑥 = ∫ ln (sin ( − 𝑥)) 𝑑𝑥
0 0 2
𝜋
2
= ∫ ln(cos 𝑥) 𝑑𝑥
0
𝜋 𝜋
2 2
I = ∫ ln(sin 𝑥) 𝑑𝑥 = ∫ ln(cos 𝑥) 𝑑𝑥
0 0

Then,
𝜋 𝜋
2 2
2I = ∫ ln(sin 𝑥) 𝑑𝑥 + ∫ ln(cos 𝑥) 𝑑𝑥
0 0
𝜋
2
= ∫ [ln(sin 𝑥) + ln(cos 𝑥)] 𝑑𝑥
0
𝜋
2
= ∫ [ln(sin 𝑥 ∙ cos 𝑥)] 𝑑𝑥
0
𝜋
2 sin 2𝑥
= ∫ [ln ( )] 𝑑𝑥
0 2

119
Chapter 3: DEFINITE INTEGRALS
𝜋
2
= ∫ [ln(sin 2𝑥) − ln(2)] 𝑑𝑥
0
𝜋 𝜋
2 2
= ∫ ln(sin 2𝑥) 𝑑𝑥 − ln(2) ∫ 𝑑𝑥
0 0
𝜋
2 𝜋
= ∫ ln(sin 2𝑥) 𝑑𝑥 − ln(2)
0 2
Integrating by substituting, let

𝑢 = 2𝑥 𝑑𝑢 = 2 𝑑𝑥
1
𝑑𝑥 = 𝑑𝑢
2
at 𝑥 = 0 → 𝑢 = 0 𝜋
𝑥= → 𝑢=𝜋
2
Then,

1 𝜋 𝜋
2I = ∫ ln(sin 𝑢) 𝑑𝑢 − ln(2)
2 0 2
𝜋
1 2 𝜋
= 2 ∫ ln(sin 𝑢) 𝑑𝑢 − ln(2) [using property 8]
2 0 2
𝜋
2 𝜋
= ∫ ln(sin 𝑢) 𝑑𝑢 − ln(2)
0 2
𝜋
2I = I − ln(2)
2
𝜋
∴ I = − ln(2)
2

Example 15: Given


6, 𝑥>1
𝑓(𝑥) = {
3𝑥 2 , 𝑥≤1

120
Chapter 3: DEFINITE INTEGRALS
Find:
22 3
𝐚. ∫ 𝑓(𝑥)𝑑𝑥 , 𝐛. ∫ 𝑓(𝑥)𝑑𝑥
10 −2

Solution:
22 22
22
𝐚. ∫ 𝑓(𝑥)𝑑𝑥 = ∫ 6 𝑑𝑥 = 6[𝑥]10 = 6[22 − 10] = 72
10 10
3 1 3
𝐛. ∫ 𝑓(𝑥)𝑑𝑥 = ∫ 𝑓(𝑥)𝑑𝑥 + ∫ 𝑓(𝑥)𝑑𝑥
−2 −2 1
1 3
2
= ∫ 3𝑥 𝑑𝑥 + ∫ 6 𝑑𝑥
−2 1

= [𝑥 3 ]1−2 + 6[𝑥]13
= [1 − (−8)] + 6[3 − 1]
= 9 + 12 = 21.

3.3 Improper Integrals

𝑏
In defining a definite integral ∫𝑎 𝑓(𝑥) 𝑑𝑥 we dealt with a function 𝑓 defined on a
finite interval [𝑎, 𝑏] and we assumed that 𝑓 does not have an infinite discontinuity.
In this section we extend the concept of a definite integral to the case where the
interval is infinite and also to the case where 𝑓 has an infinite discontinuity in [𝑎, 𝑏].
In either case the integral is called an improper integral. One of the most important
applications of this idea, probability distributions.

121
Chapter 3: DEFINITE INTEGRALS
3.3.1 Type 1: Infinite Intervals

Consider the infinite region 𝑆 that lies under the curve


𝑦 = 1⁄𝑥 2 , above the x-axis, and to the right of the line
𝑥 = 1. You might think that, since 𝑆 is infinite in
extent, its area must be infinite, but let’s take a closer
look. The area of the part of 𝑆 that lies to the left of the
Fig. 3.4
line 𝑥 = 𝑡 (shaded in Figure 3.4) is

1𝑡
1𝑡 1
𝐴(𝑡) = ∫ 2 𝑑𝑥 = [− ] = 1 − .
1 𝑥 𝑥1 𝑡

Notice that 𝐴(𝑡) < 1 no matter how large 𝑡 is chosen.

We also observe that

1
lim 𝐴(𝑡) = lim (1 − ) = 1
𝑡→∞ 𝑡→∞ 𝑡

The area of the shaded region approaches 1 as 𝑡 → ∞ (see Figure 3.5), so we say
that the area of the infinite region 𝑆 is equal to 1 and we write
∞ 𝑡
1 1
∫ 2 𝑑𝑥 = lim ∫ 2 𝑑𝑥 = 1
1 𝑥 𝑡→∞ 1 𝑥

Fig. 3.5

122
Chapter 3: DEFINITE INTEGRALS
Using this example as a guide, we define the integral of 𝑓 (not necessarily a positive
function) over an infinite interval as the limit of integrals over finite intervals.

DEFINITION: We define the improper integral of 𝑓(𝑥) over the interval [𝑎, ∞) by
the limit:
∞ 𝑡
∫ 𝑓(𝑥)𝑑𝑥 = lim ∫ 𝑓(𝑥)𝑑𝑥
𝑎 𝑡→∞ 𝑎

If the limit exists, we say the improper integral converges, otherwise we say it
diverges.

Example 16: Investigate the convergence of



∫ 𝑒 −𝑥 𝑑𝑥.
0

Solution:
∞ 𝑡
−𝑥
∫ 𝑒 𝑑𝑥 = lim ∫ 𝑒 −𝑥 𝑑𝑥
0 𝑡→∞ 1

= lim [−𝑒 −𝑥 ]𝑡0 = lim [−𝑒 −𝑡 + 1] = 1.


𝑡→∞ 𝑡→∞

Thus, the integral converges and has value equal to 1.

Example 17: Investigate the convergence of



1
∫ 𝑑𝑥 .
1 𝑥

Solution:
∞ 𝑡
1 1
∫ 𝑑𝑥 = lim ∫ 𝑑𝑥
1 𝑥 𝑡→∞ 1 𝑥

123
Chapter 3: DEFINITE INTEGRALS
= lim [ln 𝑥]𝑡0 = lim [ln 𝑡 − ln 1]
𝑡→∞ 𝑡→∞

= lim [ln 𝑡] = ∞.
𝑡→∞

Thus, the limit does not exist as a finite number and so the improper integral is
divergent.

Example 18: Investigate the convergence of



1
∫ 𝑑𝑥 .
1 𝑥𝑝

Solution: For 𝑝 ≠ 1,

∞ 𝑡 𝑡
1 1
∫ 𝑑𝑥 = lim ∫ 𝑝 𝑑𝑥 = lim ∫ 𝑥 −𝑝 𝑑𝑥
1 𝑥 𝑡→∞ 1 𝑥 𝑡→∞ 1

𝑡
𝑥 1−𝑝
= lim [ ]
𝑡→∞ 1 − 𝑝
0

1
= lim [𝑡 1−𝑝 − 11−𝑝 ]
1 − 𝑝 𝑡→∞

1
1 1 , 𝑝>1
= lim [ 𝑝−1 − 1] = {1 − 𝑝
1 − 𝑝 𝑡→∞ 𝑡
∞, 𝑝<1

because,

1 0, 𝑝>1
lim [ 𝑝−1 ] = {
𝑡→∞ 𝑡 ∞, 𝑝<1

Therefore, the integral converges to the value 1⁄(𝑝 − 1) if 𝑝 > 1 and it diverges if

𝑝 < 1.

124
Chapter 3: DEFINITE INTEGRALS
If 𝑝 = 1, the integral also diverges
∞ 𝑡
1 1
∫ 𝑑𝑥 = lim ∫ 𝑑𝑥
1 𝑥 𝑡→∞ 1 𝑥

= lim [ln 𝑥]𝑡0 = lim [ln 𝑡 − ln 1]


𝑡→∞ 𝑡→∞

= lim [ln 𝑡] = ∞.
𝑡→∞

DEFINITION: We define the improper integral of 𝑓(𝑥) over the interval (−∞, 𝑎]
by the limit:
𝑏 𝑏
∫ 𝑓(𝑥)𝑑𝑥 = lim ∫ 𝑓(𝑥)𝑑𝑥
−∞ 𝑡→−∞ 𝑡

If the limit exists, we say the improper integral converges, otherwise we say it
diverges.

Example 19: Investigate the convergence of


0
∫ 𝑒 𝑥 𝑑𝑥.
−∞

Solution:
0 0
∫ 𝑒 𝑑𝑥 = lim ∫ 𝑒 𝑥 𝑑𝑥
𝑥
−∞ 𝑡→−∞ 𝑡

= lim [𝑒 𝑥 ]0𝑡 = lim [1 − 𝑒 𝑡 ] = 1.


𝑡→−∞ 𝑡→−∞

Thus, the integral converges and has value equal to 1.

125
Chapter 3: DEFINITE INTEGRALS
Example 20: Investigate the convergence of
1
1
∫ 2
𝑑𝑥.
−∞ 1 + 𝑥

Solution:
1 1
1 1
∫ 2
𝑑𝑥 = lim ∫ 𝑑𝑥
−∞ 1 + 𝑥 𝑡→−∞ 𝑡 1 + 𝑥 2

= lim [tan−1 𝑥]1𝑡 = lim [tan−1 1 − tan−1 𝑡]


𝑡→−∞ 𝑡→−∞

𝜋 𝜋 3𝜋
= + =
4 2 4
Thus, the integral converges and has value equal to 3𝜋⁄4.

DEFINITION: We define the improper integral of 𝑓(𝑥) over the interval (−∞, ∞)
by the limit:
∞ 𝑐 ∞
∫ 𝑓(𝑥)𝑑𝑥 = ∫ 𝑓(𝑥)𝑑𝑥 + ∫ 𝑓(𝑥)𝑑𝑥
−∞ −∞ 𝑐

Where 𝑐 is any real number, we say the improper integral converges only if both
𝑐 ∞
integrals ∫−∞ 𝑓(𝑥)𝑑𝑥 and ∫𝑐 𝑓(𝑥)𝑑𝑥 converges, otherwise we say the improper
integral diverges.

Example 21: Investigate the convergence of



1
∫ 2
𝑑𝑥.
−∞ 1 + 𝑥

Solution:

126
Chapter 3: DEFINITE INTEGRALS
∞ 0 ∞
1 1 1
∫ 2
𝑑𝑥 = ∫ 2
𝑑𝑥 + ∫ 2
𝑑𝑥
−∞ 1 + 𝑥 −∞ 1 + 𝑥 0 1+𝑥
0 𝑏
1 1
= lim ∫ 𝑑𝑥 + lim ∫ 𝑑𝑥
𝑎→−∞ 𝑎 1 + 𝑥 2 𝑏→∞ 0 1 + 𝑥 2

= lim [tan−1 𝑥]0𝑎 + lim [tan−1 𝑥]𝑏0


𝑎→−∞ 𝑏→∞

= lim [tan−1 0 − tan−1 𝑎] + lim [tan−1 𝑏 − tan−1 0]


𝑎→−∞ 𝑏→∞
𝜋 𝜋
= (0 + ) + ( − 0) = 𝜋.
2 2
Thus, the integral converges and has value equal to 𝜋.

Example 22: Investigate the convergence of



2
∫ 𝑥𝑒 −𝑥 𝑑𝑥.
−∞

Solution:

∞ 0 ∞
−𝑥 2 −𝑥 2 2
∫ 𝑥𝑒 𝑑𝑥 = ∫ 𝑥𝑒 𝑑𝑥 + ∫ 𝑥𝑒 −𝑥 𝑑𝑥
−∞ −∞ 0

0 𝑏
−𝑥 2 2
= lim ∫ 𝑥𝑒 𝑑𝑥 + lim ∫ 𝑥𝑒 −𝑥 𝑑𝑥
𝑎→−∞ 𝑎 𝑏→∞ 0

−1 −𝑥 2 0 −1 −𝑥 2 𝑏
= lim [ 𝑒 ] + lim [ 𝑒 ]
𝑎→−∞ 2 𝑎 𝑏→∞ 2 0

−1 1 −𝑎2 −1 2 1
= lim [ + 𝑒 ] + lim [ 𝑒 −𝑏 + ]
𝑎→−∞ 2 2 𝑏→∞ 2 2

−1 1 −1 1
=[ + ]+[ + ] = 0.
2 2 2 2

127
Chapter 3: DEFINITE INTEGRALS
Thus, the integral converges and has value equal to 0.

Example 23: Investigate the convergence of



∫ 𝑒 −𝑥 𝑑𝑥.
−∞

Solution:

∞ 0 ∞
−𝑥 −𝑥
∫ 𝑒 𝑑𝑥 = ∫ 𝑒 𝑑𝑥 + ∫ 𝑒 −𝑥 𝑑𝑥
−∞ −∞ 0

0 𝑏
−𝑥
= lim ∫ 𝑒 𝑑𝑥 + lim ∫ 𝑒 −𝑥 𝑑𝑥
𝑎→−∞ 𝑎 𝑏→∞ 0

= lim [−𝑒 −𝑥 ]0𝑎 + lim [−𝑒 −𝑥 ]𝑏0


𝑎→−∞ 𝑏→∞

= lim [−1 + 𝑒 −𝑎 ] + lim [−𝑒 −𝑏 + 1]


𝑎→−∞ 𝑏→∞

= [−1 + ∞] + [0 + 1] = ∞.

Thus, the integral diverges.

Example 24: Investigate the convergence of



𝑑𝑥
∫ −𝑥 + 𝑒 𝑥
.
−∞ 𝑒

Solution:

∞ ∞ ∞
𝑑𝑥 𝑒𝑥 𝑒𝑥
∫ −𝑥 + 𝑒 𝑥
=∫ 2𝑥
𝑑𝑥 = ∫ 𝑥 )2
𝑑𝑥
−∞ 𝑒 −∞ 1 + 𝑒 −∞ 1 + (𝑒

128
Chapter 3: DEFINITE INTEGRALS
0 ∞
𝑒𝑥 𝑒𝑥
=∫ 𝑥 2
𝑑𝑥 + ∫ 𝑥 2
𝑑𝑥
−∞ 1 + (𝑒 ) 0 1 + (𝑒 )

0 𝑏
𝑒𝑥 𝑒𝑥
= lim ∫ 𝑑𝑥 + lim ∫ 𝑑𝑥
𝑎→−∞ 𝑎 1 + (𝑒 𝑥 )2 𝑏→∞ 0 1 + (𝑒 𝑥 )2

= lim [tan−1 (𝑒 𝑥 )]0𝑎 + lim [tan−1 (𝑒 𝑥 )]𝑏0


𝑎→−∞ 𝑏→∞

= lim [tan−1 (𝑒 0 ) − tan−1 (𝑒 𝑎 )] + lim [tan−1 (𝑒 𝑏 ) − tan−1 (𝑒 0 )]


𝑎→−∞ 𝑏→∞

𝜋 𝜋
= − lim [tan−1 (𝑒 𝑎 )] + lim [tan−1 (𝑒 𝑏 )] −
4 𝑎→−∞ 𝑏→∞ 4
𝜋 𝜋
= − tan−1 (𝑒 −∞ ) + tan−1 (𝑒 ∞ ) = 0 + = .
2 2

Thus, the integral converges and has value equal to 𝜋⁄2.

3.3.2 Type 2: Discontinuous Integrands

Another type of improper integral arises when the integrand has a vertical
asymptote—an infinite discontinuity—at a limit of integration or at some point
between the limits of integration. If the integrand 𝑓 is positive over the interval of
integration, we can again interpret the improper integral as the area under the graph
of 𝑓 and above the x-axis between the limits of integration.

129
Chapter 3: DEFINITE INTEGRALS
DEFINITION Integrals of functions that become infinite at a point within the

interval of integration are improper integrals of Type 2.

1- If 𝑓(𝑥) is continuous on (𝑎, 𝑏] and discontinuous at 𝑎, then


𝑏 𝑏
∫ 𝑓(𝑥) 𝑑𝑥 = lim+ ∫ 𝑓(𝑥) 𝑑𝑥.
𝑎 𝑐→𝑎 𝑐

2- If 𝑓(𝑥) is continuous on [𝑎, 𝑏) and discontinuous at 𝑏, then


𝑏 𝑐
∫ 𝑓(𝑥) 𝑑𝑥 = lim− ∫ 𝑓(𝑥) 𝑑𝑥.
𝑎 𝑐→𝑏 𝑎

3- If 𝑓(𝑥)is discontinuous at 𝑐, where 𝑎 < 𝑐 < 𝑏, and continuous on [𝑎, 𝑐) ∪


(𝑐, 𝑏], then
𝑏 𝑐 𝑏
∫ 𝑓(𝑥) 𝑑𝑥 = ∫ 𝑓(𝑥) 𝑑𝑥 + ∫ 𝑓(𝑥) 𝑑𝑥
𝑎 𝑎 𝑐

In each case, if the limit is finite, we say the improper integral converges and that
the limit is the value of the improper integral. If the limit does not exist, the integral

diverges.

Example 25: Investigate the convergence of


1
𝑑𝑥
∫ .
0 1−𝑥

Solution: The integrand 𝑓(𝑥) = 1⁄(1 − 𝑥) is continuous on[0,1) but is


discontinuous at 𝑥 = 1 and becomes infinite as 𝑥 → 1− (Figure 3.6).

130
Chapter 3: DEFINITE INTEGRALS
We evaluate the integral as
𝑏
𝑑𝑥
lim− ∫ = lim−[ln(1 − 𝑥)]𝑏0
𝑏→1 0 1 − 𝑥 𝑏→1

= lim−[ln(1 − 𝑏) − ln(1)]
𝑏→1

= ln ∞ − 0 = ∞.

The limit is infinite, so the integral diverges.

Fig. 3.6
Example 26: Investigate the convergence of
3
𝑑𝑥
∫ 2.
0 (𝑥 − 1)3
Solution: The integrand has a vertical asymptote at 𝑥 = 1
and is continuous on [0,1) and (1,3] (Figure 3.7). Thus, by
Part 3 of the definition above,
3 1 3
𝑑𝑥 𝑑𝑥 𝑑𝑥
∫ 2 =∫ 2+∫ 2
0 (𝑥 − 1)3 0 (𝑥 − 1)3 1 (𝑥 − 1)3
𝑏 3
𝑑𝑥 𝑑𝑥
= lim− ∫ 2 + lim+ ∫ 2
𝑏→1 0 𝑐→1 𝑐
(𝑥 − 1)3 (𝑥 − 1)3
Fig. 3.7
1 𝑏 1 3
= lim− [3(𝑥 − 1)3 ] + lim+ [3(𝑥 − 1)3 ]
𝑏→1 0 𝑐→1 𝑐

1 𝑏 1 1 3
= lim− [3(𝑏 − 1)3 + 3] + lim+ [3(3 − 1)3 − 3(𝑐 − 1)3 ]
𝑏→1 0 𝑐→1 𝑐

3
= 3 + 3√2.

131
Chapter 3: DEFINITE INTEGRALS
3
Thus, the integral converges and has value equal to (3 + 3√2).

132
Chapter 3: DEFINITE INTEGRALS
Exercises (3)

1. Evaluate the following integrals:

3 5 1
∫ 𝑥 2 𝑑𝑥 ∫ (𝑥 2 + 4)2 𝑑𝑥
1 0
2 14
∫ (𝑥 2 + +3𝑥 − 5)𝑑𝑥 ∫ 𝑥 −0.5 𝑑𝑥
1 1
4 𝜋
2
∫ √𝑥𝑑𝑥 ∫ cos 𝑥 𝑑𝑥
1 0
𝜋 𝜋
6 3
∫ cos 3𝑥 𝑑𝑥 ∫ sin 𝑥 𝑑𝑥
0 0
𝜋 𝜋
2 4 𝜋
∫ (cos 𝑥 − sin 2𝑥)𝑑𝑥 ∫ cos (2𝜃 + ) 𝑑𝜃
0 0 4
2 𝑥 𝜋
2
∫ 𝑒2 𝑑𝑥 ∫ sin2 𝑥 𝑑𝑥
0 0
𝜋 𝜋
2 4
∫ cos 2 𝑥 𝑑𝑥 ∫ sec 2 𝑥 𝑑𝑥
0 0
𝜋 𝜋
4 2
∫ tan2 𝑥 𝑑𝑥 ∫ 𝑥 cos 𝑥 𝑑𝑥
0 0
𝜋 𝜋
2 2
∫ 𝑥 sin 𝑥 𝑑𝑥 ∫ 𝑥 2 sin 𝑥 𝑑𝑥
0 0
1 2
∫ 𝑥 ln 𝑥 𝑑𝑥 ∫ 𝑥 2 ln 𝑥 𝑑𝑥
0.5 1
3 𝑏
𝑥
∫ 𝑑𝑥 ∫ 𝑒 𝑘𝑥 𝑑𝑥
2 1 + 𝑥2 𝑎

133
Chapter 3: DEFINITE INTEGRALS
−1 1
𝑑𝑥 𝑑𝑥
∫ 3

−2 (𝑥 − 2) 0 √12 − 4𝑥 − 𝑥 2
2 1
𝑑𝑥 𝑑𝑥
∫ ∫
0 √𝑥 2 + 2𝑥 + 2 0 √𝑎2 + 𝑥 2 2

2. Evaluate the following integrals:


𝜋 𝜋
2 sin2 𝑥 2 sin3 𝑥
∫ 2 𝑥 + cos 2 𝑥
𝑑𝑥 ∫ 3 𝑥 + cos 3 𝑥
𝑑𝑥
0 sin 0 sin
𝜋 𝜋
2 sin 𝑥 2 √sin 𝑥
∫ 𝑑𝑥 ∫ 𝑑𝑥
0 sin 𝑥 + cos 𝑥 0 √sin 𝑥 + √cos 𝑥
𝜋 3 𝜋
2 sin2 𝑥 2
∫ ∫ ln(tan 𝑥) 𝑑𝑥 .
3 3 𝑑𝑥
0 sin2 0
𝑥+ cos 2 𝑥
𝜋
∫ 𝑒 −|𝑥| sin 𝑥 𝑑𝑥
−𝜋

3. Investigate the convergence of:

∞ ∞
−𝑥
𝑑𝑥
∫ 3 𝑑𝑥 ∫
0 1 𝑥2
∞ 0
𝑑𝑥 𝑑𝑥
∫ 2 ∫
1 𝑥 ln 𝑥 −∞ 𝑥 2 + 16
0 ∞
𝑑𝑥 𝑑𝑥
∫ ∫
2
−∞ 3𝑥 + 4 −∞ 𝑥2 + 5

𝑑𝑥
∫ 2
−∞ 5𝑥 + 13

134
Chapter 3: DEFINITE INTEGRALS
Notes:

135
Chapter 3: DEFINITE INTEGRALS

136
Chapter 3: DEFINITE INTEGRALS

137
Chapter 3: DEFINITE INTEGRALS

138
Chapter 4: APPLICATIONS OF INTEGRATIONS

Chapter 4. APPLICATIONS OF
INTEGRATIONS

Overview Integration, a cornerstone of calculus, serves as an indispensable tool


for solving complex geometric problems that defy simple geometric formulas. By
transforming abstract mathematical concepts into tangible solutions, integration
allows us to analyze and compute properties of curves, surfaces, and solids with
remarkable precision. This chapter focuses on the profound geometric applications
of integration, bridging theory and practice to unlock solutions in fields such as
engineering, architecture, physics, and computer graphics.

From calculating the area between intricate curves to determining the volume of
irregularly shaped objects, integration provides a universal framework for
understanding spatial relationships. We will explore how techniques like the disk
method, washer method, and shell method enable us to reconstruct three-
dimensional solids from two-dimensional slices. Additionally, integration extends
to finding arc lengths of nonlinear paths, surface areas of revolved shapes.

By mastering these applications, you will not only deepen your mathematical
intuition but also gain the analytical tools to tackle real-world challenges where
geometry and calculus intersect. Let us delve into the elegance and utility of
integration in shaping our understanding of space, form, and function.

139
Chapter 4: APPLICATIONS OF INTEGRATIONS
4.1 Areas

In this section, we will learn how to find the area


bounded by the curve 𝑦 = 𝑓 (𝑥), the ordinates 𝑥 =
𝑎, 𝑥 = 𝑏 and 𝑥-axis, while calculating definite
integral as the limit of a sum, and we shall study a
specific application of integrals to find the area under
simple curves, area between lines and arcs of circles,
parabolas and ellipses (standard forms only). We
Fig. 4.1
shall also deal with finding the area bounded by the
above said curves.

4.1.1 Area under Simple Curves

In the previous chapter, we have studied definite integral as the limit of a sum and
how to evaluate definite integral using Fundamental Theorem of Calculus. Now, we
consider the easy and intuitive way of finding the area bounded by the curve 𝑦 =
𝑓(𝑥), 𝑥-axis and the ordinates 𝑥 = 𝑎 and 𝑥 = 𝑏. From Fig 4.2, we can think of area
under the curve as composed of large number of very thin vertical strips. Consider
an arbitrary strip of height 𝑦 and width 𝑑𝑥, then
𝑑𝐴 (area of the elementary strip) = 𝑦𝑑𝑥, where, 𝑦 = 𝑓(𝑥).

Fig. 4.2

140
Chapter 4: APPLICATIONS OF INTEGRATIONS
This area is called the elementary area which is located at an arbitrary position within
the region which is specified by some value of 𝑥 between 𝑎 and 𝑏. We can think of
the total area 𝐴 of the region between 𝑥-axis, ordinates 𝑥 = 𝑎, 𝑥 = 𝑏 and the curve
𝑦 = 𝑓(𝑥) as the result of adding up the elementary areas of thin strips across the
shaded region. Symbolically, we express

𝑏 𝑏 𝑏
𝐴 = ∫ 𝑑𝐴 = ∫ 𝑦 𝑑𝑥 = ∫ 𝑓(𝑥) 𝑑𝑥
𝑎 𝑎 𝑎

The area 𝐴 of the region bounded by the curve


𝑥 = 𝑔 (𝑦), 𝑦-axis and the lines 𝑦 = 𝑎, 𝑦 = 𝑏 is given
by
𝑏 𝑏
𝐴 = ∫ 𝑥 𝑑𝑦 = ∫ 𝑔(𝑦) 𝑑𝑦
𝑎 𝑎 Fig. 4.3

If the position of the curve under consideration is below the 𝑥-axis, then since
𝑓(𝑥) < 0 from 𝑥 = 𝑎 to 𝑥 = 𝑏, the area bounded by the curve, 𝑥-axis and the
ordinates 𝑥 = 𝑎, 𝑥 = 𝑏 come out to be negative. But it is only the numerical value
of the area which is taken into consideration. Thus, if the area is negative, we take
𝑏
its absolute value, i.e., |∫𝑎 𝑓(𝑥) 𝑑𝑥|.

Example 1: Find the area 𝐴 under 𝑦 = 2 over the


interval [1,4].

Solution: (area of rectangle)

𝑏 4
𝐴 = ∫ 𝑦 𝑑𝑥 = ∫ 2 𝑑𝑥 = 2𝑥|14 = 2(3) = 6.
𝑎 1
Fig. 4.4

141
Chapter 4: APPLICATIONS OF INTEGRATIONS
Example 2: Find the area 𝐴 under 𝑦 = 𝑥 + 2 over the interval [−1,2].

Solution: (area of Trapezoid)

2 2
𝑥2 1
𝐴 = ∫ (𝑥 + 2) 𝑑𝑥 = + 2𝑥| = (4 − 1) + 2(3)
−1 2 −1
2

1 15
= (1 + 4)(3) = .
2 3

Fig. 4.5

Example 3: Find the area 𝐴 under 𝑦 = √1 − 𝑥 2 over the


interval [0,1].

Solution: (area of quarter-circle)

The graph of 𝑦 = √1 − 𝑥 2 is the upper semicircle of radius


1, centered at the origin, so the region is the right quarter-
circle extending from 𝑥 = 0 to 𝑥 = 1 Fig. 4.6

1
1 𝜋
𝐴 = ∫ √1 − 𝑥 2 𝑑𝑥 = 𝜋(12 ) = .
0 4 4

Example 4: Find the area bounded by the curve 𝑦 = 2𝑥 2 + 3 ,the 𝑥-axis and the
lines 𝑥 = −1 and 𝑥 = 3.

Solution:

𝑏 3
𝐴 = ∫ 𝑦 𝑑𝑥 = ∫ (2𝑥 2 + 3) 𝑑𝑥
𝑎 −1

142
Chapter 4: APPLICATIONS OF INTEGRATIONS
3
2 3 −2
= [ 𝑥 + 3𝑥] = [(18 + 9) − ( − 3)]
3 −1 3

11 92
= 27 + = .
3 3

Example 5: Find the area bounded by the curve 𝑦 = 𝑥 ,the 𝑥-axis and the lines
𝑥 = 0 and 𝑥 = 2.

Solution:

𝑏 2
1 2 2
𝐴 = ∫ 𝑦 𝑑𝑥 = ∫ 𝑥 𝑑𝑥 = [ 𝑥 ] = (2 − 0) = 2.
𝑎 0 2 0

Example 6: Find the area bounded by the curve 𝑦 = 𝑥 2 − 4 ,the 𝑥-axis and the
lines 𝑥 = −1 and 𝑥 = 2.

Solution:

𝑏 2 2
1 3
𝐴 = ∫ 𝑦 𝑑𝑥 = ∫ (𝑥 2 − 4) 𝑑𝑥 = [ 𝑥 − 4𝑥]
𝑎 −1 3 −1

8 −1 16 11 27
= [( − 8) − ( + 4)] = − − =−
3 3 3 3 3

∴ 𝐴 = 9.

143
Chapter 4: APPLICATIONS OF INTEGRATIONS
Example 7: Find the area bounded by the curve
𝑦 = sin 𝑥 ,the 𝑥-axis over [0,2𝜋].

Solution: The area between the graph of 𝑦 =


sin 𝑥 and the x-axis over [0,2𝜋]. is calculated by

breaking up the domain of sin 𝑥 into two pieces:


the interval [0, 𝜋]. over which it is nonnegative
and the interval [𝜋, 2𝜋].over which it is
nonpositive
𝜋 2𝜋
𝐴 = ∫ sin 𝑥 𝑑𝑥 + ∫ sin 𝑥 𝑑𝑥
0 𝜋

𝜋
∫ sin 𝑥 𝑑𝑥 = [− cos 𝑥]𝜋0 = −[cos 𝜋 − cos 0] = −[−1 − 1] = 2
0

2𝜋
∫ sin 𝑥 𝑑𝑥 = [− cos 𝑥]2𝜋
𝜋 = −[cos 𝜋 − cos 2𝜋] = −[1 − (−1)] = −2
𝜋

∴ 𝐴 = 2 + |−2| = 4.

Example 8: Find the area of the region between the


𝑥-axis and the graph of 𝑓(𝑥) = 𝑥 3 − 𝑥 2 − 2𝑥,
−1 ≤ 𝑥 ≤ 2.

Solution: First find the zeros of 𝑓(𝑥). Since

𝑓(𝑥) = 𝑥 3 − 𝑥 2 − 2𝑥 = 𝑥(𝑥 2 − 𝑥 − 2)
= 𝑥(𝑥 + 1)(𝑥 − 2),

144
Chapter 4: APPLICATIONS OF INTEGRATIONS
the zeros are 𝑥 = 0, −1, and 2 .The zeros subdivide [−1,2] into two subintervals:

[−1,0], on which 𝑓(𝑥) ≥ 0, and [0,2], on which 𝑓(𝑥) ≤ 0. We integrate 𝑓 over each
subinterval and add the absolute values of the calculated integrals.

0 0
𝑥4 𝑥3
∫ (𝑥 3 − 𝑥 − 2𝑥) 𝑑𝑥 = [ − − 𝑥 2 ]
2
−1 4 3 −1

1 1 5
= 0 − [ + − 1] = ,
4 3 12

2 2
𝑥4 𝑥3
∫ (𝑥 3 − 𝑥 − 2𝑥) 𝑑𝑥 = [ − − 𝑥 2 ]
2
0 4 3 0

8 8
= [4 − − 4] − 0 = −
3 3

The total enclosed area is obtained by adding the absolute values of the calculated
integrals.

5 8 37
∴𝐴= + |− | = .
12 3 12

Example 9: Find the area of the region bounded by the curve 𝑦 = √𝑥 − 1, the 𝑦-
axis and the lines 𝑦 = 1, 𝑦 = 5.

Solution:

∵ 𝑦 = √𝑥 − 1

∴ 𝑦2 = 𝑥 − 1 ⟹ 𝑥 = 𝑦2 + 1

145
Chapter 4: APPLICATIONS OF INTEGRATIONS
𝑏 5 5
𝑦3
𝐴 = ∫ 𝑥 𝑑𝑦 = ∫ (𝑦 2 + 1) 𝑑𝑦 = [ + 𝑦]
𝑎 1 3 1

125 1 140 4 136


= [( + 5) − ( + 1)] = − = .
3 3 3 3 3

Example 10: Find the area of the circle of radius 𝑟.

Solution: The equation of circle of radius 𝑎 is given by

𝑥 2 + 𝑦 2 = 𝑎2

The circle is symmetric with respect to 𝑥 and 𝑦 axes. Hence, we can find the area of
one quarter of the circle and multiply by 4 in order to obtain the total area of the
circle.

We express 𝑦 as a function of 𝑥 as

𝑥 2 + 𝑦 2 = 𝑎2 ⟹ 𝑦 = ±√𝑎2 − 𝑥 2

We use the integrals to find the area of the upper right quarter of the circle as follows
𝑏 𝑎
𝐴 = ∫ 𝑦 𝑑𝑥 = 4 ∫ √𝑎2 − 𝑥 2 𝑑𝑥
𝑎 0

Let 𝑥 = 𝑎 sin 𝜃 𝑑𝑥 = 𝑎 cos 𝜃 𝑑𝜃

At 𝑥 = 0 ⟹ 𝜃 = 0
𝜋
𝑥=𝑎 ⟹𝜃=
2
𝜋
2
𝐴 = 4 ∫ √𝑎2 − 𝑎2 sin2 𝜃 𝑎 cos 𝜃 𝑑𝜃
0

146
Chapter 4: APPLICATIONS OF INTEGRATIONS
𝜋
2
𝐴 = 4 ∫ √𝑎2 (1 − sin2 𝜃) 𝑎 cos 𝜃 𝑑𝜃
0

𝜋 𝜋
2 21
𝐴 = 4𝑎2 ∫ cos 2 𝜃 𝑑𝜃 = 4𝑎2 ∫ (1 + cos 2𝜃) 𝑑𝜃
0 0 2
𝜋
1 2 𝜋
= 2𝑎2 [𝜃 + sin 2𝜃] = 2𝑎2 [ − 0]
2 0 2

∴ 𝐴 = 𝜋𝑎2 .

Example 11: Find the area of the ellipse with major axis length 2𝑎 and minor axis
length 2𝑏.

Solution: we can find the area of the first


quadrant and multiply it by 4 to find the total
area.

The equation of the ellipse is given by

𝑥2 𝑦2
+ =1
𝑎2 𝑏 2

𝑦2 𝑥2 𝑎2 − 𝑥 2 𝑏2 2
=1− 2 = ⟹ 𝑦 = 2 (𝑎 − 𝑥 2 )
2
𝑏2 𝑎 𝑎2 𝑎
𝑏
∴ 𝑦 = ± √𝑎 2 − 𝑥 2
𝑎
𝑏
𝑏 𝑎 2
𝐴 = ∫ 𝑦 𝑑𝑥 = 4 ∫ √𝑎 − 𝑥 2 𝑑𝑥
𝑎 𝑎 0

Let 𝑥 = 𝑎 sin 𝜃 𝑑𝑥 = 𝑎 cos 𝜃 𝑑𝜃

147
Chapter 4: APPLICATIONS OF INTEGRATIONS
At 𝑥 = 0 ⟹ 𝜃 = 0
𝜋
𝑥=𝑎 ⟹𝜃=
2
𝜋
𝑏 2
𝐴 = 4 ∫ √𝑎2 − 𝑎2 sin2 𝜃 𝑎 cos 𝜃 𝑑𝜃
𝑎 0
𝜋 𝜋
𝑏 2 21
= 4 𝑎2 ∫ cos 2 𝜃 𝑑𝜃 = 4𝑎𝑏 ∫ (1 + cos 2𝜃) 𝑑𝜃
𝑎 0 0 2

𝜋 𝜋
1 2 1 2 𝜋
= 2𝑎𝑏 [𝜃 + sin 2𝜃] = 2𝑎𝑏 [𝜃 + sin 2𝜃] = 2𝑎𝑏 [ − 0]
2 0 2 0 2

∴ 𝐴 = 𝜋 𝑎𝑏.

4.1.2 Areas Between Curves

Suppose we want to find the area of a region that is


bounded above by the curve 𝑦 = 𝑓(𝑥), below by the
curve = g(𝑥), and on the left and right by the lines 𝑥 =
𝑎 and 𝑥 = 𝑏 (Figure 4.7). The region might accidentally
have a shape whose area we could find with geometry,
Fig. 4.7
but if 𝑓 and g are arbitrary continuous functions, we
usually have to find the area with an integral. To see
what the integral should be, we first approximate the
region with 𝑛 vertical

rectangles based on a partition 𝑃 = {𝑥0 , 𝑥1 , … , 𝑥𝑛 } of


[𝑎, 𝑏] (Figure 4.8).
Fig. 4.8

148
Chapter 4: APPLICATIONS OF INTEGRATIONS
DEFINITION If 𝑓 and g are continuous with 𝑓(𝑥) ≥ g(𝑥) throughout [𝑎, 𝑏], then
the area of the region between the curves 𝒚 = 𝒇(𝒙) and 𝒚 = 𝐠(𝒙) from 𝒂 to 𝒃 is
the integral of (𝑓 − g) from 𝑎 to 𝑏:
𝑏
𝐴 = ∫ [𝑓(𝑥) − g(𝑥)] 𝑑𝑥.
𝑎

It is possible that the upper and lower boundaries of a region may intersect at one or

both endpoints, in which case the sides of the region will be points, rather than
vertical line segments (Figure 4.9). When that occurs, you will have to determine the
points of intersection to obtain the limits of integration.

Fig. 4.9

Integration with Respect to 𝑦

If a region’s bounding curves are described by functions of y, the approximating


rectangles are horizontal instead of vertical and the basic formula has y in place of
x.

For regions like these:

149
Chapter 4: APPLICATIONS OF INTEGRATIONS

Fig. 4.10
use the formula
𝑑
𝐴 = ∫ [𝑓(𝑦) − g(𝑦)] 𝑑𝑦.
𝑐

In this equation 𝑓 always denotes the right-hand curve and g the left-hand curve, so

𝑓(𝑦) − g(𝑦) is nonnegative.

Example 12: Find the area of the region enclosed by the parabola 𝑦 = 2 − 𝑥 2 and

the line 𝑦 = −𝑥.

Solution: The limits of integration are found by


solving 𝑦 = 2 − 𝑥 2 and 𝑦 = −𝑥 simultaneously for 𝑥.

2 − 𝑥 2 = −𝑥 [Equate 𝑓(𝑥) and g(𝑥)]

𝑥2 − 𝑥 − 2 = 0

(𝑥 + 1)(𝑥 − 2) = 0

𝑥 = −1, 𝑥 = 2.

The region runs from 𝑥 = −1 to 𝑥 = 2. The area Fig. 4.11

between the curves is

150
Chapter 4: APPLICATIONS OF INTEGRATIONS
𝑏 2
𝐴 = ∫ [𝑓(𝑥) − g(𝑥)] 𝑑𝑥 = ∫ [(2 − 𝑥 2 ) − (−𝑥)] 𝑑𝑥
𝑎 −1

2 2
2
𝑥2 𝑥3
= ∫ (2 + 𝑥 − 𝑥 ) 𝑑𝑥 = [2𝑥 + − ]
−1 2 3 −1

4 8 1 1 9
= [(4 + − ) − (−2 + + )] = .
2 3 2 3 2

Example 13: Find the area of the region in the first quadrant that is bounded above

by 𝑦 = √𝑥 and below by the 𝑥 −axis and the line 𝑦 = 𝑥 − 2.

Solution: It takes two integrations to find the


area of this region if we integrate with respect
to 𝑥. It takes only one if we integrate with
respect to 𝑦.

The lower limit of integration is 𝑦 = 0. We


find the upper limit by solving 𝑥 = 𝑦 + 2 and

𝑥 = 𝑦 2 simultaneously for 𝑦:

𝑦 + 2 = 𝑦2 [Equate 𝑓(𝑦) and g(𝑦)] Fig. 4.12

𝑦2 − 𝑦 − 2 = 0

(𝑦 + 1)(𝑦 − 2) = 0

𝑦 = −1, 𝑦 = 2.

The upper limit of integration is 𝑏 = 2. (The value 𝑦 = −1 gives a point of


intersection below the 𝑥-axis.)

151
Chapter 4: APPLICATIONS OF INTEGRATIONS
The area of the region is
𝑑 2
𝐴 = ∫ [𝑓(𝑦) − g(𝑦)] 𝑑𝑦 = ∫ [𝑦 + 2 − 𝑦 2 ] 𝑑𝑦
𝑐 0

2
𝑦2 𝑥3 4 8 10
= [ + 2𝑦 − ] = + 4 − = .
2 3 0 2 3 3

Example 14: Find the area of the region enclosed by the parabolas 𝑦 = 𝑥 2 and
𝑦 = 2𝑥 − 𝑥 2 .

Solution: We first find the points of intersection


of the parabolas by solving their equations
simultaneously

𝑥 2 = 2𝑥 − 𝑥 2 [Equate 𝑓(𝑥) and g(𝑥)]

2𝑥 2 − 2𝑥 = 0

2𝑥(𝑥 − 1) = 0 Fig. 4.13

𝑥 = 0, 𝑥 = 1.

The region lies between 𝑥 = 0 and 𝑥 = 1. So, the total area is


𝑏 1
𝐴 = ∫ [𝑓(𝑥) − g(𝑥)] 𝑑𝑥 = ∫ [2𝑥 − 2𝑥 2 ] 𝑑𝑥
𝑎 0

1
𝑥2 𝑥3 1 1 1
= 2[ − ] = 2[ − ] = .
2 3 0 2 3 3

152
Chapter 4: APPLICATIONS OF INTEGRATIONS
Example 15: Find the area of the region bounded by the curves 𝑦 = sin 𝑥, 𝑦 =
cos 𝑥, 𝑥 = 0 and 𝑥 = 𝜋⁄2.

Solution: The points of intersection occur

when sin 𝑥 = cos 𝑥, that is, when 𝑥 = 𝜋⁄4.

𝜋
2
𝐴 = ∫ |cos 𝑥 − sin 𝑥| 𝑑𝑥 = 𝐴1 + 𝐴2
0

𝜋
4
= ∫ (cos 𝑥 − sin 𝑥) 𝑑𝑥
0 Fig. 4.14
𝜋
2
+ ∫ (sin 𝑥 − cos 𝑥) 𝑑𝑥
𝜋
4

𝜋 𝜋
= [sin 𝑥 + cos 𝑥]04 + [− cos 𝑥 − sin 𝑥]𝜋2
4

1 1 1 1
=( + − 0 − 1) + (−0 − 1 + + )
√2 √2 √2 √2

𝐴 = 2√2 − 2.

In this particular example we could have saved some work by noticing that the region

is symmetric about 𝑥 = 𝜋⁄4 and so,


𝜋
4
𝐴 = 2𝐴2 = 2 ∫ (cos 𝑥 − sin 𝑥) 𝑑𝑥.
0

153
Chapter 4: APPLICATIONS OF INTEGRATIONS
Example 16: Find the area enclosed by the line 𝑦 = 𝑥 − 1 and the parabola 𝑦 2 =
2𝑥 + 6.
Solution: By solving the two equations we find
that the points of intersection are
(−1, −2) and (5,4).

We must integrate between the appropriate 𝑦-


values, 𝑦 = −2 and 𝑦 = 4. Thus

1
𝑥 =𝑦+1 and 𝑥 = 𝑦2 − 3
2
4
1 Fig. 4.15
𝐴 = ∫ [(𝑦 + 1) − ( 𝑦 2 − 3)] 𝑑𝑦
−2 2

4 4
1 2 1 𝑦3 𝑦2
= ∫ (− 𝑦 + 𝑦 + 4) 𝑑𝑦 = [− ( ) + + 4𝑦]
−2 2 2 3 2 −2

1 4
= − (64) + 8 + 16 − ( + 2 − 8) = 18.
6 3

We could have found the area in Example 16 by


integrating with respect to 𝑥 instead of 𝑦, but the
calculation is much more involved. It would
have meant splitting the region in two and
computing the areas labeled 𝐴1 and 𝐴2 in Figure
4.16. The method we used in Example 16 is
much easier.

Fig. 4.16

154
Chapter 4: APPLICATIONS OF INTEGRATIONS
4.2 Volumes

In this section we define volumes of solids using the areas of their cross-sections. A
cross section of a solid 𝑆 is the plane region formed by intersecting 𝑆 with a plane
(Figure 4.18). We present three different methods for obtaining the cross-sections
appropriate to finding the volume of a particular solid: the method of slicing, the
disk method, and the washer method.

Suppose we want to find the volume of a solid 𝑆 like


the one in Figure 4.17. We begin by extending the
definition of a cylinder from classical geometry to
cylindrical solids with arbitrary bases (Figure 4.18).
If the cylindrical solid has a known base area 𝐴 and
height ℎ, then the volume of the cylindrical solid is
Fig. 4.17
Volume = area × height = 𝐴 ∙ ℎ.

This equation forms the basis for defining the volumes of many solids that are not
cylinders, like the one in Figure 4.17. If the cross-section of the solid 𝑆 at each point
𝑥 in the interval [𝑎, 𝑏] is a region 𝑆(𝑥) of area 𝐴(𝑥), and 𝐴 is a continuous function
of 𝑥, we can define and calculate the volume of the solid 𝑆 as the definite integral of
𝐴(𝑥). We now show how this integral is obtained by the method of slicing.

Fig. 4.18

155
Chapter 4: APPLICATIONS OF INTEGRATIONS
4.2.1 Slicing by Parallel Planes

We partition [𝑎, 𝑏] into subintervals of width (length)


∆𝑥𝑘 and slice the solid, as we would a loaf of bread,
by planes perpendicular to the 𝑥-axis at the partition
points 𝑎 = 𝑥0 < 𝑥1 < ⋯ < 𝑥𝑛 = 𝑏. The planes 𝑃𝑥𝑘 ,
perpendicular to the 𝑥-axis at the partition points, slice
𝑆 into thin “slabs”. A typical slab is shown in Figure
4.19. Fig. 4.19

We define the limiting definite integral of the Riemann sum to be the volume of the
solid 𝑆.

DEFINITION The volume of a solid of integrable cross-sectional area 𝐴(𝑥) from

𝑥 = 𝑎 to 𝑥 = 𝑏 is the integral of 𝐴 from 𝑎 to 𝑏,


𝑏
𝑉 = ∫ 𝐴(𝑥) 𝑑𝑥.
𝑎

This definition applies whenever 𝐴(𝑥) is integrable, and in particular when it is


continuous. To apply the definition to calculate the volume of a solid using cross-
sections perpendicular to the 𝑥-axis, take the following steps:

1. Sketch the solid and a typical cross-section.


2. Find a formula for 𝐴(𝑥), the area of a typical cross-section.
3. Find the limits of integration.
4. Integrate 𝐴(𝑥) to find the volume.

156
Chapter 4: APPLICATIONS OF INTEGRATIONS
Example 17: Derive the formula for the volume of a right pyramid whose altitude
is ℎ and whose base is a square with sides of length 𝑎.

Solution: As illustrated in Figure 4.20(a), we introduce


a rectangular coordinate system in which the 𝑦-axis
passes through the apex and is perpendicular to the base,
and the 𝑥-axis passes through the base and is parallel to
a side of the base.

At any y in the interval [0, ℎ] on the 𝑦-axis, the cross


section perpendicular to the 𝑦-axis is a square. If 𝑠
denotes the length of a side of this square, then by similar
triangles (Figure 4.20(b)).

1
𝑠
2 =ℎ−𝑦 or 𝑠=
𝑎
(ℎ − 𝑦)
1 ℎ ℎ
𝑎
2

Thus, the area 𝐴(𝑦) of the cross section at 𝑦 is Fig. 4.20

𝑎2
𝐴(𝑦) = 𝑠 = 2 (ℎ − 𝑦)2
2

The volume is
𝑏 ℎ
𝑎2 𝑎2 ℎ
𝑉 = ∫ 𝐴(𝑥) 𝑑𝑥 = ∫ 2 (ℎ − 𝑦) 𝑑𝑥 = 2 ∫ (ℎ − 𝑦)2 𝑑𝑥
2
𝑎 0 ℎ ℎ 0


𝑎2 ℎ 2
𝑎2 1 3
= − 2 ∫ −(ℎ − 𝑦) 𝑑𝑥 = − 2 [ (ℎ − 𝑦) ]
ℎ 0 ℎ 3 0

𝑎2 1 1
= − 2 [ (0 − ℎ3 )] = 𝑎2 ℎ. ∎
ℎ 3 3

157
Chapter 4: APPLICATIONS OF INTEGRATIONS
4.2.2 Solids of Revolution: The Disk Method

The solid generated by rotating (or revolving) a plane region about an axis in its
plane is called a solid of revolution. To find the volume of a solid like the one shown
in Figure 4.21, we need only observe that the cross-sectional area 𝐴(𝑥) is the area
of a disk of radius 𝑅(𝑥), the distance of the planar region’s boundary from the axis
of revolution. The area is then

𝐴(𝑥) = 𝜋(radius)2 = 𝜋[𝑅(𝑥)]2

So, the definition of volume in this case gives

Volume by Disks for Rotation About the x-axis


𝑏 𝑏
𝑉 = ∫ 𝐴(𝑥) 𝑑𝑥 = ∫ 𝜋[𝑅(𝑥)]2 𝑑𝑥.
𝑎 𝑎

This method for calculating the volume of a solid of revolution is often called the
disk method because a cross-section is a circular disk of radius 𝑅(𝑥).

Example 18: The region between the curve 𝑦 = √𝑥, 0 ≤ 𝑥 ≤ 4, and the 𝑥-axis is

revolved about the 𝑥-axis to generate a solid. Find its volume.

Solution:

158
Chapter 4: APPLICATIONS OF INTEGRATIONS
𝑏
𝑉 = ∫ 𝜋[𝑅(𝑥)]2 𝑑𝑥
𝑎

4 4
2
= ∫ 𝜋[√𝑥] 𝑑𝑥 = 𝜋 ∫ 𝑥 𝑑𝑥
0 0

4
𝑥2 (4)2
= 𝜋[ ] = 𝜋( − 0)
2 0 2

∴ 𝑉 = 8𝜋.

Fig. 4.21

Example 19: The circle 𝑥 2 + 𝑦 2 = 𝑎2 is rotated about the 𝑥-axis to generate a


sphere. Find its volume.

Solution: The cross-sectional area at a typical point x between −a and a is

𝐴(𝑥) = 𝜋𝑦 2 = 𝜋(𝑎2 − 𝑥 2 )

𝑅(𝑥) = √𝑎2 − 𝑥 2 for rotation about 𝑥-axis

Therefore, the volume is


𝑏
𝑉 = ∫ 𝜋[𝑅(𝑥)]2 𝑑𝑥
𝑎

𝑎
= ∫ 𝜋(𝑎2 − 𝑥 2 ) 𝑑𝑥
−𝑎

𝑎 Fig. 4.22
2
𝑥3
= 𝜋 [𝑎 𝑥 − ]
3 −𝑎

𝑎
𝑎3 𝑎3 4
= 𝜋 [(𝑎 − ) − (−𝑎 + )] = 𝜋𝑎3 .
3 3

3 3 −𝑎 3

159
Chapter 4: APPLICATIONS OF INTEGRATIONS
Example 20: Find the volume of the solid generated by revolving the region

bounded by 𝑦 = √𝑥 and the lines 𝑦 = 1, 𝑥 = 4 about the line 𝑦 = 1.

Solution:

Fig. 4.23
𝑏
𝑉 = ∫ 𝜋[𝑅(𝑥)]2 𝑑𝑥
𝑎

4 4
2
= ∫ 𝜋[√𝑥 − 1] 𝑑𝑥 = ∫ 𝜋[𝑥 − 2√𝑥 + 1] 𝑑𝑥
1 1

4
𝑥2 2 3 7
= 𝜋 [ − 2 ∙ 𝑥 2 + 𝑥] = 𝜋.
2 3 1
6

To find the volume of a solid generated by revolving a region between the 𝑦-axis
and a curve 𝑥 = 𝑅(𝑦), 𝑐 ≤ 𝑦 ≤ 𝑑, about the 𝑦-axis, we use the same method with 𝑥
replaced by 𝑦. In this case, the area of the circular cross-section is

𝐴(𝑦) = 𝜋(radius)2 = 𝜋[𝑅(𝑦)]2 ,


and the definition of volume gives

160
Chapter 4: APPLICATIONS OF INTEGRATIONS
Volume by Disks for Rotation About the y-axis
𝑑 𝑑
𝑉 = ∫ 𝐴(𝑦) 𝑑𝑦 = ∫ 𝜋[𝑅(𝑦)]2 𝑑𝑦.
𝑐 𝑐

Example 21: Find the volume of the solid generated by revolving the region
between the 𝑦-axis and the curve 𝑥 = 2⁄𝑦, 1 ≤ 𝑦 ≤ 4, about the 𝑦-axis.

Solution:

Fig. 4.24
𝑑
𝑉 = ∫ 𝜋[𝑅(𝑦)]2 𝑑𝑦
𝑐

4
22 4
4
= ∫ 𝜋 [ ] 𝑑𝑦 = 𝜋 ∫ 2 𝑑𝑦
1 𝑦 1 𝑦

14 3
= 4𝜋 [− ] = 4𝜋 ( ) = 3𝜋.
𝑦1 4

161
Chapter 4: APPLICATIONS OF INTEGRATIONS
Example 22: Find the volume of the solid generated by revolving the region
between the parabola 𝑥 = 𝑦 2 + 1 and the line 𝑥 = 3 about the line 𝑥 = 3.

Solution:

Fig. 4.25

𝑑
𝑉 = ∫ 𝜋[𝑅(𝑦)]2 𝑑𝑦
𝑐

Radius 𝑅(𝑦) = 3 − (𝑦 2 + 1) for rotation around axis 𝑥 = 3.

√2 √2
2 ]2 [4 − 4𝑦 2 + 𝑦 4 ] 𝑑𝑦
𝑉=∫ 𝜋[2 − 𝑦 𝑑𝑦 = 𝜋 ∫
−√2 −√2

√2
𝑦3 𝑦5 64𝜋√2
= 𝜋 [4𝑦 − 4 + ] = .
3 5 − 15
√2

162
Chapter 4: APPLICATIONS OF INTEGRATIONS
4.2.3 Solids of Revolution: The Washer Method

If the region we revolve to generate a solid does not border on or cross the axis of
revolution, the solid has a hole in it (Figure 4.26). The cross-sections perpendicular
to the axis of revolution are washers (the purplish circular surface in Figure 4.26)
instead of disks. The dimensions of a typical washer are

Outer radius: 𝑅(𝑥)

Inner radius: 𝑟(𝑥)

The washer’s area is

𝐴(𝑥) = 𝜋[𝑅(𝑥)]2 − 𝜋[𝑟(𝑥)]2 = 𝜋([𝑅(𝑥)]2 − [𝑟(𝑥)]2 )

Consequently, the definition of volume in this case gives


Volume by Washers for Rotation About the x-axis
𝑏 𝑏
𝑉 = ∫ 𝐴(𝑥) 𝑑𝑥 = ∫ 𝜋([𝑅(𝑥)]2 − [𝑟(𝑥)]2 ) 𝑑𝑥.
𝑎 𝑎

This method for calculating the volume of a solid of revolution is called the washer

method because a thin slab of the solid resembles a circular washer of outer radius
𝑅(𝑥) and inner radius 𝑟(𝑥).

Fig. 4.26

163
Chapter 4: APPLICATIONS OF INTEGRATIONS
Example 23: The region bounded by the curve 𝑦 = 𝑥 2 + 1 and the line
𝑦 = −𝑥 + 3 is revolved about the 𝑥-axis to generate a solid. Find the volume of the
solid.

Solution:

Outer radius: 𝑅(𝑥) = −𝑥 + 3

Inner radius: 𝑟(𝑥) = 𝑥 2 + 1

Find the limits of integration by finding the 𝑥-


coordinates of the intersection points of the curve
and line in Figure 4.27(a).

𝑥 2 + 1 = −𝑥 + 3

𝑥2 + 𝑥 − 2 = 0

(𝑥 + 2)(𝑥 − 1) = 0

𝑥 = −2, 𝑥=1

Therefore, the volume is


𝑏
𝑉 = ∫ 𝜋([𝑅(𝑥)]2 − [𝑟(𝑥)]2 ) 𝑑𝑥
𝑎

1
𝑉 = ∫ 𝜋((−𝑥 + 3)2 − (𝑥 2 + 1)2 ) 𝑑𝑥 Fig. 4.27
−2

1
= 𝜋 ∫ (8 − 6𝑥 − 𝑥 2 − 𝑥 4 ) 𝑑𝑥
−2

1
𝑦2 𝑦3 𝑦5 117 𝜋
= 𝜋 [8𝑥 − 6 − − ] = .
2 3 5 −2 5

164
Chapter 4: APPLICATIONS OF INTEGRATIONS
Example 24: The region bounded by the parabola 𝑦 = 𝑥 2 and the line 𝑦 = 2𝑥 in
the first quadrant is revolved about the 𝑦-axis to generate a solid. Find the volume
of the solid.

Solution:

Outer radius: 𝑅(𝑦) = √𝑦

Inner radius: 𝑟(𝑦) = 𝑦⁄2

The line and parabola intersect at 𝑦 = 0 and 𝑦 = 4, so


the limits of integration are 𝑐 = 0 and 𝑑 = 4. We
integrate to find the volume:
𝑑
𝑉 = ∫ 𝜋([𝑅(𝑦)]2 − [𝑟(𝑦)]2 ) 𝑑𝑦
𝑐

4
𝑦 2
2
𝑉 = ∫ 𝜋 ((√𝑦) − ( ) ) 𝑑𝑦
0 2

4
𝑦2
= 𝜋 ∫ (𝑦 − ) 𝑑𝑦
0 4

4
𝑦2 𝑦3 8𝜋
= 𝜋[ − ] = .
2 12 0 3

Fig. 4.28

165
Chapter 4: APPLICATIONS OF INTEGRATIONS
Exercises (4)

4.1. Find the area bounded by the curve 𝑦 = 2𝑥 2 + 3 and the 𝑥- axis between
𝑥 = −1 and 𝑥 = 3.
4.2. Find the area bounded by the curve 𝑦 = 𝑥 and the 𝑥- axis and the lines
𝑥 = 0 and 𝑥 = 2.
4.3. Find the area bounded by the curve 𝑦 = 𝑥 2 − 4 and the 𝑥- axis and the
lines 𝑥 = −2 and 𝑥 = 2.
4.4. Find the area enclosed between the curve 𝑦 = 𝑥(1 − 𝑥)(1 + 𝑥) and 𝑥-
axis.
4.5. Find the area bounded by the curves 𝑦 = √𝑥 and 𝑦 = 𝑥 2 .
4.6. Find the area bounded by the curves 𝑦 = 2𝑥 2 + 10 and 𝑦 = 4𝑥 + 16.
4.7. Find the area bounded by the curves 𝑦 = 𝑥 2 and 𝑦 = 𝑥 + 6.
4.8. Find the area bounded by the curves 𝑦 2 − 2𝑥 − 6 = 0 and 𝑥 − 𝑦 − 1 = 0.
4.9. Find the area bounded by the curves 𝑦 = 𝑒 𝑥 , 𝑦 = 𝑒 2𝑥 and 𝑦 = 7.
4.10. Determine the volume of the solid generated by rotating the region
bounded by 𝑦 = 𝑥 3 + 1, 𝑥 = 0, 𝑥 = 3 and the 𝑥-axis about 𝑥-axis.
4.11. Determine the volume of the solid generated by rotating the region
bounded by 𝑦 = 𝑥 2 − 4𝑥 + 5, 𝑥 = 1, 𝑥 = 4 and the 𝑥-axis about 𝑥-axis.
4.12. Determine the volume of the solid generated by rotating the region
bounded by 𝑦 = 𝑥 3 , 𝑦 = 0 and 𝑦 = 4 about 𝑦-axis.
4.13. Find the volume of the solid generated by rotating the region bounded by
𝑦 = √𝑥 and 𝑦 = 𝑥 2 about 𝑥-axis.
4.14. Find the volume of the solid generated by rotating the region bounded by
3
𝑦 = √𝑥 and 𝑦 = 𝑥 ⁄4 that lies in the first quadrant about 𝑦-axis.

166
Chapter 4: APPLICATIONS OF INTEGRATIONS
4.15. Find the volume of the solid generated by rotating the region bounded by
𝑦 = 𝑥 and 𝑦 = 𝑥 2 about the line 𝑦 = 2.
4.16. Determine the volume of the ellipsoid obtained when the graph of the
ellipse

𝑥2 𝑦2
+ =1
𝑎2 𝑏 2

is revolved about the 𝑥-axis.

4.17. Find the volume of the cylinder of radius 𝑟 and height ℎ.


4.18. Find the volume of the cone of height ℎ and bottom radius 𝑟.
4.19. Find the volume of the frustum of a right circular cone with height ℎ, lower
base radius 𝑅 and top radius 𝑟.

167
Chapter 4: APPLICATIONS OF INTEGRATIONS
Notes:

168
Chapter 4: APPLICATIONS OF INTEGRATIONS

169
Chapter 4: APPLICATIONS OF INTEGRATIONS

170
Bibliography

1. Anton, Howard, Irl C. Bivens, and Stephen Davis. Calculus. John Wiley &
Sons, 2021.
2. Anton, Howard, et al. Calculus early transcendentals. Chichester, 2013.
3. Courant, Richard, et al. Introduction to calculus and analysis. Vol. 1. New
York: Interscience Publishers, 1965.
4. Falduto, Valeree, et al. "Algebra and trigonometry." (2015).
5. Spivak, Michael. Calculus. Cambridge University Press, 2006.
6. Thomas Jr, George B., et al. "Early TraNSCENDENTalS."
(2014).McCormick, M.E., 2009. Ocean engineering mechanics: with
applications. Cambridge University Press.

171

You might also like

pFad - Phonifier reborn

Pfad - The Proxy pFad of © 2024 Garber Painting. All rights reserved.

Note: This service is not intended for secure transactions such as banking, social media, email, or purchasing. Use at your own risk. We assume no liability whatsoever for broken pages.


Alternative Proxies:

Alternative Proxy

pFad Proxy

pFad v3 Proxy

pFad v4 Proxy